Методические рекомендации по практическим работам ПМ 01
Оценка 4.6

Методические рекомендации по практическим работам ПМ 01

Оценка 4.6
doc
11.11.2021
Методические рекомендации по практическим работам ПМ 01
Методические рекомендации по практическим работам ПМ.01 Техническое обслуживание и ремонт тс.doc

Департамент образования

Ямало-Ненецкого автономного округа

государственное бюджетное профессиональное

образовательное учреждение

Ямало-Ненецкого автономного округа

«Новоуренгойский  многопрофильный колледж»

 

 

 

РАССМОТРЕНО И ОДОБРЕНО

на заседании предметной

(цикловой) комиссии

технических дисциплин

Председатель П(Ц)К

___________________

 

 

 

 

УТВЕРЖДЕНО

на заседании научно-методического совета

Председатель НМС

_________________

Зам.директора по УР

_________________

 

 

 

 

 

 

 

 

 

 

 

 

МЕТОДИЧЕСКИЕ РЕКОМЕНДАЦИИ

 

для выполнения практических работ

 

по дисциплине ПМ.01 Техническое обслуживание и ремонт автотранспорта

программы подготовки специалистов среднего звена

по профессии 23.01.03 Автомеханик

 

 

 

 

 

 

 

 

Новый Уренгой 2020

Методические рекомендации для обучающихся по выполнению практических работ являются частью основной профессиональной программы ГБПОУ ЯНАО «НУРМК» по профессии 23.01.03 Автомеханик в соответствии с требованиями ФГОС третьего поколения и рабочих программ по ПМ.01 Техническое обслуживание и ремонт автотранспорта.

Методические рекомендации включают в себя учебную цель, перечень образовательных результатов, (заявленных в рабочей программе),  задачи, обеспеченность занятия, краткие теоретические и учебно-методические материалы по теме, вопросы для закрепления теоретического материала, задания для практической работы, инструкцию по ее выполнению, образец отчета о проделанной работе, критерии оценивания.

 

РАЗРАБОТЧИК:

Симакова Анна Алексеевна, мастер производственного обучения, преподаватель профессионального цикла, первая квалификационная категория

 

 

Данные методические рекомендации являются собственностью ГБПОУ ЯНАО «Новоуренгойский многопрофильный колледж»

 

 

 

 

 

 

 

 

 

 

 

 

 

 

 

 

ПЕРЕЧЕНЬ ПРАКТИЧЕСКИХ РАБОТ
ПО МДК.01.01 Слесарное дело и технические измерения

 

1.    Организация труда на рабочем месте и безопасность труда.

2.    Разметочный инструмент.

3.    Инструмент, применяемый при правке и гибке  металлов.

4.    Инструмент, приспособления применяемые при рубке.

5.    Инструмент, оборудование применяемые при резке.

6.    Инструмент, применяемый при опиливании.

7.    Инструмент, применяемый при сверлении, зенкеровании и развертывании отверстий.

8.    Инструмент, применяемый при нарезании резьбы.

9.    Приемы, применяемые при ручной  клепке, Приемы, применяемые при машинной клепке.

10.  Инструмент, применяемый при шабрении и притирке.

11.  Разборка и сборка разъемных соединений, Сборка неразъемных соединений.

 

 

 

ПЕРЕЧЕНЬ ПРАКТИЧЕСКИХ РАБОТ
ПО МДК.01.02.01  Устройство, техническое обслуживание и ремонт автомобилей категории "С"

 

1.      Технологическая карта по трансмиссии.

2.      Технологическая карта по рулевому управлению автомобиля.

3.      Технологическая карта по тормозной системе автомобиля.

 

 

 

ПЕРЕЧЕНЬ ПРАКТИЧЕСКИХ РАБОТ
ПО МДК.01.03   Ремонт транспортных средств

 

 

1.      Способы восстановления посадок и взаимного расположения деталей и сборочных единиц.

2.      Ремонт деталей и сопряжений  полимерными материалами.

3.      Мойка, чистка и разборка.

4.      Дефектовочно-комплектовочные работы.

5.      Восстановление деталей методами гальванических и химических покрытий.

6.      Ремонт деталей системы охлаждения.

7.      Ремонт деталей системы смазки.

8.      Разборка двигателя.

9.      Сборка двигателя.

10.  Ремонт сцепления.

11.  Ремонт коробки передач и раздаточной коробки.

12.  Ремонт карданной передачи.

13.  Ремонт ведущих мостов.

14.  Монтаж  и демонтаж шин.

15.  Основные неисправности и способы устранения рулевого управления  с гидроусилителем.

16.  Основные неисправности и способы устранения тормозной системы с пневмоприводом.

17.  Основные неисправности и способы устранения тормозной системы с гидроприводом.

18.  Ремонт кузова, кабины и оперения.

19.  Испытание автомобиля после ремонта.

20.  Основы технологии сборки автомобиля.

21.  Ремонт механизмов подъема платформы автомобиля-самосвала.

 

 

 

 

 

 

 

 

 

 

 

 

 

 

 

 

 

 

 

 

 

 

 

 

 

 

 

 

 

 

 

 

 

 

 

УВАЖАЕМЫЙ СТУДЕНТ!

 

            Методические указания по ПМ.01 Техническое обслуживание и ремонт автотранспорта для выполнения практических  работ созданы Вам в помощь для работы на занятиях, подготовки к практическим работам.

             Приступая к выполнению практической работы, Вы должны внимательно прочитать цель и задачи занятия, ознакомиться с краткими теоретическими и учебно-методическими материалами по теме практической работы, ответить на вопросы для закрепления теоретического материала.

            Все задания к практической Вы должны выполнять в соответствии с инструкцией, анализировать полученные  результаты по приведенной методике.

            Отчет о практической  Вы должны выполнять по приведенному алгоритму, опираясь на образец.

            В случае Вашего отсутствия на уроке по любой причине или получения неудовлетворительной оценки за практическую работу Вы должны выполнить ее самостоятельно  и сдать преподавателю.

           

            Если в процессе подготовки к практической или при решении задач у Вас возникают вопросы, разрешить которые самостоятельно не удается, необходимо обратиться к преподавателю для получения разъяснений или указаний в дни консультаций.

 

 

 

Желаем Вам успехов!

 

 

 

 

 

 

 

 

 

 

 

 

 

Введение

Содержанием данного документа являются методические указания и индивидуальные задания для выполнения практических работ по дисциплине профессионального цикла ПМ.01 Техническое обслуживание и ремонт автотранспорта.

Данные методические указания предназначены для использования на практических занятиях обучающихся. При работе с ним обучающиеся имеют возможность сформировать общие и профессиональные компетенции,  закрепить знания и умения, полученные на учебных занятиях. Приведенные вопросы для самоконтроля помогут обучающимся проверить степень усвоения изученного теоретического материала.

    В результате освоения дисциплины обучающийся должен уметь:

·               выполнять метрологическую проверку средств измерений;

·               выбирать и пользоваться инструментами и приспособлениями для                                         слесарных работ; 

·               снимать и устанавливать агрегаты и узлы автомобиля;

·               определять неисправности и объем работ по их устранению и ремонту;

·               определять способы и средства ремонта;

·               применять диагностические приборы и оборудование;

·               использовать специальный инструмент, приборы, оборудование;

·               оформлять учетную документацию.

В результате освоения дисциплины обучающийся должен знать:

·               средства метрологии, стандартизации и сертификации;

·               методы обработки автомобильных деталей;

·               устройство и конструктивные особенности обслуживаемых автомобилей;

·               назначение и взаимодействие основных узлов ремонтируемых автомобилей;

·               технические условия на регулировку и испытание отдельных механизмов;

·               виды и методы ремонта;

·               способы восстановления деталей.

 

Критерии оценки результатов выполнения работ:

 

Оценка «отлично» - работа выполнена в полном объеме, отчет оформлен в соответствии с требованиями стандартов, аккуратно и сдан своевременно.

Оценка «хорошо» - работа выполнена в полном объеме, отчет оформлен с незначительными отклонениями от требований стандартов и сдан несвоевременно.

Оценка «удовлетворительно» - работа выполнена не в полном объеме, отчет оформлен с отклонениями от требований стандартов и сдан несвоевременно.

Оценка «неудовлетворительно» - работа выполнена очень слабо, отчет оформлен со значительными отклонениями от требований стандартов и сдан несвоевременно.

 

 

 

 

 

 

 

 

 

 

 

 

Организация и порядок проведения практических работ

Тематика и очередность выполнения практических работ определяется программой курса и сообщается преподавателем на первом занятии группы.

Практические работы выполняются в соответствии с расписанием учебных занятий. Работа студентов на рабочем месте производится в соответствии с методическими указаниями к каждой практической работе. Студент должен быть подготовлен к выполнению очередной практической работе, изучив необходимый материал учебных и методических пособий.

По всем практическим работам оформляются отчеты. Отчет по практической работе составляется каждым студентом самостоятельно.

Все отчеты выполняются в одной специально отведенной тетради. Отчет выполняется в течение практического занятия и при необходимости оформляется за счет самостоятельной работы. Выполненный отчет представляется на следующее занятие.

В начале каждого отчета указывается тема работы, приводится цель и ход работы.

Общий зачет по практическим работам выставляется студенту после выполнения им всех работ, оформления и защиты отчетов. Форма проведения зачета – собеседование по всем темам практических занятий.

Прежде чем приступить к выполнению практической работы, не­обходимо: тщательно изучить содержание работы и порядок ее вы­полнения; повторить теоретический материал; подготовить табли­цы для занесения результатов наблюдений и вычислений. Студент должен иметь отдельную рабочую тетрадь для записей, необходи­мых для составления отчета о проделанной работе. Чтобы избежать возможных ошибок при чтении принципиальных схем, нужно знать условные обозначе­ния и буквенные коды электротехнических элементов и устройств, соответствующих действующему стандарту.

Практические работы выполняются бригадами, обычно из 2 — 5 человек. При завершении работы студенты составляют отчет. Практическая работа засчитывается, если отчет соответствует предъявля­емым требованиям и если студент ответил на вопросы преподава­теля. При этом студент должен знать устройство и принцип работы объекта исследования, назначение всех элементов схемы и пони­мать физические процессы, объясняющие полученные результаты, а также уметь объяснить порядок действий при выполнении любо­го эксперимента в практической работе.

Практические работы рассчитаны на 1-4 академических часа

 

 

 

 

 

 

 

 

 

 

 

 

 

 

 

Практические работы по МДК.01.01 Слесарное дело и технические измерения

 

 Практическая работа № 1

 

Цель:

1.Изучить основные требования к планировке рабочих мест.

2.Упражнение в выборе слесарных инструментов.

3. Научиться по данному заданию обработки детали выбирать необходимый слесарный инструмент.

Задание и методические рекомендации

1. Изучить основные теоретические положения и кратко их изложить по предложенной форме.

Содержание отчета

1.Наименование и цель работы.

2.Оборудование и материалы, используемые в работе.

3.Основные положения по теоретическому материалу(кратко).

4.Описание экспериментальной части работы и полученные результаты с необходимым графическим материалом, анализом, выводами.

5.Обоснованное решение указанной преподавателем задачи.

Содержание работы

hello_html_m1f2a7d78.jpg

Рабочее место –часть производственной площади цеха, участка или мастерской, которая закрепляется за определенным работником для выполнения определенного вида работ и должна быть оснащена оборудованием, приспособлениями, инструментами и материалами, необходимыми для их проведения.

При работе в слесарной мастерской необходима правильная организация рабочего места, которая улучшает условия труда и снижает опасность травматизма.

К организации рабочего места слесаря предъявляется ряд требований:

hello_html_ad7650d.jpg

Одноместный слесарный верстак –основным оборудованием рабочего места слесаря является как правило, одноместный слесарный верстак с установленными на нем тисками.

Слесарный верстак должен быть прочным и устойчивым, верстак состоит из стального каркаса, выполненного из труб или профильного проката (уголка). На каркасе установлена столешница, изготовленная из дерева твердых пород и покрыта листовой сталью толщиной 1…2мм. По периметру столешница окантована бортиком из стального уголка.

hello_html_m2fd18144.jpghello_html_m37297f4a.jpg5hello_html_7e3f1147.jpg

Под столешницей расположены выдвижные ящики для хранения инструментов, мелких деталей и технической документации. Для обеспечения удобства работы на верстаке располагаются планшет для режущих инструментов (чертилки, кернеры, зубила, напильники и т.д.) и инструментальная полка для измерительных инструментов.

Высота верстака должна соответствовать росту работающего. Если высота тисков не соответствует росту работающего, их регулируют винтом подъёма или на полу укладывают деревянную решетку, которая должна плотно прилегать к полу и не скользить. Для защиты работников от возможного травматизма при выполнении операций, связанных с образованием стружки, на верстаке устанавливается сменный защитный экран из сетки или органического стекла. При размещении инструментов на верстаке необходимо учитывать частоту их использования в процессе обработки и располагать инструменты таким образом, чтобы обеспечить удобный доступ к ним.

hello_html_584112e2.jpg

Стуловые тиски имеют весьма ограниченную область применения. Они предназначены для выполнения тяжелых работ, связанных с большими ударными нагрузками, например, рубка, гибка, клепка.

hello_html_m56878fd8.jpg

Параллельные поворотные слесарные тиски наиболее распространенный тип тисков, применяемых при слесарной обработке. Параллельными тиски называются потому, что при перемещении подвижной губки она во всех положениях остается параллельной неподвижной губке.

hello_html_m39503c20.jpg

Параллельные тиски поворотного типа должны прочно и надежно крепиться к верстаку. Зажимать деталь в тисках надо только усилием рук, а не весом тела. Зажимая или освобождая детали из тисков, рычаг следует опускать плавно, не бросая его чтобы не произошел ушиб руки или ноги. Содержать тиски надо в частоте и исправности.

Подставку под ноги следует применять, когда высота тисков не соответствует росту учащегося. Высота верстака с тисками считается нормальной, если у стоящего прямо учащегося согнута в локтевом суставе под углом 90º, рука находится на уровне губок тисков при вертикальном положении её плечевой части.

hello_html_mac759c9.jpg

Во время работы спецодежда работающего должна быть аккуратной и чистой.

Халат или комбинезон не должны стеснять движений. Во время работы спецодежда всегда должна быть застегнута на все пуговицы, а рукава должны иметь застегивающиеся обшлага плотно охватывающие нижнюю локтевую часть руки. На голову обязательно должен быть надет головной убор (берет или косынка) под который необходимо тщательно убрать волосы.

Организация рабочего места слесаря.

Эффективность сборочного процесса во многом зависит от правильной организации рабочего места сборщика.

В понятие организации рабочего места включается ряд факторов, и прежде всего:

·                                 его планировка,

·                                 правильная расстановка и удобная конструкция верстаков,

·                                 зональное размещение на рабочем месте инструментов и приспособлений,

·                                 освещенность,

·                                 режим труда и др.

Планировка рабочего места должна обеспечить Высокую производительность работы при минимальных затратах сил и времени сборщика; максимальное использование производственной площади; удобство обслуживания сборочного процесса; соответствие правилам и Требованиям охраны труда и техники безопасности.

Рабочее место слесаря-сборщика должно быть оснащено всем необходимым оборудованием, инструментом) приспособлениями, подъемно-транспортными устройствами. В единичном производстве рабочее вместо слесаря-сборщика оснащается в основном, универсальным оборудованием и инструментом. В серийном — рабочее место оборудуется с учетом выполнения нескольких конкретных сборочных операций. В массовом производстве на рабочем месте выполняются одна или две сборочные операции и оно оснащается соответственно специализированным оборудованием, оснасткой и инструментом.

Кроме основного технологического оборудования, приспособлений и инструмента на рабочем месте слесаря-сборщика имеется оргоснастка, обычно располагаемая на верстаке. Это — тумбочки, этажерки, стеллажи, планшеты для технологической документации, даолы.

При планировании рабочего места все предметы труда и инструменты необходимо располагать с учетом предела досягаемости вытянутых рук и наиболее удобных зон движений, наименее утомительных положений рук, корпуса, головы и ног, пределов обзора в пространстве и т. д.

Для обеспечения симметричности движений рабочего все то, что ему необходимо брать левой рукой, должно быть расположено слева, а правой — справа. Предметы труда Следует располагать в строгом порядке и всегда в одних и тех же местах, тогда у рабочего вырабатывается автоматизм движений, что уменьшает утомляемость и напряжение в процессе работы.

При организации рабочего места необходимо стремиться тому, чтобы движения выполнялись в меньшем количестве плоскостей (лучше всего в одной плоскости). Рациональная планировка предполагает такое размещение предметов труда, при котором максимально используются обратные движения рук после перемещения сборочных единиц и инструментов.

Оргоснастка располагается таким образом, чтобы наиболее часто употребляемая в процессе рабочего дня была ближе к рабочему. Собираемые единицы должны находиться на рабочем месте в специальной таре. Детали, которые имеют индивидуальную укладку, обычно располагаются в той таре, в которой они доставляются к рабочему месту. Детали легче, доставать из тары, если ее расположить с наклоном в сторону оператора.

Детали средних размеров (до 5 кг), которые могут взгромоздить верстак, располагаются на стеллажах. Если детали имеют круглую или плоскую форму, то применяются стеллажи-скалы. Такие стеллажи изготовляются многоярусными с наклоном яруса в сторону рабочего. Более крупные детали располагаются на полу вблизи верстака  на невысоких стеллажах. Число стеллажей и их емкость должны обеспечивать бесперебойность комплектования рабочего в процессе сборки необходимыми деталями и сборочными единицами. Стеллажи размещаются с учетом кратчайшего пути при доставке деталей на сборочные базы. Расположение, сборочных единиц во многом зависит от типа производства и принятого порядка обеспечения ими рабочих мест.

В условиях единичного производства, когда на одном рабочем месте осуществляется сборка разных сборочных единиц или общая сборка изделий, поступающие на сборку детали группируются по сборочным единицам. Ближе к верстаку располагаются те детали, которые используются в начале сборки. Детали, необходимые к концу сборки, хранятся на стеллажах или в специальной Таре. Отдельно хранятся до начала сборки ранее собранные и опробованные узлы.

Для обеспечения высокой производительности труда и ритмичности сборки важное значение имеет обслуживание рабочих мест слесарей- сборщиков — бесперебойная подача на рабочее место сборочных единиц, обеспечение технической документацией, организация ухода за оборудованием, а также технический надзор за его эксплуатацией. В единичном и мелкосерийном производстве обслуживание рабочих мест производится слесарями-сборщиками, а в серийном и массовом— вспомогательными рабочими. В условиях массового производства обслуживание осуществляется по специальному графику, в серийном — согласовывается с запуском партии сборок, а в единичном и мелкосерийном — обеспечивает бесперебойность сборочных работ в течение более коротких отрезков времени.

Материально-техническое снабжение рабочих мест в сборочном производстве выполняется комплектовщиками. Инструментальную подготовку выполняют работники инструментальных групп цехов, мастера, бригадиры. Они получают на центральных складах и обеспечивают рабочие места режущими и измерительными инструментами, комплектуют рабочие места технологической оснасткой и приспособлениями. Наладочные работы на рабочих местах в условиях автоматизированных и механизированных производств осуществляют наладчики. В комплекс наладочных работ входят осмотр оборудования, подготовка инструмента и приспособлений к наладке, наладка оборудования, подналадка инструментов, апробация инструмента и оборудования после наладки.

К рабочему инструменту слесаря относят те инструменты, которыми непосредственно выполняют нужные операции при изготовлении деталей или их ремонте, сборке различных узлов и машин.

hello_html_67ed4e94.jpg

Ящик с набором слесарных инструментов: 1 - щетка, 2 - скребок для очистки напильников, 3 - отвертка, 4 - крейцмейсель, 5 - зубило, 6 - клупп, 7 - ножонка, 8 - клеши, 9 - плоскогубцы, 10 - разметочный циркуль, 11 - угольник 90°, 12 - линейка, 13 - штангенциркуль, 14 - чертилка, 15 - кернер, 16 - раздвижной ключ, 17 - накидной ключ для круглых гаек, 18 - ручные тиски,19 плоский драчевый напильник, 20, 22 - плоский и круглый личные напильники, 21 - трехгранный напильник, 23, 24 - шаберы, 25 - молоток.

На рисунке показан примерный набор универсального слесарного инструмента слесаря работающего вне мастерских. В мастерских, в ящиках слесарного стола, должен быть более разнообразный и специализированный инструмент для выполнения различных производственных заданий. Например: отвёртки, напильники, ключи гаечные, зубило и прочее, не по одному, а оп 2 - 4 и более разного размера, конфигурации и назначения.

 

 

 

 

Практическая  работа №2

Цель:

Изучение устройства штангенинструментов и приобретения навыков по их использованию для измерения деталей.

Приборы и материалы: Измерительные или контрольные инструменты и приборы – штангенциркули разных типов.

Измеряемое или контролируемое изделие – типовая деталь (вал, втулка, пластина и др.).

Ход работы:

1. Получить деталь и выполнить ее эскиз. На эскизе указать все необходимые размерные линии.

2. Выбрать необходимый штангенинструмент.

3. Пользуясь формулами b = a/n  = (gи L + a)/n ´ a, определить отсчет по нониусу и модуль штангенинструментов.

4. Определить действительные значения размеров. В случае, если для измерения используется штангенциркуль ШЦ-II, необходимо:

- открепить подвижную губку и хомутик, передвинуть их вдоль штанги и расположить рамку так, чтобы измеряемую деталь можно было установить между измерительными поверхностями губок;

- сняв штангенциркуль с детали, отсчитать показания по шкале штанги и по нониусу. При измерении внутренних размеров к полученному размеру следует прибавить размер толщины губок.

5. Проставить на эскизе детали измеренные размеры с точностью, которую позволяет выполнить имеющийся штангенинструмент.

6. Оформить отчет по прилагаемой форме.

Форма отчета

нониусу

 

Модуль

нониуса

 

Пределы

измерения

 

Вывод: в результате ПЗ обучающиеся освоили навык Измерение размеров детали.

 

Практическая работа №3

Цель работы:

Освоение и приобретение практических навыков слесарной операции – правки и гибки металлов.

Время выполнения:

90 минут

Гибка металлов по приемам работы и характеру рабочего процесса близка к правке. Сущность ее заключается в том, что одна часть заготовки перегибается по отношению к другой на какой-либо заданный угол. Напряжение изгиба должны превышать предел упругости, а деформация заготовки быть пластичной, Только в этом случае заготовка сохранит преданную ей форму после снятия нагрузки.

Ручную гибку производят в тисках с помощью слесарного молотка и различных приспособлений. Последовательность выполнения гибки зависит от размеров контура и материала заготовки.

Гибку тонколистового металла производят киянкой. При использовании для гибки металлов различных оправок их форма должна соответстовать форме профиля деталей с учетом деформации металла.

Выполняя гибку заготовки, важно правильно определить ее размеры. Расчет длины заготовки выполняют по чертежу с учетом радиусов всех изгибов. Для деталей, изгибаемых под прямым углом без закруглений с внутренней стороны, припуск заготовки на изгиб должен составлять от 0,6 до 0,8 толщины металла. При гибке надо учитывать. Что после снятия нагрузки угол изгиба несколько увеличится. Изготовление деталей с очень малыми радиусами изгиба связано с опасностью разрыва наружного слоя заготовки в месте изгиба.

Величина минимально допустимого радиуса изгиба зависит от механических свойств материала заготовки, от технологии к качества поверхности заготовки. Детали с малыми радиусами закруглений необходимо изготавливать из пластичных металлов или предварительно подвергать отжигу.

 

Правка и рихтовка представляют собой операции по выправке металла, заготовок и деталей, имеющих вмятины, выпучины, волнистость, коробление, искривления и др. Правка и рихтовка имеют одно и то же назначение, но отличаются приемами выполнения и применяемыми инструментами и приспособлениями.

Листовой материал и заготовки из него могут быть покороблены по краям и в середине, иметь изгибы и местные неровности в виде вмятин и выпучин различных форм. При рассмотрении деформированных заготовок можно заметить, что вогнутая сторона их короче выпуклой. Волокна на выпуклой стороне растянуты, а на вогнутой сжаты.

Металл подвергается правке как в холодном, так и в нагретом состоянии. Выбор способа зависит от величины прогиба, размеров и материала изделия.

Правка может выполняться ручным способом — на стальной или чугунной плите, или на наковальне — и машинным — на правильных вальцах, прессах.

Правильную плиту изготовляют достаточно массивной, масса ее не менее чем в 80—150 раз больше массы молотка. Правильные плиты изготовляют из стали, из серого чугуна монолитными или с ребрами жесткости.

Плиты бывают следующих размеров: 400 х 400; 750 х 1000; 1000 х 1500; 1500х 2000; 2000х 2000; 1500х 3000 мм. Рабочая поверхность плиты должна быть ровной и чистой. Устанавливают плиты на металлические или деревянные подставки, обеспечивающие кроме устойчивости и горизонтальность положения.

Рихтовальные бабки используют для правки (рихтовки) закаленных деталей, изготовляют их из стали и закаливают. Рабочая часть поверхности может быть цилиндрической или сферической радиусами 150-200 мм.

Молотки для правки применяют с круглым гладким полированным бойком. Молотки с квадратным бойком оставляют следы в виде забоин (квадратов, углов).

Для правки закаленных деталей (рихтовки) применяют молотки с радиусным бойком (массой 400 — 500 г) из стали У10. Хорошо зарекомендовали себя рихтовальные молотки, оснащенные твердым сплавом, корпус которых выполняют из стали У7 и У8. В рабочие концы молотка вставляют пластинки твердого сплава ВК8 и ВК6. Рабочую часть бойка затачивают и доводят по радиусу до 0,05-0,1 мм.

Молотки со вставными бойками из мягких металлов применяют при правке деталей с окончательно обработанной поверхностью и деталей или заготовок из цветных металлов и сплавов. Вставные бойки могут быть медные, свинцовые или деревянные.

Гладилки (деревянные или металлические бруски) применяют при правке тонкого листового и полосового металла.

hello_html_62372018.jpg

Рис. 3. Правильная плита (а), рихтовальные бабки (б)

hello_html_4076418e.jpg

Рис. 4. Рихтовальные молотки: а — с радиусным бойком, б — с круглым гладким полированным

hello_html_m5345323c.jpg

Рис. 5. Правка металла: а — проверка на глаз, б — момент правки

 

Правку и рихтовку стальных листовых и полосовых заготовок производят для придания им плоскостности. Основным инструментом и приспособлениями для правки и рихтовки деталей при сборочных работах являются молотки, правильные плиты и рихтовочные специальные бабки.

Для правки применяют молотки с круглым сферическим или гладким бойком для того, чтобы не оставлять забоин на заготовке. Для правки закаленных деталей (рихтовки) применяют молотки с радиусным бойком. При изготовлении рихтовочного молотка нужно в прорезь его корпуса плотно запрессовать пластинку из твердого сплава ВК6 или ВК8 и заточить ее радиусом 0,1 — 0,2 мм.

Для правки деталей с окончательно обработанной поверхностью и деталей из цветных металлов и сплавов применяют молотки со вставными бойками из мягких металлов. Вставные бойки могут быть медные, свинцовые и деревянные в зависимости от материала обрабатываемой детали. Правке или рихтовке подвергают детали как в холодном, так и в нагретом состоянии. Выбор приемов и способов правки и рихтовки зависит от величины прогиба, размеров и материала детали.

Правку можно выполнять ручным способом на стальной или чугунной плите или наковальне и машинным — на правильных вальцах.

Правильную плиту изготовляют из стали, серого чугуна, монолитной или с ребрами жесткости, и обязательно устанавливают на массивной деревянной опоре (столе) для того, чтобы в процессе правки поглощался шум. Рабочая поверхность плиты должна быть ровной и чистой. Плиту размером свыше 750X 1000 мм устанавливают на деревянных столах, которые должны обеспечить устойчивость и горизонтальность ее положения.

Рихтовочные плиты используют для правки (рихтовки) закаленных деталей, изготовляют их из стали и термически обрабатывают, затем поверхность шлифуют.

hello_html_m323c47a7.jpg

Рис. 6. Приемы рихтовки заготовок из листового (а), полосового (б, в) и пруткового (г) материала

 

Детали обычно правят вручную с помощью различных инструментов и приспособлений, реже для этой цели применяют гидравлические прессы. При правке и рихтовке независимо от формы и толщины детали необходимо надевать рукавицы.

Детали из листового материала (для трактора, автомобиля, комбайна и др.) могут быть покороблены по краям и в середине, иметь изгибы и местные неровности в виде вмятин и выпучин различной формы. При рассмотрении деформированных деталей можно заметить, что вогнутая сторона их короче выпуклой. Волокна на выпуклой стороне растянуты, а на вогнутой сжаты. Для того чтобы выровнять поверхность детали, нужно положить ее на плиту, установленную на специальном деревянном столе. Правку листового материала толщиной от 0,5 до 1,2 мм производят перекрестным способом. Удары молотком должны быть частые и легкие их надо наносить от края к центру выпучины. При этом левой рукой придерживая деталь, все время перемещают ее по плите и меняют место нанесения удара молотком.

Выпуклые участки обводят мелом или мягким графитовым карандашом, затем заготовку кладут на плиту выпуклыми участками вверх, так, чтобы края ее не свешивались, а лежали полностью на опорной поверхности плиты. Правку начинают с ближайшего к вмятине края, по которому наносят ряд ударов молотком в пределах, отмеченных мелом. Затем наносят удары по другому краю. После этого по первому краю наносят повторные удары, указанные точками на детали 3, и переходят опять ко второму краю и так до тех пор, пока постепенно не приблизятся к середине. Удары молотком наносят часто, но не сильно, особенно перед окончанием правки. Не допускают несколько ударов по одному и тому же месту, так как это может привести к образованию новой неровности.

Под ударами молотка материал вокруг выпуклого места вытягивается и постепенно выравнивается. Если на поверхности детали на небольшом расстоянии друг от друга имеется несколько выпуклых мест, под ударами молотка выпуклости соединяются в одну, которую правят молотком вокруг ее границ.

Тонкие листы правят легкими деревянными молотками (киянками) или молотками с деревянными наконечниками, медными, латунными или свинцовыми молотками, а очень тонкие листы выглаживают на ровной плите металлическими или деревянными брусками.

Правку полосового металла осуществляют в следующем порядке. Изгиб отмечают мелом, после чего надевают на левую руку рукавицу, в правую берут молоток и в левую полосу. Полосу располагают на плите так, чтобы она лежала выпуклостью вверх, соприкасаясь с плитой в двух точках. Наносят удары по выпуклым частям, регулируя силу удара в зависимости от толщины полосы и величины кривизны (чем больше искривление и чем толще полоса, тем сильнее удары). По мере выправления полосы силу удара уменьшают и чаще поворачивают полосу с одной стороны на другую до полного выправления. При нескольких выпуклостях сначала выправляют крайние, а затем средние.

Результаты правки (прямолинейность заготовки) проверяют на глаз, а при необходимости более точного контроля — на разметочной плите по просвету или наложением линейки на полосу.

После закалки стальные детали иногда коробятся. Правку закаленных деталей называют рихтовкой. В зависимости от характера рихтовки применяют различные молотки с закаленным бойком или специальные рихтовочные молотки с закругленной узкой стороной бойка. Деталь при этом лучше располагать не на плоской плите, а на рихтовочную бабке с выпуклой поверхностью. Удары наносят рихтовочным молотком не по выпуклой, а по вогнутой стороне поверхности полосовой детали 3. Прежде чем приступить к рихтовке детали, нужно в первую очередь определить место изгиба. Затем надеть на левую руку рукавицу и положить деталь на рихтовочную бабку, установленную на деревянном столе. После этого рихтовочным молотком 4 наносят легкие удары по вогнутой поверхности детали, при этом деталь перемещают вверх и вниз по поверхности бабки и периодически проверяют прямолинейность лекальной линейкой или на контрольной плите с помощью щупа.

При правке заготовок из прутка на выпуклой стороне мелом отмечают границы изгибов. Затем надевают на левую руку рукавицу, в правую берут молоток и укладывают пруток на наковальню выпуклостью вверх. Удары молотком наносят по выпуклой части от краев изгиба к середине. Сила удара зависит от диаметра прутка и величины изгиба. По мере выправления изгиба силу удара уменьшают, заканчивая правку легкими ударами при поворачивании прутка вокруг оси. Если пруток имеет несколько изгибов, сначала правят крайние, а затем расположенные в середине.

hello_html_m9092d79.jpg

Рис. 7. Рихтовка заготовок квадратного сечения

Детали квадратного сечения правят в такой последовательности: выявляют покоробленные участки, устанавливают места, где имеется больше вогнутых и выпуклых изгибов, и приступают к правке или рихтовке так, как было указано выше.

На рис. 5, а показан способ рихтовки штанги квадратного сечения с установленными на ней ножами на круглой бабке рихтовочным молотком. Перед рихтовкой надевают на левую руку рукавицу и захватывают конец штанги, после чего укладывают штангу на поверхность бабки так, чтобы вогнутая поверхность находилась в центре металлического рихтовочного диска бабки. Затем бойком рихтовочного молотка наносят легкие поперечные удары по поверхности штанги, а левой рукой перемещают штангу вперед и назад по поверхности диска до тех пор, пока не выровняются плоскости штанги.

Если ножи штанги не параллельны друг другу, ее устанавливают в тиски между планками и на вилку стойки. Вилку закрепляют гайкой, а губки тисков зажимают так, чтобы штанга не провисала. Затем правой рукой снизу на штангу надевают специальный рычаг (плечо рычага 500 мм). Придерживая штангу левой рукой, правой рукой нажимают на конец рычага и выравнивают штангу так, чтобы ножи ее стали параллельны друг другу. Освободив штангу из тисков, устанавливают ее на вилку на уровне глаз и закрепляют гайкой. Затем проверяют параллельность расположения ножей.

 

Практическая работа №4

 

Цель работы:

Освоение и приобретение практических навыков слесарной операции – рубки металлов.

Время выполнения:

90 минут

Рубкой называется операция, при которой с помощью зубила и слесарного молотка с заготовки удаляют слои металла или разрубают заготовку.

Физической основой рубки является действие клина, форму которого имеет рабочая (режущая) часть зубила. Рубка применяется в тех случаях, когда станочная обработка заготовок трудно выполнима или нерациональна.

С помощью рубки производится удаление (срубание) с заготовки неровностей металла, снятие твердой корки, окалины, острых кромок детали, вырубание пазов и канавок, разрубание листового металла на части.

Рубка производится, как правило, в тисках. Разрубание листового материала на части -может выполняться на плите. Основным рабочим (режущим) инструментом при рубке является зубило, а ударным — молоток.

Слесарное зубило изготовляется из инструментальной углеродистой стали. Оно состоит из трех частей: ударной, средней и рабочей. Ударная часть выполняется суживающейся кверху, а вершина ее (боек) —закругленной; за среднюю часть зубило держат во время рубки; рабочая (режущая) часть имеет клиновидную форму.

Угол заострения выбирается в зависимости от твердости обрабатываемого материала. Для наиболее распространенных материалов рекомендуются следующие углы заострения: для твердых материалов (твердая сталь, чугун) — 70°; для материалов средней твердости (сталь) ~ 60°; для мягких материалов (медь, латунь) '— 45°; для алюминиевых сплавов — 35°.

Рабочая и ударная части зубила подвергаются термической обработке (закалке и отпуску). Степень закалки зубила можно определить, проведя напильником по закаленной части зубила: если напильник не снимает стружку, а скользит по поверхности, закалка выполнена хорошо.

Для вырубания узких пазов и канавок пользуются зубилом с узкой режущей кромкой — крейцмейселем. Такое зубило может применяться и для снятия широких слоев металла: сначала прорубают канавки узким зубилом, а оставшиеся выступы срубают широким зубилом. Для вырубания профильных канавок (полукруглых, двугранных и др.) применяются специальные крейцмейсели— канавочники, отличающиеся только формой режущей кромки.

Слесарные молотки, используемые при рубке металлов бывают двух типов: с круглым и с квадратным бойком. Основной характеристикой молотка является его масса. Для рубки металлов применяют молотки массой от 400 до 600 г.

Рубка металлов — операция очень трудоемкая. Для облегчения труда и повышения его производительности используют механизированные инструменты. Среди них наибольшее распространение имеет пневматический рубильный молоток. Он приводится в действие сжатым воздухом, который подается по шлангу от постоянной пневмосети или передвижного компрессора. Во время рубки пневматический рубильный молоток держат обеими руками: правой — за рукоятку левой — за конец ствола, и направляют зубило по линии рубки.

Рубка металла бывает горизонтальная и вертикальная в зависимости от расположения зубила во время операции. Горизонтальную рубку производят в тисках. При этом заднюю грань зубила устанавливают к плоскости губок тисков под углом не более 5°. Вертикальную рубку производят на плите или наковальне. Зубило устанавливают вертикально, а перерубаемый материал укладывают на плите горизонтально.

hello_html_4a302c7c.jpg

Рис. 1 Зубило слесарное

При рубке надо стоять у тисков устойчиво вполоборота к ним, корпус должен находиться левее от тисков, левая нога вперед, правая назад.

Молоток необходимо держать так, чтобы было расстояние 20-30 мм от ее конца. Держать зубило надо левой рукой не сжимая сильно пальцы, на расстоянии 20-30 мм от головки.

Наиболее производительной считается рубка. при которой за 1 подход снимается слой металла 1.5 -2 мм. Точность обработки достигаемая при рубке . составляет 0.4 - 1.0 мм.

Слесарное зубило состоит из трех частей: рабочей, средней и ударной . Средняя часть зубила имеет форму удобную для держания его в процессе рубки. Зубило длинной 100-125 мм применяют при выполнении мелких работ, 150-200 мм - при грубой работе.

Неисправным инструментом является, инструмент не прошедший процесс заточки.

Угол заточки зубил и крейцмейселей зависит от твердости обрабатываемого металла. Для рубки чугуна, твердой стали и твердой бронзы угол заточки инструмента равен 70°, для рубки средней и мягкой стали ― 60°, для рубки латуни, меди и цинка ― 45°, для рубки очень мягких металлов (алюминия, свинца)―35―45°.

Слесарный инструмент затачивают на заточных станках с абразивными кругами. Во время заточки рабочая часть инструмента (лезвие) сильно нагревается и может произойти ее отпуск. При отпуске твердость закалки теряется и инструмент становится негодным для дальнейшей работы. Во избежание этого рабочую часть инструмента во время заточки охлаждают водой.

На рис. 2 показано, как надо держать зубило при заточке и проверять правильность заточки угла. Производительность и чистота рубки металла зависят от правильных приемов работы.

hello_html_42d7a80a.jpg

Рис. 2. Заточка зубила на заточном станке: а ― приемы держания зубила при заточке, б ― шаблон для проверки правильности угла заточки

 

Изучение особенностей техники безопасности при рубке металла:

1) При рубке металлов в тисках необходимо следить, чтобы отлетающая стружка не летела в направление работающего рядом.

2) При заточке зубил следует пользоваться предохранительными очками или экраном на точиле.

3) Боек зубила не должен иметь рваных (разбитых ) краев.

Содержание отчета

Назначение рубки металла и область ее применения.

Правила рубки металла

 

 

Практическая работа №5

Цель работы: Цель работы: ознакомление с различными видами инструментов и приспособлений, применяемых при резке металла, овладение практическими навыками резки металла листового проката.

 

При резке металла пользуются различными инструментами: кусачками, ножницами,

ножовками, труборезами. Применение того или иного инструмента зависит от материала,

профиля и размеров обрабатываемой заготовки или детали. Например, для резки

проволоки применяют кусачки (а), которые изготовляют из инструментальной

стали марки У7 или У8. Губки кусачек подвергаются закалке с последующим низким

(нагрев до 200° С и медленное охлаждение) отпуском.

 

 

 Инструменты для резки металла: а — кусачки, б — стуловые ножницы, в —

рычажные ножницы

Для резки листового материала используют ручные, стуловые, рычажные, электрические,

пневматические, гильотинные, дисковые ножницы. Тонкий листовой материал (до 3 мм)

обычно режут ручными или стуловыми ножницами (б), а толстый (от 3 до 6 мм)

рычажными (в).Такие ножницы изготовляют из углеродистой инструментальной стали У8, У10. Режущие кромки ножниц закаливают. Угол заострения режущих кромок ножниц обычно не превышает 20—30°.

При резке ножницами предварительно размеченный металлический лист располагают

между лезвиями ножниц с таким расчетом, чтобы разметочная линия совпадала с верхним

лезвием ножниц.

Все более широкое применение находят электрические и пневматические ножницы. В

корпусе электрических ножниц имеется электродвигатель, ротор которого при

помощи червячной передачи приводит во вращение эксцентриковый валик, с которым

связан шатун, приводящий в движение подвижный нож. Нижний неподвижный нож

жестко связан с корпусом ножниц.

 

 

                                           Электрические ножницы И-31

 

Пневматические ножницы работают под действием сжатого воздуха.

Гильотинными ножницами с механическим приводом разрезают стальные листы

толщиной до 40 мм. Дисковыми ножницами разрезают листовой материал толщиной до 25

мм по прямой или кривой линиям.

Для резки небольших заготовок или деталей применяют ручные и электромеханические

ножовки.

Ручная ножовка представляет собой стальную раздвижную рамку, называемую

станком, в которой укреплено стальное ножовочное полотно. Ножовочное полотно имеет

форму пластины длиной до 300 мм, шириной от 3 до 16 мм и толщиной от 0,65 до 0,8 мм.

Зубья ножовочного полотна разводятся в разные стороны с таким расчетом, чтобы

ширина пропила, образующегося при резке, получалась на 0,25—0,5 мм больше толщины

ножовочного полотна.

Ножовочные полотна бывают с мелкими и крупными зубьями. При разрезании деталей с

тонкими стенками, тонкостенных труб и тонкого профильного проката применяют

полотна с мелкими зубьями, а для резки мягких металлов и чугуна — с крупными

зубьями.

Ножовочное полотно устанавливают в станке зубьями вперед и натягивают так, чтобы оно

во время работы не перекашивалось. Перед началом работы разрезаемую заготовку или

деталь устанавливают и зажимают в тисках так, чтобы разметочная линия (линия разреза)

была расположена как можно ближе к губкам тисков.

Во время работы слесарь должен держать ножовку за рукоятку правой рукой, а левая рука

должна лежать на переднем конце станка. При перемещении ножовки от себя совершается

рабочий ход. При этом ходе нужно делать нажим, а при обратном перемещении ножовки,

т. е. при перемещении на себя, происходит холостой ход, при котором нажима не следует

делать.

Работа ручной ножовкой малопроизводительная и утомительна для рабочего. Применение

электромеханических ножовок резко повышает производительность труда. В корпусе ножовки имеется электродвигатель, приводящий во вращение вал, на котором насажен барабан.

                                                   Электромеханическая ножовка

 

На барабане имеется спиральный паз, по которому перемещается палец, закрепленный в

ползуне. К ползуну прикреплено ножовочное полотно. При работе электродвигателя барабан вращается, а ножовочное полотно, прикрепленное к ползуну, совершая

возвратно-поступательное движение, режет металл. Планка предназначена для упора

инструмента при работе.

                                                        Полотно ножовки.

 

Ножовка:

1 — станок, 2 — неподвижная серьга, 3 — рукоятка, 4 — ножовочное полотно, 5 — лупа,

6 — барашек, 7 — подвижная серьга

 

 

                                                                        Труборез

 

Для резки труб применяется труборез. Он состоит из скобы  с тремя дисковыми

резцами, из которых резцы неподвижны, а резец подвижный, и рукоятки, установленной

на резьбе. При работе труборез надевают на трубу, поворотом рукоятки придвигают

подвижный диск до соприкосновения с поверхностью трубы, затем, вращая труборез

вокруг трубы, разрезают ее.

Трубы и профильный материал режут также ленточными или дисковыми пилами.

На станине пилы имеется стол с прорезью, предназначенной для прохода (ленты) полотна пилы. В нижней части станины находятся электродвигатель и ведущий шкив пилы, а в верхней части станины — ведомый шкив. При помощи маховичка натягивают полотно пилы.

В дисковых пилах вместо режущей ленты имеется режущий диск. Особенностью

дисковых пил является возможность резки профильного металла под любым углом.

Для резки закаленной стали и твердых сплавов применяют также тонкие шлифовальные

круги.

 

Содержание отчета

1 Нарисовать рисунок ручной ножовки. Описать устройство ручной ножовки,

пользование ею.

2 Изучить и описать технику ручной резки металлов.

 

Контрольные вопросы:

1 Перечислите способы резки металла. Назовите инструменты, которыми производится резка.

2 Как устроена ножовка и для чего она применяется? Как закрепляется ножовочное полотно в станке?

3 Каким инструментом производится резка труб?

4 Какой угол заострения придаются ножницам?

5 Перечислите правила безопасной работы на ножницах.

 

 

Практическая работа №6

 

Цель работы:

Ознакомиться с основными способами опиливания металла. Основными инструментами применяемыми для опиливания. Приобрести практические навыки по опиливанию металлов.

Опиливанием называется способ резания, при котором осуще­ствляется снятие слоя материала с поверхности заготовки с по­мощью напильника.

Напильник — это многолезвийный режущий инструмент, обес­печивающий сравнительно высокую точность и малую шерохова­тость обрабатываемой поверхности заготовки (детали).

Опиливанием придают детали требуемую форму и размеры, про­изводят пригонку деталей друг к другу при сборке и выполняют другие работы. С помощью напильников обрабатывают плоскости, криволинейные поверхности, пазы, канавки, отверстия различной формы, поверхности, расположенные под разными углами и т. д.

Припуски на опиливание оставляют небольшие  от 0,5 до 0,025 мм. Достигаемая точность обработки может быть от 0,2 до 0,05 мм, и в отдельных случаях — до 0,005 мм.

Напильник (а) представляет собой стальной брусок определенного профиля и длины, на поверхности которого имеется насечка (нарезка). https://pandia.ru/text/77/484/images/image001_233.jpg

                                                           Напильники:

а — основные части (1— ручка; 2 — хвостовик; 3 — кольцо; 4 — пятка; 5 — грань;

6 — насечка; 7 — ребро; 8 — нос); б — одинарная насечка; в — двойная насечка;

г — рашпильная насечка; д — дуговая насечка; е — насадка ручки; ж — снятие ручки напильника.

Насечка образует мелкие и острозаточенные зубья, имеющие в сечении форму клина. Для напильников с насе­ченным зубом угол заострения β обычно 70°, передний угол γ до 16°, задний угол α от 32 до 40°.

Насечка может быть одинарной (простой), двойной (перекрест­ной), рашпильной (точечной) или дуговой (б — д).

Напильники с одинарной насечкой снимают широкую стружку, равную длине всей насечки. Их применяют при опиливании мягких металлов.

Напильники с двойной насечкой применяют при опиливании ста­ли, чугуна и других твердых материалов, так как перекрестная насечка размельчает стружку, чем облегчает работу.

Напильниками с рашпильной насечкой, имеющей между зубьями вместительные выемки, что способствует лучшему размещению стружки, обрабатывают очень мягкие металлы и неметаллические материалы.

Напильники с дуговой насечкой имеют большие впадины между зубьями, что обеспечивает высокую производительность и хорошее качество обрабатываемых поверхностей.

Изготовляются напильники из стали У13 или У13 А. После на­сечки зубьев напильники подвергают термической обработке,

Ручки напильников изготовляют обычно из древесины (березы, клена, ясеня и других пород). Приемы насадки ручек показаны на рисунке е и ж.

По назначению напильники делят на следующие группы: общего назначения, специального назначения, надфили, рашпили, машин­ные напильники.

Для общеслесарных работ применяют напильники общего назначения. По числу насечек на 1 см длины их подразделяют на 6 номе­ров.

Напильники с насечкой №0 и 1 (драчевые) имеют наиболее крупные зубья и служат для грубого (чернового) опиливания с точностью 0,5—0,2 мм.

Напильники с насечкой №2 и 3 (личные) слу­жат для чистового опиливания деталей с точностью 0,15—0,02 мм.

Напильники с насечкой №4 и 5 (бархатные) применяются для окончательной точной отделки изделий. Дости­гаемая точность обработки — 0,01—0,005 мм.

По длине напильники могут изготовляться от 100 до 400 мм.

По форме поперечного сечения они подразделяются на плоские, квадратные, трехгранные, круглые, полукруглые, ромбические и ножовочные.

Для обработки мелких деталей служат малогабаритные напиль­ники-надфили. Они изготовляются пяти номеров с числом насечек на 1 см длины до 112.

Обработку закаленной стали и твердых сплавов производят специальными надфилями, у которых на стальном стержне закреп­лены зерна искусственного алмаза.

https://pandia.ru/text/77/484/images/image002_117.jpg

                                    Формы сечений напильников:

а и б — плоские; в — квадратный; г — трехгранные; д — круглые; е — полукруглый;

ж — ромбический; з — ножовочные.

Улучшение условий и повышение производительности труда при опиливании металла достигаются путем применения механизиро­ванных (электрических и пневматических) напильников.

В условиях учебных мастерских возможно применение механи­зированных ручных опиловочных машинок, которые широко ис­пользуются на производстве.

Универсальная шлифовальная машина ( г), работаю­щая от асинхронного электродвигателя 1, имеет шпиндель, к кото­рому крепится гибкий вал 2 с державкой 3 для закрепления рабо­чего инструмента, и сменные прямые и угловые головки, позволяю­щие с помощью круглых фасонных напильников производить опиливание в труднодоступных местах и под разными углами.

Опиливание металла

При опиливании заготовку закрепляют в тисках, при этом опиливаемая поверхность долж­на выступать над уровнем гу­бок тисков на 8—10 мм. Чтобы предохранить заготовку от вмя­тин при зажиме, на губки тисков надевают нагубники из мягкого материала. Рабочая поза при опи­ливании металла аналогична ра­бочей позе при разрезании ме­талла ножовкой.

Правой рукой берут за ручку напильника так, чтобы она упи­ралась в ладонь руки, четыре пальца охватывали ручку снизу, а большой палец помещался сверху (а).

Ладонь левой руки накладывают несколько поперек напильни­ка на расстоянии 20—30 мм от его носка (б).

Перемещают напильник равномерно и плавно на всю длину. Движение напильника вперед является рабочим ходом. Обратный ход — холостой, его выполняют без нажима. При обратном ходе не рекомендуется отрывать напильник от изделия, так как можно потерять опору и нарушить правильное положение инструмента.

https://pandia.ru/text/77/484/images/image003_84.jpg https://pandia.ru/text/77/484/images/image004_68.jpg

б

https://pandia.ru/text/77/484/images/image005_62.jpg https://pandia.ru/text/77/484/images/image006_55.jpg

в

                       Хватка напильника и балан­сировка им в процессе опиливания:

а — хватка правой рукой; б — хватка ле­вой рукой; в — силы нажима в начале движения;

г — силы нажима в конце движения.

В процессе опиливания необходимо соблюдать координацию усилий нажима на напильник (балансировку). Она заключается в постепенном увеличении во время рабочего хода небольшого вна­чале нажима правой рукой на ручку с одновременным уменьше­нием более сильного вначале нажима левой рукой на носок на­пильника (в, г).

Длина напильника должна превышать размер обрабатываемой поверхности заготовки на 150—200 мм.

Наиболее рациональным темпом опиливания считают 40—60 двойных ходов в минуту.

Опиливание начинают, как правило, с проверки припуска на обработку, который мог бы обеспечить изготовление детали по размерам, указанным на чертеже. Проверив размеры заготовки, определяют базу, т. е. поверхность, от которой следует выдержи­вать размеры детали и взаимное расположение ее поверхностей.

Если степень шероховатости поверхностей на чертеже не ука­зана, то опиливание производят только драчевым напильником. При необходимости получить более ровную поверхность опилива­ние заканчивают личным напильником.

В практике ручной обработки металлов встречаются следую­щие виды опиливания: опиливание плоскостей сопряженных, парал­лельных и перпендикулярных поверхностей деталей; опиливание криволинейных (выпуклых или вогнутых) поверхностей; распиливание и припасовка поверхностей.

Опиливание широких плоских поверхно­стей является одним из самых сложных видов опиливания. Для получения правильно опиленной прямолинейной поверхности глав­ное внимание должно быть сосредоточено на обеспечении прямо­линейности движения напильника. Опиливание ведут перекрест­ным штрихом (с угла на угол) под углом 35—40° к боковым сторо­нам тисков. При опиливании по диагонали не следует выходить напильником на углы заготовки, так как при этом уменьшается площадь опоры напильника и снимается большой слой металла. Образуется так называемый «завал» края обрабатываемой поверх­ности.

Проверку правильности плоскости производят линейкой «на просвет», для чего накладывают ее вдоль, поперек и по диагонали обработанной поверхности. Поверочная линейка по длине должна перекрывать проверяемую поверхность.

В случае опиливания параллельных плоских поверхностей про­верку параллельности производят измерением расстояния между этими поверхностями в нескольких местах, которое должно быть везде одинаковым.

При обработке узких плоскостей на тонких деталях применяют продольное и поперечное опиливание. При опиливании поперек заготовки напильник соприкасается с меньшей поверхностью, по ней проходит больше зубьев, что позволяет снять большой слой металла. Однако при поперечном опиливании поло­жение напильника неустойчивое и легко «завалить» края поверх­ности. Кроме этого, образованию «завалов» может способствовать изгиб тонкой пластинки во время рабочего хода напильника. Про­дольное опиливание создает лучшую опору для напильника и исключает вибрацию плоскости, но снижает производительность обработки.

Для создания лучших условий и повышения производительно­сти труда при опиливании узких плоских поверхностей применяют специальные приспособления: опиловочные призмы, универсаль­ные наметки, наметки-рамки, специальные кондукторы и другие.

Простейшим из них является наметка-рамка (а). Ее применение исключает образование «завалов» обрабатываемой по­верхности. Лицевая сторона наметки-рамки тщательно обработана и закалена до высокой твердости.

Размеченную заготовку вставляют в рамку, слегка прижимая ее винтами к внутренней стенке рамки. Уточняют установку, добиваясь совпадения риски на заготовке с внутренним ребром рам­ки, после чего окончательно закрепляют винты.

https://pandia.ru/text/77/484/images/image007_51.jpg

                                     Опиливание поверхностей:

а — опиливание с помощью наметки-рамки; б — прием опиливания выпуклых поверхностей; в — прием опиливания вогнутых поверхностей; г — опиливание с помощью уни­версальной шлифовальной машины (1 — электродвигатель; 2 — гибкий вал; 3 — державка с инструментом).

Затем рамку зажимают в тисках и опиливают узкую поверхность заготовки. Обработку ведут до тех пор, пока напильник не коснет­ся верхней плоскости рамки. Поскольку эта плоскость рамки об­работана с высокой точностью, то и опиливаемая плоскость будет точной и не потребует дополнительной проверки при помощи ли­нейки.

При обработке плоскостей, расположен­ных под углом 90°, сначала опиливают плоскость, прини­маемую за базовую, добиваясь ее плоскостности, затем плоскость, перпендикулярную к базовой. Наружные углы обрабатывают пло­ским напильником. Контроль осуществляют внутренним углом угольника. Угольник прикладывают к базовой плоскости и, при­жимая к ней, перемещают до соприкосновения с проверяемой по­верхностью. Отсутствие просвета указывает, что перпендикуляр­ность поверхностей обеспечена. Если световая щель сужается или расширяется, то угол между поверхностями больше или меньше 90°.

Внутренние углы обрабатывают следующим образом. Размечают заготовку, используя в качестве баз наружные поверхности. Они же будут базами и при контроле. Затем ножовкой вырезают лишний металл, оставляя припуск на опиливание около 0,5 мм. Если сто­роны внутреннего угла должны сходиться без закругления, в нем просверливается отверстие диаметром 2—3 мм или делается неглу­бокий пропил под углом 45° (обработать внутренний угол без за­кругления внутри практически невозможно). Опиливая стороны угла, в первую очередь добиваются их плоскостности, а затем пер­пендикулярности. Опиливание поверхностей по внутреннему углу ведут так, чтобы ко второй поверхности было обращено ребро на­пильника, на котором нет насечки. Контроль правильности внут­реннего угла производится также угольником.

Поверхности, расположенные под углом больше или меньше 90°, обрабатываются аналогичным образом. Наружные углы обрабатываются плоскими напильника­ми, внутренние — ромбическими, трехгранными и другими. Конт­роль обработки ведется угломерами или специальными шабло­нами.

При обработке криволинейных поверх­ностей, кроме обычных приемов опиливания, применяются и специальные.

Выпуклые криволинейные поверхности можно обрабатывать, ис­пользуя прием раскачивания напильника (б). При пере­мещении напильника сначала его носок касается заготовки, ручка опущена. По мере продвижения напильника носок опускается, а ручка приподнимается. Во время обратного хода движения напиль­ника противоположные.

Вогнутые криволинейные поверхности в зависимости от радиу­са их кривизны обрабатываются круглыми или полукруглыми напильниками. Напильник совершает сложное движение — вперед и в сторону с поворотом вокруг своей оси (в). В процессе обработки криволинейных поверхностей заготовку обычно перио­дически перезажимают с тем, чтобы обрабатываемый участок рас­полагался под напильником.

При изготовлении партии деталей целесообразно изготовить специальный копир, подобный наметке-рамке, лицевая часть ко­торого имеет форму криволинейной поверхности. В этом случае копир с закрепленной в нем заготовкой зажимают в тисках и ведут опиливание до касания напильником закаленной поверхности ко­пира.

Распиливанием называется обработка отверстий (пройм) различ­ной формы и размеров при помощи напильников. По применяе­мому инструменту и приемам работы распиливание аналогично опиливанию и является его разновидностью.

Для распиливания применяются напильники различных типов и размеров. Выбор напильников определяется формой и размерами проймы. Проймы с плоскими поверхностями и пазы обрабатывают­ся плоскими напильниками, а при малых размерах — квадратными. Углы в проймах распиливаются трехгранными, ромбическими, но­жовочными и другими напильниками. Проймы криволинейной фор­мы обрабатывают круглыми и полукруглыми напильниками.

Распиливание обычно выполняют в тисках. В крупных дета­лях проймы распиливают на месте установки этих деталей.

Подготовка к распиливанию начинается с разметки проймы. За­тем удаляется излишний металл из ее внутренней полости.

При больших размерах проймы и наибольшей толщине заго­товки металл вырезается ножовкой. Для этого сверлят по углам проймы отверстия, заводят в одно из отверстий ножовочное полот­но, собирают ножовку и, отступя от разметочной линии на величину припуска на распиливание, вырезают внутреннюю полость.

Пройму средних размеров обсверливают по контуру свер­лом диаметром

3—5 мм вблизи разметочных линий, затем крейцмейселем или зубилом прорубают оставшиеся перемычки.

Для подготовки к распиливанию небольших пройм часто бывает достаточно просверлить одно отверстие диаметром на-0,3—0,5 мм меньше диаметра окружности, вписанной в пройму.

Непосредственно распиливание производится, как уже отме­чалось, приемами, аналогичными опиливанию.

Контроль осуществляется штангенциркулем и специальными шаблонами.

Припасовкой называется взаимная пригонка двух деталей, соп­рягающихся без зазора. Припасовывают как замкнутые, так и по­лузамкнутые контуры. Припасовка характеризуется большой точ­ностью обработки. Из двух припасовываемых деталей отверстие принято называть, как и при распиливании, проймой, а деталь, входящую в пройму, — вкладышем.

Припасовка применяется как окончательная операция при об­работке деталей шарнирных соединений и чаще всего при изготов­лении различных шаблонов. Выполняется припасовка напильни­ками с мелкой или очень мелкой насечкой.

Сначала обрабатывают заготовки для вкладыша и проймы. Раз­мечают их, распиливают пройму и опиливают вкладыш, оставляя припуск (0,1—0,4 мм) на припасовку.

Первой обычно подготовляют к припасовке и припасовывают ту из сопряженных деталей, которую легче обработать и проконт­ролировать, с тем чтобы затем использовать ее для контроля при изготовлении сопряженной детали.

Точность припасовки считается достаточной, если вкладыш входит в пройму без перекоса, качки и просветов.

Возможные виды брака при опиливании металла и их причины:

- неточность размеров опиленной заготовки (снятие очень большого или малого слоя металла) вследствие неточности разметки, непра­вильности измерения или неточности измерительного инструмента;

- неплоскостность поверхности и «завалы» краев заготовки как результат неумения правильно выполнять приемы опиливания;

- вмятины и другие повреждения поверхности заготовки в ре­зультате неправильного ее зажима в тисках.

При опиливании металла ручными и механизированными ин­струментами следует соблюдать правила техники безопасности. Поль­зоваться только исправным инструментом. Ручки напильников должны быть прочно насажены. Запрещается работать напильни­ками без ручек или с треснутыми, расколотыми ручками. Обра­зовавшуюся в процессе опиливания стружку следует сметать спе­циальной щеткой. Запрещается сдувать ее или смахивать голыми руками, чтобы избежать ранения рук или засорения глаз. При рабо­те электроинструментами соблюдать правила электробезопаснос­ти. Следить за исправностью токопроводящих частей инструмента.

Общие правила обращения и ухода за напильниками:

- применять напильники только по их назначению;

- нельзя обрабатывать напильником материалы, твердость ко­торых равна или превышает его твердость;

- предохранять напильники даже от незначительных ударов, которые могут повредить зубья;

- оберегать от попадания на напильники влаги, что вызывает их коррозию;

- периодически очищать напильники от стружки кордовой щет­кой;

- хранить напильники на деревянных подставках в положении, исключающем соприкосновение их между собой.

Задание

По заданию преподавателя опилить заготовки с уз­кими и широкими поверхностями с самостоятельным подбором необходимых напильников и контрольно-измерительного инстру­мента. Опилить криволинейные поверхности на предложенных заготовках, предварительно подобрать напильники необходимого профиля и инструменты для контроля работы.

Вопросы:

1. Какой способ обработки металла называется опиливанием?

2. В каких случаях применяют опиливание металла?

3. Какие бывают виды насечек для образования зубьев напиль­ников?

4. Из какого материала изготовляют напильники?

5. На какие группы делят напильники по их назначению?

6. Что такое надфили и для чего они служат?

7. Каковы общие правила обращения и ухода за напильниками?

8. Какова техника выполнения приемов опиливания?

9. Какие механизированные инструменты применяются при опиливании металла?

10. Какие возможны виды брака при опиливании и в чем их причины?

11. Какие правила техники безопасности надо соблюдать при опиливании металлов?

 

 

Практическая работа №7

Цель работы

Овладеть основными приёмами обработки отверстий металлов.

 

Сверление, зенкование, зенкерование и развертывание —  это основные операции получения и обработки цилиндрических  и конических отверстий резанием. Сущность данных операций заключается в том, что процесс резания (снятие слоя материала) осуществляется с помощью

движений: вращательного движения режущего инструмента (сверла, зенкера и т. д. ) или заготовки и поступательного движения заготовки или инструмента вдоль оси

получаемого или обрабатываемого отверстия. Эти движения создаются с помощью ручных (коловорот, дрель) или механизированных (электрическая дрель) приспособлений, а также станков (сверлильных, токарных и др.). Вращательное движение инструмента (заготовки)

называют главным движением. Поступательное движение заготовки (инструмента) называют вспомогательным или движением подачи. Сверление представляет собой один из видов получения и обработки отверстий резанием с помощью специального инструмента— сверла.

Как и всякий другой режущий инструмент, сверло работает по принципу клина. В современном производстве применяются преимущественно спиральные сверла и реже перовые, центровочные и другие специальные виды сверл. Спиральное сверло (рис. 1, а) состоит из рабочей части 2,

хвостовика 1 и шейки 3 Рабочая часть сверла в свою  очередь состоит из цилиндрической

(направляющей) и режущей частей. На цилиндрической части есть две винтовые канавки, по которым отводится стружка в процессе резания. Направление винтовых канавок обычно правое (левые сверла применяют очень редко). Вдоль канавок на цилиндрической части сверла имеются узкие полосочки, называемые ленточками. Они служат уменьшения трения сверла о стенки отверстия (сверла диаметром 0,25—0,5 мм выполняются без ленточек). Режущая часть сверла (рис. 1, е) образуется двумя режущими кромками, расположенными под определенным углом друг к другу. Этот угол называют углом при вершине. Его величина зависит от свойств обрабатываемого материала. Для стали и чугуна средней твердости он составляет 116—118°.

 

Рис. 1 Инструменты для обработки отверстий: а — спиральное сверло;

б — цилиндрический зенкер; в — конические (угловые) зенковки;

г — торцовая зенковка (цековка); д — развертка; е — режущая часть сверла

(1 — хвостовики; 2 — рабочие части; 3 — шейки; 4 — лапки).

 

Хвостовик служит для закрепления сверла в сверлильном патроне или шпинделе станка и может быть цилиндрической или конической формы. Конический хвостовик имеет на конце лапку 4, которая не позволяет ему провертываться в шпинделе и служит упором при выталкивании сверла из гнезда. Шейка сверла, соединяющая рабочую часть с хвостовиком, служит для выхода абразивного круга в процессе шлифования сверла при его изготовлении. На шейке обычно обозначают марку сверла. Изготовляют сверла из углеродистой инструментальной (У

10 и У12А), легированной (9Х и 9ХС) и быстрорежущей (Р9 и Р18) стали. Все шире применяют металлокерамические твердые сплавы марок ВК6, ВК8 и Т15К6. Пластинками из твердых

сплавов обычно оснащают только рабочую (режущую) часть сверла. В процессе работы

режущая притупляется, поэтому сверла периодически затачивают. При заточке сверло слегка

прижимают поверхности наждачного круга и перемещают его так, чтобы режущие кромки стали прямолинейными и острыми. Правильно заточенное сверло должно иметь одинаковую длину режущих кромок, симметрично расположенных относительно оси сверла. Величина угла при вершине сверла должна соответствовать свойствам материала, для обработки которого предназначается сверло. Правильность заточки режущей части сверла проверяется

специальным шаблоном. Сверлами производят не только сверление глухих и сквозных отверстий, т. е. получение этих отверстий в сплошном материале, но и рассверливание, т. е. увеличение размера (диаметра) уже полученных отверстий.

Зенкованием называется обработка отверстий с целью получения конических или цилиндрических углублений, например под потайную головку заклепки. Выполняется зенкование с помощью зенковок (рис. 1,в и г) или сверлом большего диаметра.

 

Зенкерование — это обработка отверстий, полученных литьем, штамповкой или сверлением, для придания им более правильной формы и размеров. Оно выполняется специальными

инструментами-зенкерами. Зенкер (рис. 1, б) — стержень из стали марок У10 или У12 с

режущими кромками на боковой цилиндрической конической поверхности (цилиндрические

зенкеры). Могут быть зенкеры с режущими расположенными на торце (торцовые зенкеры). Для обеспечения соосности обрабатываемого отверстия и зенкера на торце зенкера иногда делают гладкую направляющую часть. Зенкерование может быть процессом обработки отверстия или подготовительным к развертыванию. В последнем случае при зенкеровании оставляют припуск на дальнейшую обработку.

Развертывание —это, как правило, (окончательная) обработка отверстий. По своей сущности она

подобна зенкерованию, но обеспечивает точность и низкую шероховатость поверхности отверстий. Выполняется эта операция ручными развертками. Развертка (рис. 1, д) состоит из рабочей части, шейки и хвостовика. Рабочая часть, в свою очередь, подразделяется на заборную режущую (коническую) и калибрующую (цилиндрическую) части. Калибрующая часть ближе к шейке имеет обратный конус (уменьшение диаметра в сторону шейки на 0,04— 0,6

мм) для снижения трения развертки о стенки отверстия. Зубья на рабочей части (винтовые или прямые) могут быть расположены равномерно по окружности или неравномерно. Развертки с неравномерным шагом зубьев используются обычно для обработки отверстий вручную. Они позволяют избежать образования так называемой огранки, т. е. получения отверстий

неправильной цилиндрической формы.

Хвостовик ручной развертки имеет квадрат для установки воротка. Хвостовик машинных разверток диаметром до 10—12 мм выполняется цилиндрическим, у других разверток

коническим с лапкой, как и у сверл. Развертки выполняются комплектами из двух или трех

штук для черновой и чистовой обработки отверстий. Изготавливают развертки из тех же материалов, что и другие режущие инструменты для обработки отверстий.

Все операции по обработке отверстий выполняют в основном на сверлильных или токарных станках. Однако в тех случаях, когда деталь невозможно установить на станок или

когда отверстия расположены в труднодоступных обработку выполняют вручную с помощью  коловоротов, воротков, ручных или механизированных (электрических и

пневматических) дрелей. Коловорот представляет собой изогнутый коленчатого вала стержень. Верхнее звено имеет шляпку для нажатия на коловорот с целью придания режущему инструменту поступательного движения. Среднее звено служит рукояткой для вращения коловорота, а нижнее снабжено патроном или имеет гнездо для крепления инструмента. Применяют коловороты для обработки мягких металлов или неметаллических материалов

(рис. 2).

Рис. 2 Коловорот

 

 

 

Рис. 3 Ручная и электрические дрели:

а — ручная (1 — упор; 2 — корпус с зубчатой передачей; 3 — привод с рукояткой; 4 — патрон; 5 — сверло); б - электродрель тяжелого типа; в — электродрель среднего типа; г — электродрель легкого типа.

Вороток с квадратными отверстиями используют при работе с инструментом, имеющим на хвостовике квадрат, например ручной разверткой. Ручная дрель (рис. 3, а) состоит из остова с упором 1 для нажатия на дрель и придания сверлу поступательного движения, зубчатой передачи 2 с ручным приводом 3, рукоятки для удержания дрели, шпинделя с установленным на нем патроном 4 для закрепления режущего инструмента.

Для облегчения  труда при обработке отверстия  и  повышения его производительности используются механизированные дрели (ручные сверлильные машинки). Они могут быть электрическими или пневматическими. В практике работы в учебных мастерских более широкое применение имеют электрические дрели, так как пневматические дрели требуют

подвода к ним сжатого воздуха. Электрические сверлильные машинки изготовляются трех

типов:

·тяжелого,

·среднего,

·легкого.

Машинки тяжелого типа (рис. 3, б) применяются для получения и обработки отверстий диаметром 20—30 мм. Они имеют две рукоятки на корпусе (или две рукоятки и упор) для

удержания машинки и придания поступательного движения рабочему инструменту.

Машинки среднего типа (рис.3,в) обычно имеют замкнутую рукоятку на задней части корпуса. Они используются для сверления отверстий диаметром до 15 мм. Машинки легкого типа предназначены для сверления отверстий диаметром до 8—9 мм. Корпус таких машинок часто выполняется в форме пистолета. Сверлильные машинки различают: прямые — с расположением оси шпинделя соосно или параллельно оси двигателя и угловые — с расположением оси шпинделя под углом к оси двигателя. Последние применяют для получения и обработки

отверстий в труднодоступных местах. Независимо от типа и мощности электрические сверлильные машинки состоят из трех основных частей: электродвигателя с напряжением 220 или 360 В, зубчатой передачи и шпинделя, на который устанавливается сверлильный патрон.

При сверлении электрической сверлильной машинкой устанавливают сверло в патроне или коническом отверстии шпинделя машинки, присоединяют токоподводящий провод

машинки к электросети, включают машинку и проверяют ее работу на холостом ходу, а также убеждаются в отсутствии биения сверла. Установив вершину сверла в керновое углубление, включают машинку и просверливают отверстие. В процессе работы следят, чтобы ось сверла была перпендикулярна к плоскости заготовки. Выводят сверло из отверстия, выключают двигатель и отсоединяют машинку от электросети. Зенкование отверстий выполняют после просверливания отверстия заданного диаметра. Для этого:

· выключают станок и, не снимая заготовку со стола, заменяют сверло соответствующей (цилиндрической или конической) зенковкой;

· включают станок и зенкуют отверстие до размера, указанного на чертеже.

Зенкерование отверстий выполняют после их просверливания с учетом припуска на зенкерование. Для этого:

· останавливают станок и, не снимая заготовку со стола, заменяют сверло на соответствующий зенкер;

· включают станок и зенкеруют отверстие.

Развертывание ручными развертками выполняют после просверливания отверстия с припуском под развертывание.

Для этого:

· снимают заготовку со стола станка и устанавливают ее в тисках;

· берут черновую развертку, смазывают ее рабочую часть минеральным маслом и вставляют (без перекоса) в отверстие;

· надевают на хвостовик развертки вороток и, слегка нажимая одной рукой на развертку, другой рукой вращают вороток по часовой стрелке, при необходимости развертку периодически извлекают из отверстия для очистки ее от стружки и смазывания. Заканчивают черновое развертывание, когда 3/4 рабочей части развертки выйдет из отверстия;

· такими же приемами выполняют развертывание отверстия чистовой разверткой;

· правильность развертывания проверяют калибром-пробкой.

При получении и обработке отверстий вручную и на станках возможны следующие виды брака и их причины. При сверлении:

1 диаметр отверстия больше заданного (неправильно выбран размер сверла или несимметрично заточены режущие кромки сверла, биение сверла);

2 ось отверстия перекошена (неправильная установка детали

на столе станка или в приспособлении, неперпендикулярность стола по отношению к шпинделю

станка);

3 грубая поверхность просверленного отверстия (тупое сверло, слишком большая подача, недостаточное охлаждение сверла).

При зенкеровании:

1 увод зенкера в процессе обработки отверстия (несовпадение оси зенкера с осью обрабатываемого отверстия);

2 диаметр отверстия больше заданного (неправильный выбор размера диаметра зенкера, биение шпинделя);

3 неудовлетворительная шероховатость поверхности отверстия (повышенная величина подачи, большой припуск на обработку, повышенный износ режущих кромок зенкера).

При развертывании:

1 следы дробления на поверхности отверстия (вращение развертки рывками, большой припуск на

обработку, неправильное закрепление развертки);

2 задиры на поверхности отверстия (неправильные приемы развертывания, тупые режущие кромки, повышенный припуск).

При сверлении и обработке отверстий необходимо соблюдать следующие меры безопасности.

Обрабатываемая деталь должна быть прочно закреплена. Не рекомендуется допускать образования длинных, завивающихся стружек, так как они могут поранить работающего. Во избежание этого необходимо периодически выводить сверло из отверстия и очищать его.

Используя в работе электрические сверлильные машинки, нужно соблюдать следующие правила электробезопасности. Перед включением электрической сверлильной машинки убедиться в исправности электропроводящих частей машинки и соответствии напряжения в сети требуемому. Корпус машинки должен быть обязательно заземлен. Работать только в резиновых перчатках и калошах. При отсутствии калош подкладывать под ноги резиновый коврик.

Вынимать сверло или другой режущий инструмент из патрона, а также снимать сам патрон только после отключения машинки от электрической сети.

 

Вопросы для самоконтроля

1 В чем заключается сущность операций по получению и обработке отверстий резанием?

2 Какова конструкция спиральных сверл. Из какого материала их изготовляют?

3 Как затачивают сверла?

4 Что такое зенкование, каким инструментом его выполняют?

5 Что такое зенкерование, каким инструментом его выполняют?

6 Что такое развертывание отверстий, каким инструментом его выполняют?

7 Какие ручные и механизированные приспособления применяют при получении и обработке отверстий?

8 Какой возможен брак при обработке отверстий и в чем его причины?

9 Какие правила техники безопасности необходимо соблюдать при работе с электрическими сверлильными машинками?

 

Задание

1 Расскажите правила техники безопасности при сверлении и зенковании металла.

2 Определите вид сверла из числа предложенных.

3 Произведите заточку спирального сверла.

4 Установите сверло заданного диаметра в дрель.

5 Произведите сверление листового металла ручной дрелью.

6 Произведите зенковку отверстия.

7 Произведите сверление сквозного отверстия электродрелью.

 

 

Практическая работа №8

 

 

Цель: Овладеть основными приёмами нарезания резьбы.

 

 

Вопросы

Ответы

1

 

Возьмите цилиндрический стер­жень диаметром D и вырежьте из бу­маги прямоугольный треугольник АВС, сторона которого АВ равна дли­не окружности цилиндра D, т. е. 3,14 диаметра (рис. 1, а, в). Оберните треугольник АВС вокруг цилиндра так, чтобы сторона АВ совместилась с окружностью нижнего основания ци­линдра, тогда другая сторона тре­угольника ВС расположится по обра­зующей, а гипотенуза АС образует на поверхности цилиндра винтовую ли­нию. При этом сторона треугольника ВС составит шаг винтовой линии, АС — длину одного витка, а угол САВ — угол подъёма винтовой ли­нии α.

Рис. 1. Образование винтовой линии

2

Как различают левую или пра­вую резьбу, однозаходную (одноходо­вую) и многозаходную (многоходо­вую)?

Если винтовая линия при на­вивании треугольника на цилиндр, удаляясь от основания, постепенно поднимается вправо, то она называет­ся правой. Если винтовая линия, удаляясь от основания цилиндра, под­нимается влево, то она называется левой (рис. 1, б, г).

3

Как разделяют резьбы по чис­лу ниток и как они характеризуются?

По числу ниток резьбы разделяют на одноходовые (однозаходные) и мно­гоходовые (многозаходные). У однозаходной резьбы на торце винта или гайки ви­ден только один конец витка, у многозаходных — два, три и более.

Од­ноходовые резьбы имеют малые углы подъёма винтовой линии и большое трение, они применяются там, где требуется надёжное соединение (для крепёжных резьб). Многоходовые (многозаходные) имеют по сравнению с однозаходными угол подъёма винто­вой линии значительно больше. Такие резьбы применяются в тех случаях, когда необходимо быстрое перемеще­ние по резьбе, при наименьшем тре­нии, при этом за один оборот винта (или гайки) они перемещаются на размер хода винтовой линии резьбы. Многозаходные резьбы используются в механизмах, служащих для переда­чи движения.

4

Как определяют число ходов резьбы и какая зависимость между ходом, шагом и заходом?

Ход — это осевое перемещение винта на один его оборот. Например, при завинчива­нии гайка переместится за один обо­рот на длину, равную ходу резьбы. Для одноходовых резьб шаг равен хо­ду.

Для многозаходных винтов ход резьбы tв получим умножением шага s (расстояния между смежными витка­ми) на число заходов (ниток резьбы) tв = sz.

У одноходовых резьб: tв = P

5

На рис. 2, а, б показаны резьбовые детали. Определить число заходов для указанных резьб.

 

 

Рhttps://fsd.multiurok.ru/html/2020/01/02/s_5e0e033d08d4b/1303583_2.png ис. 2. Системы резьб: а, б — резьбовые детали, в — метрическая, г — дюймовая, д— трубная

На рис. 2, а показана трёхзаход ная резьба, а на рис. 2, б — восьмизаходная.

6

Какие резьбы применяют в машиностроении и чем они отличают­ся друг от друга (рис. 2, в, г, д)?

В машиностроении применя­ются: метрическая, дюймовая и труб­ная резьбы (рис. 2).

Метрическая резьба имеет треугольный профиль с углом 60° с плоскосрезанной вершиной, диаметр и шаг резьбы даются в метрической сис­теме единиц — в миллиметрах (рис. 2, в).

Дюймовая резьба имеет тре­угольный плоскосрезанный профиль с углом 55° (резьба Витворта) или 60° (резьба Силлерса). Все размеры да­ются в дюймах (1" = 25,4 мм). Шаг выражается числом ниток (витков) на длине одного дюйма (рис. 2, г).

Трубная (цилиндрическая) резь­ба — это дюймовая с мелким шагом. Вершины витков у неё закруглённые и резьбовые детали соединяются без за­зора, это обеспечивает герметичность (рис. 2, д).

7

https://fsd.multiurok.ru/html/2020/01/02/s_5e0e033d08d4b/1303583_3.png При нарезании внутренней резьбы используют комплект метчиков (рис. 3). Объясните, как и чем от­личается устройство каждого из мет­чиков в комплекте, в какой последова­тельности и почему их применяют?

 

 

 

 

Рис. 3. Комплект метчиков

В комплект метчиков, состоя­щий из трёх штук, входят (рис. 3, а, б, в): I — черновой метчик, II — средний, III — чистовой. Все они имеют разные диаметры. Чтобы определить, какой метчик является черновым, какой средним и какой чис­товым, на хвостовике наносят круго­вые риски (кольца) или ставят соот­ветствующий номер I; II; III. Первый (черновой) метчик имеет скос 3— 4 нитки, снимает до 60% металла; второй (средний) метчик имеет скос только 3 нитки и даёт уже более точ­ную резьбу, снимает до 30% металла и третий (чистовой) метчик имеет скос 12° только 2 нитки, снимает до 10% металла, имеет полный профиль резь­бы и используется для точного на­резания и калибровки резьбы.

8

Определить момент пары сил, под действием которого при нарезании резьбы призматическими плашками вращается клупп (рис. 4), если сила P, с которой каждая рука действует на ручку клуппа, будет равна 20 H, а расстояние между точками приложения сил равно 400 мм?

https://fsd.multiurok.ru/html/2020/01/02/s_5e0e033d08d4b/1303583_4.png

 

 

Рис. 4. Определение момента при нарезании резьбы

M = Pd = 20 × 0,4=8Н ∙ м.

9

https://fsd.multiurok.ru/html/2020/01/02/s_5e0e033d08d4b/1303583_5.pngЧ то может послужить причи­ной поломки метчика и как удалить сломанные метчики из отверстия (рис. 5)?

 

 

 

 

 

 

 

 

 

 

 

 

 

 

 

 

 

 

 

 

 

 

 

 

 

 

 

 

 

 

 

 

 

 

 

 

 

 

 

 

Рис. 5. Приёмы удаления сломанных метчиков из отверстия

Поломка метчика может быть по причине защемления стружки при вывёртывании метчика, заниженного отверстия под резьбу (рис. 5).

Сломанный метчик удаляют из от­верстия следующим образом:

а) если из отверстия торчит обло­мок метчика, то нужно выступающую его часть захватить плоскогубцами или ручными тисочками и вывернуть обломки из отверстия;

б) при отсутствии выступающей части — в канавки метчика продеть концы согнутой проволоки и вывер­нуть метчик из отверстия;

в) если небольшой обломок метчи­ка не удаётся вывернуть с помощью проволоки, то метчик разламывают на мелкие куски закалённым пробойни­ком, напоминающим кернер, и куски извлекают из отверстия;

г) концы сломанного метчика из быстрорежущей стали можно удалить так: деталь с обломком метчика на­гревают в муфельной печи и дают ей остыть вместе с печью (оставляют ос­тывать от окончания смены и до на­чала следующей);

д) если сломан метчик из углеро­дистой стали — деталь вместе с за­стрявшим обломком нагревают докрас­на, медленно охлаждают и затем вы­сверливают застрявшую часть мет­чика;

е) если деталь очень большая и

её нагреть нельзя, применяют сле­дующие способы:

1) удаляют поло­манный метчик из отверстия с по­мощью специальной оправки (рис. 5, а), имеющей на торце четыре выступа, которыми она входит в ка­навки метчика, поворачивая оправку с помощью воротка за квадратный хвостовик;

2) с помощью специально­го зенкера (рис. 5, б);

3) если мет­чик поломан в детали из силумина или алюминия то на обломок метчика (рис. 5, в) электродом наплавляют (наращива­ют) хвостовик, а после охлаждения метчик вывёртывают из отверстия; с помощью ключа, надеваемо­го на квадратный конец специ­альной оправки, приваренной к поломанному метчику (рис. 5, г); путём травления (из алюминиевой детали). В теле метчика высверлива­ют отверстие, стараясь не повредить резьбу детали. Травят раствором азот­ной кислоты, которая хорошо раство­ряет сталь (материал метчика), не­значительно действует на алюминие­вый сплав (материал детали).

10

Как качественно нарезать резьбу и предупредить возможности возникновения брака?

Причинами неудовлетвори­тельного качества нарезаемой резьбы, часто приводящему к браку, являются:

1. Рваная резьба. Причины: тупой метчик или плашка; неудовлет­ворительное охлаждение; перекос метчика или плашки относительно от­верстия.

2. Тупая резьба. Причины: большой диаметр просверленного от­верстия под резьбу или мал диаметр стержня.

3. Неточный профиль резьбы. Причины: тупой или неправильно заточенный инструмент; несоответ­ствие смазочно-охлаждающей жидко­сти; чрезмерно высокая скорость ре­зания; недостаточная длина заборного конуса.

4. Ослабленная резьба. Причины: разбивание резьбы метчи­ком при неправильной его установке; биение инструмента; повышенная ско­рость резания.

5. Тугая резьба. Причины: диаметр инструмента не соответствует заданному диаметру резьбы.

6. Срыв резьбы. Причины: диаметр просверленного отверстия под резьбу меньше требуемого; затупился метчик; стружка забивает канавки метчика.

Все перечисленные дефекты могут быть предупреждены при добросовестном отношении к работе и устранении причин их возникновения.

 

Практическая работа №9

Цель работы:

Освоение и приобретение практических навыков слесарной операции – ручная и машинная клепка металлов.

 

 

 

Вопросы

Ответы

1

Нhttps://fsd.multiurok.ru/html/2020/01/02/s_5e0e03831494c/1303586_1.pngазовите виды заклёпок, по­казанные на рис. 1.

 

 

Рис. 1. Виды заклёпок

На рис. 1 показаны следую­щие типы заклёпок: а — с полукруг­лой высокой головкой,

б — с полукруг­лой низкой головкой, в — с плоской головкой, г — с потайной головкой, д — с полупотайной головкой, е — взрывная двухкамерная, ж — заклёпка с сердечником.

2

Нhttps://fsd.multiurok.ru/html/2020/01/02/s_5e0e03831494c/1303586_2.png арисуйте показанные на рис. 2, д-е виды заклёпок и назо­вите их элементы.

Рис. 2. Заклёпочные соединения

а — однорядные внахлёстку, б — однорядные встык с одной накладкой, в — однорядные встык с двумя накладками, г — двухрядный с шахматным расположе­нием заклёпок встык с одной накладкой, д — с по­тайной головкой, е — с полукруглой головкой

На рис. 2, д дана заклёпка с потайной головкой,

а на рис. 2, е — с полукруглой головкой:

1 — замы­кающая головка, 2 — стержень, 3 — закладная головка.

3

Как различаются заклёпочные соединения по характеру располо­жения соединяемых деталей (рис. 2, а-г)?

По характеру расположения соединяемых деталей различают швы; а — однорядные внахлёстку (рис. 2, а), б — однорядный встык с одной накладкой (рис. 2, б), в — одноряд­ный встык с двумя накладками (рис. 2, в).

4

Длина заклёпки l (в мм) с полукруглой головкой (рис. 3, а) определяется по форму­ле: l = s + (1,2…1,5)d, где s — общая толщина склёпываемых листов детали, мм; d — диаметр стержня заклёпки, мм.

Пример А. К балке необходимо прикрепить равнобокий уголок сталь­ными заклёпками с полукруглой головкой. Шов однорядный, диаметр заклёпки 16 мм, толщина уголка 20 мм, толщина борта балки, к которой приклёпывается уголок, 24 мм. Определи­те длину заклёпки.

Длина заклёпки l с потайной го­ловкой (рис. 3, б) определяется по формуле: l = s + (0,8… 1,2)d.

Пример Б. Отсек корабля склёпывается стальными заклёпками диамет­ром 16 мм с потайной головкой. Надо определить длину заклёпок, если из­вестно, что толщина одного из склёпываемых элементов 12 мм, а второго 14 мм.

На рис. 4 изображена номо­грамма для определения длины стерж­ня заклёпок с полукруглой головкой. Для определения длины нужно прило­жить линейку к делениям правой и ле­вой шкал, соответствующим общей толщине склёпываемых деталей, циф­ры в прямоугольниках, пересекаемых линейкой, показывают нужную длину стержня заклёпки для каждого ди­аметра.

https://fsd.multiurok.ru/html/2020/01/02/s_5e0e03831494c/1303586_3.png

 

Рис. 3. Определение длины заклёпок

А. Общая длина заклёпок с по­лукруглой головкой (рис. 162, а): l = s + (1,2…1,5)= 68 мм.

Б. Общая длина заклёпок с потайной головкой (рис. 162, б): l = s + (0,8… 1,2)d = 12 + 14 + 1,2 × 16 = 45,2 мм.

5

Определите длину стержня заклёпки с полукруглой головкой с помощью номограммы (рис. 4).

Рhttps://fsd.multiurok.ru/html/2020/01/02/s_5e0e03831494c/1303586_4.png ис. 4. Номограмма для опреде­ления длины стержня заклёпок

Для определения длины нужно прило­жить линейку к делениям правой и ле­вой шкал, соответствующим общей толщине склёпываемых деталей, циф­ры в прямоугольниках, пересекаемых линейкой, показывают нужную длину стержня заклёпки для каждого ди­аметра. На номограмме штриховой линией показан пример определения длины стержня заклёпки при склёпывании двух деталей, имеющих общую толщи­ну 4 мм. В этом случае:

 

Диаметр стержня

2,5

3 – 3,5

4

5

6

Длина стержня

7

8

10

11

12

6

Диаметр заклёпки подсчитыва­ется в зависимости от толщины склёпываемых листов по формуле: d = https://fsd.multiurok.ru/html/2020/01/02/s_5e0e03831494c/1303586_5.png , где s — толщина склёпываемых лис­тов, мм.

Пример. Определить диаметр заклёпки для склёпывания листов тол­щиной 3 и 5 мм.

Диаметр отверстия D под стер­жень заклёпки определяется по фор­муле:

а) для дюралюминиевых заклёпок диаметром до 4 мм D = d + 0,1 мм;

б) для дюралюминиевых заклёпок диаметром 4 – 10 мм D = d + 0,2 мм;

в) для стальных заклёпок диаметр заклёпок выбирается по справочным таблицам.

Диаметр заклёпки d = https://fsd.multiurok.ru/html/2020/01/02/s_5e0e03831494c/1303586_6.png  = https://fsd.multiurok.ru/html/2020/01/02/s_5e0e03831494c/1303586_7.png  = 4 мм.

7

Определите диаметр отверстия под стержень дюралюминиевых заклёпок диаметром до 4 мм и 4 – 10 мм.

Диаметр отверстия D под стержень заклёпки определяется по соответствующим формулам: для дюралюминиевой заклёпки диаметром до 4 мм D = d + 0,1 мм; для такой же заклёпки диаметром 4 – 10 мм D = d + 0,2 мм (d – диаметр заклёпки, мм).

8

Требуется приклепать лист ко­жуха котла к основанию рамы, необ­ходимо просверлить отверстия для заклёпок из дюралюминия. Опреде­лить диаметр отверстия, если диаметр каждой заклёпки равен 9,5 мм.

Диаметр отверстия для дюра­люминиевых заклёпок: D = d + 0,2 мм, D = 9,5 + 0,2 = 9,7 мм.

9

Шаг клёпки t (расстояние меж­ду центрами заклёпок) определяется по формуле:

а) для однорядных швов t = 3d мм;

б) для двухрядных швов t = 4d мм.

 

Практическая работа №10

Цель работы:

Освоение и приобретение практических навыков слесарной операции – шабрение и притирка.

 

 

Вопросы

Ответы

1

Кhttps://fsd.multiurok.ru/html/2020/01/02/s_5e0e03c6d3aa2/1303587_1.pngак выбирать углы заточки шаберов (рис. 1) для обработки чугуна и бронзы; стали; мягких металлов?

Рис. 1. Углы заточки шаберов для разных металлов

На рис. 1, а показана за­точка шабера для обработки чугуна и бронзы,

а на рис. 1, б — для обра­ботки стали.

Заточка шабера под уг­лом 35—40° (рис. 1, в) для черново­го шабрения мягких металлов пред­ложена новаторами В. С. Горбуновым и Н. И. Пахновым (завод им. С. Орд­жоникидзе), она позволяет увеличить толщину снимаемой стружки до 0,1 мм вместо 0,01 мм.

2

В чем особенности показанных на рис. 2, а, б, в шаберов?

https://fsd.multiurok.ru/html/2020/01/02/s_5e0e03c6d3aa2/1303587_2.png

 

 

 

Рис. 2. Шаберы со сменными режущими пластинками:

а — универсальный, б — конструкция С. Г. Кононенко,

в — с зажимным патроном; 1 — сменная пластинка,

2 — держатель, 3 — корпус, 4 — зажимный винт,

5 — рукоятка

Особенность шаберов со сменными режущими пластинками состоит в том, что использование плас­тинок из быстрорежущих сталей и твёрдых сплавов позволяет заменять их при затуплении. Применяя много­гранные пластинки, можно после за­тупления одной грани поворачивать их и таким образом значительно уве­личить производительность работы шабера.

 

3

Пользуясь рис. 3, опреде­лите:

а) какие классы шероховатости можно получить при шабрении поверх­ности;

б) определите для этого вида об­работки среднее арифметическое от­клонение профиля в микрометрах (Ra) и высоту неровностей в микрометрах {Rz);

в) базовую длину в миллиметрах, на которой надо производить конт­рольные замеры (профилометром).

На рис. 3 показано стрелка­ми, как при шабрении можно полу­чить 7, 8, 9-й классы шероховатости: среднеарифметическое отклонение Ra, равным 1,25, 0,63, 0,32; высоту не­ровностей Rz, равной 6,3; 3,2; 2,6 микрометра; базовая длина равна 0,8 мм,

4

Какими должны быть припуски на шабрение для плоскостей и отвер­стий?

Поскольку за каждый проход шабер снимает слой металла 0,05— 0,07 мм, нужно стремиться, чтобы при­пуски на пришабривание плоскостей отвечали данным табл. 1;

а на при­шабривание отверстий — табл. 2.

 

Припуски на шабрение плоскостей, мм

 

Ширина плоскости, мм

Длина плоскости, мм

До 100

0,10

0,15

0,20

0,25

0,30

100 – 150

0,15

0,20

0,25

0,30

0,40

 

Припуски на шабрение отверстий, мм

 

Диаметр отверстия, мм

Длина отверстия, мм

До 80

0,05

0,08

0,12

80 – 180

0,10

0,15

0,25

180 – 360

0,15

0,25

0,35

 

 

Упражнение 1. Подготовка к притирке

Проверить поверхность притирае­мой детали:

а) поверхность должна быть точно отшлифована или пришабрена (при­пуск на притирку 0,01—0,02 мм);

б) на поверхности не должно быть царапин, забоин.

Выбрать форму и размеры при­тира в зависимости от размеров и формы притираемой детали;

а) предварительную притирку вести на плите с канавками (а);

б) окончательную притирку вести на гладкой плите ( б).

Притираемую поверхность про­мыть бензином или керосином и насу­хо протереть ветошью.

Выбрать притирочный материал; для предварительной притирки приме­нять абразивный порошок Ш 1 или № 2, насыпать его в сосуд и хорошо перемешать с керосином или жидким минеральным маслом до получения полужидкой массы (кашицы).

https://fsd.multiurok.ru/html/2020/01/02/s_5e0df3d5e039b/1303527_1.png

https://fsd.multiurok.ru/html/2020/01/02/s_5e0df3d5e039b/1303527_2.png

 

 

 

 

 

 

https://fsd.multiurok.ru/html/2020/01/02/s_5e0df3d5e039b/1303527_3.png

https://fsd.multiurok.ru/html/2020/01/02/s_5e0df3d5e039b/1303527_4.pngПритирка:

 

а — плоский притир с канавками, б — плоский притир гладкий, в — шаржирование плоского притира, г — шаржирование круглого притира



 

 

 

 

 

Грубые порошки и пасты дают ма­товую поверхность, средние — полузеркальную, мелкие — зеркальную.

Перед началом притирки рабо­чую поверхность притирочной плиты смачивают керосином и насухо выти­рают. Затем на неё наносят тонким слоем абразивный порошок в виде по­лужидкой массы или пасты.

Подготовить (шаржировать) притиры:

а) плоский притир: 1) про­мыть керосином рабочую часть плиты и насухо протереть; 2) нанести на по­верхность притира тонкий равномер­ный слой притирочной массы; 3) ка­тать стальной закалённый валик на плоскости притира (в) 3— 5 раз вперёд и назад; 4) мягкой тряп­кой, смоченной в керосине, удалить избыток притирочной массы с поверх­ности притира;

б) круглый притир (диамет­ром более 10 мм): 1) промыть рабочую поверхность плиты керосином и насухо вытереть; 2) на плиту 1 насыпать тонкий ровный слой абразивного порошка; 3) притир 2 прокатывать с помощью другой плиты 3 (г) до тех пор, пока абразив не будет вдавлен в круглый притир равномерно по всей поверхности; 4) после шаржи­рования притира остатки абразивного порошка удалить, плиту слегка сма­зать и применять для работы без до­бавок до тех пор, пока притир не пе­рестанет обрабатывать деталь.

Упражнение 2. Притирка широких поверхностей

Выбрать притир в зависимости от формы и размеров притираемой де­тали,

Смочить керосином рабочие по­верхности притиров для предваритель­ной и окончательной притирки и на­чисто протереть тряпкой.

Подобрать порошки или пасту в зависимости от требуемой шероховато­сти.

Ни поверхность притира с канав­ками (а) нанести тампоном тонкий и равномерный слой притироч­ной массы.

Положить деталь притираемой плоскостью на притир с канавками (б) и с лёгким нажимом на деталь делать 5—8 круговых движе­ний по всей плоскости притира.

Удалить с поверхности притира (по окончании) избыток притирочной массы мягкой тряпкой, смоченной ке­росином.

https://fsd.multiurok.ru/html/2020/01/02/s_5e0df3d5e039b/1303527_5.pngЧистовую (окончательную) при­тирку выполнить на гладкой плите (без канавок) до получения матового или зеркального вида. Для получения блестящей поверхности притирку за­канчивают на притире из твёрдого де­рева, покрытом разведённой на спирте венской известью.

Примечание. Не допускать сильного нажима на деталь, от этого быстро изнашива­ется порошок, тонкая деталь прогибается, на­ступает быстрый износ плиты. Нажимы долж­ны быть равномерными по всей площади де­тали, а не только на их середине или конце.

После пяти—восьми круговых движений по одному и тому же месту сработанный абразив­ный порошок (или пасту) удалять с плиты чистой тряпкой и заменять новым.

Упражнение 3. Притирка узких поверхностей

 

https://fsd.multiurok.ru/html/2020/01/02/s_5e0df3d5e039b/1303527_6.pngПодобрать необходимые вспомо­гательные приспособления, предупреж­дающие завалы плоскостей и граней, отклонения от прямолинейности и уг­ла:

а) кубики (а), исключаю­щие отклонения от прямого угла;

 

 

 

 

б) призмы (б) определённой конфигурации; для притирки гра­ней и фасок; в) призмы для притирки фасок (в); г) прямоуголь­ники (г) для тонких деталей.

Подготовить притирочную плиту.

Положить прямоугольник на притирочную плиту.

Прижать деталь широкой плос­костью к прямоугольному бруску (д).

Прямоугольному бруску, совмещённому с притираемой деталью, со­общать возвратно-поступательное дви­жение.

Притирка криволинейных по­верхностей: а) подобрать притир по конфигурации формы притираемой де­тали; б) закрепить притир 3 (е) в слесарные тиски 5 с мягкими губками 4; в) для большей устойчиво­сти шаблона наложить на притир 3 направляющую призму (брусок) 2 и притираемый шаблон 1; г) взять обе­ими руками призму (брусок) с прижа­тым шаблоном и перемещать по при­тиру в продольном направлении (воз­вратно-поступательное движение).

Притирка узких поверхностей, расположенных под внутренним углом:

а) притир в форме угольника зажать в слесарные тиски с мягкими губками в горизонтальном положении; б) при­тираемую деталь прижать широкой боковой плоскостью к бруску (ж); в) бруску совместно с дета­лью сообщать возвратно-поступатель­ное движение; г) притёртую деталь проверить по вкладышу «на краску».

Упражнение 4. Притирка конических поверхностей

Проверить притираемые детали, которые предварительно должны быть обработаны с припуском на притирку.

Очистить притираемые поверх­ности от пыли, грязи и насухо выте­реть.

Зажать корпус крана в слесар­ных тисках конусным отверстием вверх (б).

https://fsd.multiurok.ru/html/2020/01/02/s_5e0df3d5e039b/1303527_7.pngВзять конический притир (пробку) с винтовыми канавками

(а).

Нанести на притир (пробку) ровным слоем абразивно-притирочный

ма­териал.

Ввести притир-пробку в прити­раемое отверстие.

На квадратный хвостовик прити­ра надеть вороток для вращения

при­тира-пробки (б).

Делать неполные обороты ворот­ка то в одну, то в другую сторону, а затем делать полный оборот.

Примечание Пробку не просто про­кручивать в коническом отверстии, а вращать и вместе с тем то вставлять до конца, то слег­ка вынимать. Это называется «делать притир­ку с пристукиванием». Иначе на детали могут образоваться глубокие кольцевые риски.

После 15—18 оборотов притир вынуть, насухо

протереть тряпкой, на­нести на него абразивно-притирочный материал и снова ввести в притирае­мое отверстие, продолжать совместную притирку пробки с краном до тех пор, пока притираемые поверхности не ста­нут матовыми (в).

Качество притирки проверить мелом или цветным карандашом: вдоль конической поверхности пробки провести мелом черту, вставить и про­вернуть на 1—2 полных оборота с лёгким нажимом. Если меловая черта стёрлась, значит, подгонка хорошая.

Более точный результат даёт провер­ка «на карандаш».

Притёртые поверхности прове­рить на отсутствие бликов, т. е. бле­стящих пятен на матовом фоне или на герметичность.

Проверить на герметичность: деталь насухо протереть и собрать вместе. Со стороны широкого конца пустить каплю керосина. При хорошей притирке керосин не пройдёт через отверстие.

Безопасность работы при притирке

Обрабатываемую поверхность очищать не рукой, а тряпкой (вето­шью).

Пользоваться защитными устрой­ствами для отсасывания абразивной пыли.

Осторожно обращаться с паста­ми, так как они содержат кислоты.

Надёжно и устойчиво устанав­ливать притиры.

Во время работы при установке в слесарных тисках притиров следует надёжно закреплять их (круглые и тяжёлые притиры могут упасть и вы­звать ушибы).

Типичные затруднения и ошибки учащихся и их предупреждение

При овладении приёмами притир­ки учащиеся испытывают очень боль­шие затруднения, если им выдают для притирки плохо рихтованные (правле­ные) детали или с неровными плос­костями. В этом случае много времени тратится на предварительную притир­ку и очень часто безрезультатно.

Учащиеся допускают ошибку, нано­ся на притир излишнее количество шлифующего порошка. Излишний по­рошок не ускоряет притирку детали, а, наоборот, замедляет её. Поэтому надо на шаржируемые притиры поро­шок наносить тонким, ровным слоем. Порошок обязательно надо вдавливать в поверхность притира. Проще всего это делать калёным валиком.

Нужно правильно охлаждать притир. Лучше всего применять керо­син и машинное масло.

При притирке конических сопряже­ний деталей допускается большая ошибка, когда притирают лишь путём вращения пробки без освобождения её из отверстия.

Правильная притирка производит­ся путём вращения детали в обе сто­роны с «пристукиванием», т. е. одно­временно с вращением пробка должна частично выниматься из отверстия и вводиться вновь в него. Только этим путём можно избежать образования рисок.

 

               Практическая работа №11

Цель работы:

Освоение и приобретение практических навыков по сборке и разборке разъемных соединений.

 

1. Соединение болтом, винтом

Болт и винт представляют собой цилиндрический стержень, один конец которого снабжен головкой, а другой имеет резьбу. Винты в отличие от болтов чаще всего имеют в головке шлиц, предусмотренный под отвертку. В зависимости от формы головки болты и винты бывают: с шестигранными (а), полукруглыми (б, е), цилиндрическими (в, ж), потайными (г, д) и другими головками.

Болты и винты с шестигранными головками применяют чаще других, так как они допускают большую силу затяжки и требуют небольшого поворота ключа до перехвата. Болты применяются для соединения деталей не очень большой толщины (фланцев и др.) и при необходимости частого соединения и разъединения деталей. В болтовое соединение входят болт, гайка, шайба. Болты с шестигранной головкой выполняют по ГОСТ 7798-80.

 

Винты применяются для неподвижного скрепления двух деталей или для предотвращения смещения одной детали относительно другой.

Винтовое соединение состоит из винта и двух скрепляемых деталей. В одной из деталей просверливается отверстие и нарезается резьба, а в другой детали выполняется отверстие несколько больше, чем диаметр стержня. Крепежные винты с цилиндрической головкой выполняют по ГОСТ 1491-80, винты с полукруглой головкой - по ГОСТ 17473-80, винты с полупотайной головкой - по ГОСТ 17474-80 и винты с потайной головкой - по ГОСТ 17475-80.

В зависимости от формы стержня болты и винты бывают: с нормальным стержнем (а), с подголовком (б), с утолщенным точно обработанным стержнем для постановки без зазора в развернутое отверстие (в), со стержнем уменьшенного диаметра не нарезанной части для повышения упругой податливости и выносливости при переменных нагрузках (г).

В зависимости от точности изготовления болты и винты бывают нормальной и повышенной точности. В зависимости от назначения болты бывают общего назначения, установочные, специальные.

Пример обозначения: Болт М 30х80 ГОСТ 7798-80

Пример обозначения: Винт М 20х100.5.8 ГОСТ 1491-80

Гайки

Гайка навинчивается на резьбовой гаечный конец болта, при этом соединяемые детали зажимаются между головкой болта и гайкой.

В зависимости от формы гайки бывают:

- шестигранные.

В зависимости от высоты шестигранные гайки бывают нормальные ГОСТ 5915-70 (а), высокие ГОСТ 15523-70 (б), низкие ГОСТ 5916-70 (в). Высокие гайки применяют при частых разборках и сборках для уменьшения износа резьб и обмятия граней гайки ключом. Прорезные и корончатые гайки ГОСТ 5918-73 (г) также выполняют высокими.

- круглые ГОСТ 11871-88 и гайка-барашек ГОСТ 3032-76.

В зависимости от точности изготовления гайки аналогично болтам бывают нормальными и повышенной точности.

Пример обозначения: Гайка М 30 ГОСТ 5915-70

Шайбы

Шайба представляет собой штампованное или точеное кольцо, которое подкладывается под гайку или головку болта.

Шайбы служат для предохранения деталей от задиров и увеличения опорной поверхности. Шайбы бывают точеные ГОСТ 6402-70 (а) и штампованные ГОСТ 11371-78 (б). Имеется большая группа стандартных стопорных шайб, которые применяют для предохранения резьбовых соединений от самоотвинчивания.

Пример обозначения: Шайба 30 ГОСТ 11371-78

Подбор деталей и выполнение болтового соединения. Болтовое соединение скрепляемых деталей выполняется с помощью болта, шайбы и гайки. В скрепляемых деталях сверлится отверстие для болта. В полученное отверстие вставляется болт, длина стержня и длина нарезанной части которого подбирается так, чтобы была возможность соединить его с гайкой. На выступающую из детали часть болта одевается шайба и навинчивается гайка до упора. Параметры болтового соединения рассчитываются в зависимости от диаметра резьбы болта и толщины скрепляемого пакета деталей.

 

Порядок выполнения:

1) Вычерчиваются исходные данные задачи с карточки задания согласно номеру варианта, расположив их в верхней части листа.

2) Определяется диаметр болта из условия:

dб=dотв-2мм

Полученный диаметр болта уточняется по таблице ГОСТ 7798-70.

3) Подсчитывается длина болта из условия:

L=A+B+Sш+H+0,2d, где

 

А – толщина фланца (одной из деталей соединения);

В – толщина крышки (другой детали соединения);

Sш – толщина шайбы;

Н – высота гайки;

0,2d – свободный конец болта.

Полученная длина болта L уточняется по табл.ГОСТ 7798-70 и округляется до ближайшего большего размера. По значениям d и L определяется длина резьбы Lo (табл. ГОСТ 7798-70).

4) Вычерчивается соединение. На чертеже соединения болт, гайку и шайбу в разрезе условно изображают не рассеченными. На чертеже болтового соединения наносятся пять размеров: А, В, d, L, S , где S – размер «под ключ». Наносятся номера позиций, заполняется спецификация, согласно ГОСТ 2.108 – 68 и основная надпись. Чертеж обводится.

2. Соединение шпилькой

Шпилька представляет собой цилиндрический стержень, имеющий с обоих концов (или по всей длине) резьбу. Соединение шпилькой применяют в случае, если соединяемые детали имеют значительную толщину или в случае отсутствия места для размещения болта. В соединение шпилькой входит стандартная шпилька , шайба, гайка, а также деталь, в которую ввинчивается шпилька, и присоединяемая деталь. Изготавливают шпильки без канавки (а) и с канавкой (б). В зависимости от материала детали, глубина завинчивания шпилек разная. При переменных нагрузках прочность шпилек выше, чем прочность болтов.

Длина ввинчиваемого резьбового конца – l1 зависит от диаметра шпильки и от ГОСТа на шпильку. Шпильки по ГОСТ 22032-76 предназначены для ввинчивания их в стальные, латунные, бронзовые детали и имеют l1=d. Шпильки по ГОСТ 22034-76 предназначены для ввинчивания в чугунные детали и имеют l1= 1,25d. Шпильки по ГОСТ 22036-76 также предназначены для ввинчивания в чугунные детали и имеют l1 = 1,6d. Шпильки по ГОСТ 22038-76 предназначены для ввинчивания в легкие (алюминиевые, магниевые) сплавы и имеют l1 = 2d. Шпильки по ГОСТ 22040-76 также предназначены для ввинчивания в легкие сплавы и имеют l1 = 2,5d.

Пример обозначения: Шпилька М 20х90 5.8 ГОСТ 22034-76

Подбор деталей и выполнение шпилечного соединения. Шпилечное соединение деталей выполняется с помощью шпильки, гайки и шайбы. Данное соединение используется главным образом в тех случаях, когда конструкция детали не дает возможности образовывать на ней опорные площадки для головок болтов или когда одна из соединяемых деталей имеет значительную толщину. Параметры шпилечного соединения рассчитываются в зависимости от диаметра резьбы шпильки, толщины присоединяемой детали и материала детали, в которой сверлится гнездо. В одной из скрепляемых деталей сверлят глухое отверстие, в котором нарезают резьбу, в другой – отверстие большего диаметра, чем диаметр шпильки.

Порядок выполнения:

1) Перечерчиваются исходные данные задачи с карточки задания, расположив

их в верхней четверти листа.

2) Определяется диаметр резьбы шпильки - он равен размеру диаметра

резьбового отверстия, указанного в задании.

3) Длина шпильки подсчитывается:

L=A+Sш+H+0,3d, где

А – толщина присоединяемой детали;

Sш – толщина шайбы;

Н – толщина гайки;

0,3d – свободный конец шпильки.

Полученная длина шпильки уточняется по ГОСТ 22032-76 и округляется до ближайшего большего размера. По значениям d и l подбирается длина резьбового (гаечного) конца шпильки lо.

4) Далее вычерчивается соединение. На чертеже соединения шпильку, гайку и шайбу условно изображается не рассеченными и упрощенно. Шпилька вворачивается в резьбовое отверстие детали на глубину l1, которая выбирается в зависимости от прочности материала, из которого изготовлена деталь: для стали, бронзы, латуни l1=d; для ковкого чугуна l1=1,25d; для легких сплавов l1=2d; 2,5d и т.д.

На чертеже шпилечного соединения наносятся четыре размера: А, l, d, S, где S – размер «под ключ». Наносятся номера позиций, заполняется спецификация и основная надпись. Чертеж обводится.

 

Практические работы по МДК.01.02.01  Устройство, техническое обслуживание и ремонт автомобилей категории "С"

 

Практическая работа №1

 

Тема – Устройство коробки перемены передач, разработка технологической карты.

Цель занятия: изучить на примере устройства сборочных единиц коробки перемены передач. Приобрести навыки в разборке и сборке коробки перемены передач легкового автомобиля используя инструкционно – технологическую карту приложение №6. Заполнить таблицы 5.1., 5.2.

Иллюстративный материал – учебные плакаты.

Оборудование: коробка перемены передач легкового автомобиля, Приспособление для разборки сборки сцепления, для выпрессовки крышки подшипника, съемник трехлапный, комплект инструментов, выколотка из мягкого металла (например, медная, бронзовая), коловорот, пассатижи, ключи специальные торцовые, комплект рабочих инструментов, штангенциркуль.

Таблица 5.1. Основные характеристики

Тип КПП

Передаточные числа

Толщина накладки диска сцепления

(нормативная)

Толщина накладки диска сцепления (измеренная)

Материал изготовления шестерен КПП

(марка)

1

2

3

4

5

R

Таблица 5.2. Технологическая карта

 

Наименование

Время, мин

Инструмент приспособление

Трудоемкость, чел\час

Примечание

 

 

 

 

 

 

 

 

 

 

 

 

 

 

Выводы: ________________________________________________________________________________________________________________________________________________________________

________________________________________________________________________________________________________________________________________________________________________________________________________________________________________________________________________________________________________________________________________________________________________________________________________________________________________________________________________________________________________________________________________________________________________________________________________________________________________________________________________________________________________________________________________________________________________________________________________________________________________________________________________________________________________________________________________________________________________________________________________________________________________________________________________________________________________________________________________________________________________________________________________________________________________________________________________________________________________________________________________________________________________________________________________________________________________________________________________________________________________________________________________________________________________________

Контрольные вопросы:

1. Для чего служит сцепление?

2. С какой целью под нажимные пружины сцепления устанавливают шайбы

и из какого материала они изготовлены?

3. Почему для разборки сцепления его устанавливают на специальное приспособление?

4. Почему при износе фрикционных накладок сцепление «буксует»?

5. Укажите назначение коробки передач.

Тема – Устройство карданной передачи, разработка технологической карты.

Цель занятия: изучить на примере устройства сборочных единиц карданной передачи измерить основные размеры. Приобрести навыки в разборке и сборке карданной передачи легкового автомобиля используя инструкционно – технологическую карту приложение №7. Заполнить таблицы 6.1., 6.2.

Иллюстративный материал – учебные плакаты.

Оборудование: карданная передача легкового автомобиля, комплект инструментов, съемник, ключи специальные торцовые, штангенциркуль, весы электронные.

Таблица 6.1.

Тип

Диаметр внутренний, мм

Диаметр наружный, мм

Масса, кг

Критическая частота вращения

Таблица 5.2. Технологическая карта

Наименование

Время, мин

Инструмент приспособление

Трудоемкость, чел\час

Примечание

 

 

 

 

 

 

 

 

 

 

 

 

 

Выводы: ________________________________________________________________________________________________________________________________________________________________

________________________________________________________________________________________________________________________________________________________________________________________________________________________________________________________________________________________________________________________________________________________________________________________________________________________________________________________________________________________________________________________________________________________________________________________________________________________________________________________________________________________________________________________________________________________________________________________________________________________________________________________________________________________________________________________________________________________________________________________________________________________________________________________________________________________________________________________________________________________________________________________________________________________________________________________________________________________________________________________________________________________________________________________________________________________________________________________________________________________________________________________________________________________________________________________________________________________________________________________________

Контрольные вопросы:

1. Что входит в устройство карданной передачи?

2. Каких типов могут быть карданные шарниры?

3. Как устроен и работает жесткий карданный шарнир?

4. Что представляет собой упругий полукарданный шарнир?

5. Как устроена и работает промежуточная опора?

 

Практическая работа №2

 

Цель: состоит в расширении и углублении полученных знаний применительно к конструкциям автомобилей и средствам их испытаний,

Задачи: заключаются в усвоении общих принципов устройства и работы механизмов, установок для их испытаний и на этой основе – в практическом изучении конструкций конкретной базовой модели электромеханического усилителя рулевого управления.

Литература: Методическое указание по выполнению лабораторных работ [].

 

1. Изучить назначение и общее устройство рулевого управления переднеприводного легкового автомобиля (на примере автомобиля ВАЗ – 1118 «Калина»). Написать наименование в таблицу 9.1. отображенных на рисунке №3.

2. Изучить назначение и общее устройство электромеханического усилителя рулевого управления переднеприводного легкового автомобиля (на примере автомобиля ВАЗ – 1118 «Калина»). Написать наименование в таблицу 9.2. отображенных на рисунке №4.

Таблица 9.1 Общее устройство рулевого управления

Рисунок 3. Рулевой механизм в сборе

https://fsd.multiurok.ru/html/2019/03/13/s_5c88c74431342/1112103_3.jpeg

Наименование

Принцип работы, функция

1

2

3

4

5

6

7

8

9

10

11

Таблица 9.2.

 

Рисунок 4. Схема электромеханического усилителя с двумя шестернями

https://fsd.multiurok.ru/html/2019/03/13/s_5c88c74431342/1112103_4.jpeg

Наименование

Принцип работы, функция

1

2

3

4

5

6

7

8

 

3. Определить основные неисправности ЭМУР переднеприводного легкового автомобиля (на примере автомобиля ВАЗ – 1118 «Калина»). Провести диагностику неисправностей ЭМУР.

4. С использованием лабораторного стенда произвести ознакомления с основными режимами работу ЭМУР переднеприводного легкового автомобиля (на примере автомобиля ВАЗ – 1118 «Калина») в зависимости от дорожных условий.

5. С использованием лабораторного стенда произвести определение основных характеристик работы ЭМУР переднеприводного легкового автомобиля (на примере автомобиля ВАЗ – 1118 «Калина») в зависимости от дорожных условий. Написать значения в таблицу 9.3., 9.4.

 

Таблица 9.3.

V, км/час

I, А

Мр, Н*м

0

10

20

30

40

50

100

 

 

Таблица 9.4.

G, H

I, А

Мр, Н*м

7000

8000

9000

9500

10000

12000

14000

 

Выводы: _______________________________________________________________________________________________________________________________________________________________________________________________________________________________________________________________________________________________________________________________________________________________________________________________________________________________________________________________________________________________________________________________________________________________________________________________________________________________________________________________________________________________________________________________________________________________________________________________________________________________________________________________________________________________________________________________________________________________________________________________________________________________________________________________________________

Контрольные вопросы:

1. Принцип работы рулевого управления?

2. Кинематическая схема рулевого управления?

3. Основные элементы рулевого управления?

4. Принцип работы и кинематическая схема ЭМУР?

5. Достоинства ЭМУР?

6. Датчики сигнализаторы ЭМУР?

7. Режим работы ЭМУР?

8. Основные неисправности рулевого управления и методы их устранения?

9.Перечислите основные этапы проверки состояния рулевого управления?

10.Перечислите основные режимы работы ЭМУР?

 

Практическая работа №3

Цель: - Закрепить теоретические знания по устройству и работе тормозных систем;

-Формировать умение определять типы приборов и механизмов тормозных систем.

Порядок выполнения работы

1. Изучить назначение, устройство и работу тормозных механизмов автомобилей

2. Рассмотреть и уметь объяснить следующие схемы:

2.1 Конструкцию барабанно-колодочных тормозных механизмов

2.2 Конструкцию дисковых тормозных механизмов

2.3 Регулировку зазоров между фрикционными накладками тормозных колодок и барабаном (диском)

3. Изучить назначение, устройство и работу приборов гидравлического привода тормозов автомобилей

4. Рассмотреть и уметь объяснить следующие схемы:

4.1 Конструкцию главного тормозного цилиндра

4.2 Конструкцию колесных тормозных цилиндров

4.3 Конструкцию регулятора тормозных сил

4. 4 Путь тормозной жидкости при торможении

5. Рассмотреть и уметь объяснить следующие схемы:

5.1 Конструкцию гидровакуумного усилителя тормозного привода

5.2 Конструкцию вакуумного усилителя тормозного привода

5.3 Работу гидровакуумного усилителя при торможении

5.4 Работу гидровакуумного усилителя при растормаживании

5.5 Работу вакуумного усилителя при торможении

5.6 Работу вакуумного усилителя при растормаживании

6. Выписать основные параметры, характеризующие усилители тормозных приводов изучаемых автомобилей

6.1 .Тип усилителя и место его установки

6.2.Конструктивные особенности усилителей тормозного привода

7.Изучить назначение, устройство и работу приборов общего участка питания контуров сжатым воздухом

8.Рассмотреть и уметь объяснить следующие схемы:

8.1Конструкцию компрессора

8.2Конструкцию регулятора давления

8.3Конструкцию предохранителя от замерзания

9.Рассмотреть и уметь объяснить следующие схемы:

9.1 Конструкцию тормозных камер.

9.2 Конструкцию и работу двухсекционного тормозного крана

9.3 Работу тормозных камер при торможении и растормаживании

10. Выполнить практическую работу: -на основании классификационных схем сравните типы тормозных систем, результаты занесите в таблицу:

Технические параметры

ВАЗ

ГАЗ

ГАЗ

ЗиЛ

Урал

КамАЗ

Тип тормозных механизмов: -передний мост -задний мост (задняя тележка)

 

 

 

 

 

 

Тип разжимного устройства тормозных механизмов

 

 

 

 

 

 

Чем регулируются технологический зазор в тормозных механизмах

 

 

 

 

 

 

Тип тормозного привода

 

 

 

 

 

 

Число контуров тормозного привода

 

 

 

 

 

 

Тип усилителя тормозного привода (при наличие)

 

 

 

 

 

 

Тип стояночной тормозной системы

 

 

 

 

 

 

Тип привода стояночной тормозной системы

 

 

 

 

 

 

Наличие регулятора тормозных сил

 

 

 

 

 

 

Наличие вспомогательной тормозной системы

 

 

 

 

 

 

 

 

 

 

 

 

 

 

 

 

 

11.Составить отчет о работе в соответствии с пп.1 - 10, дать ответ на контрольные вопросы

Контрольные вопросы:

1. Каково принципиальное отличие барабанных и дисковых тормозных механизмов?

2. Какие типы крепления колодок барабанного тормозного механизма применяются на автомобилях?

3. Какими способами осуществляется раздвигание тормозных колодок на различных автомобилях?

4. Какие типы стояночных тормозных механизмов применяются на автомобилях?

5. Какими способами осуществляется регулирование зазоров между тормозными колодками и барабаном на различных автомобилях?

 

 

 

Практические работы по МДК.01.03   Ремонт транспортных средств

 

Практическая работа № 1

Цель занятия: формировать умения и навыки чтения и расчета линейных размеров и посадок.

Выполнение задания:

1) Ознакомтесь со справочным материалом.

2) Выполните задание 1 по вариантам ((варианты задания в таблице 4.1).

- Выполните эскиз соединения 1 (рисунок 1)

- Определите характер соединения (смотри пример 1).

3) Выполните задание 2 по вариантам ((варианты задания в таблице 4.2).

- Выполните эскиз соединения 2 (рисунок 2)

- Определите характер соединения (смотри пример 2).

 

Таблица 4.1. Варианты задания 1

Варианты

1.                               

2.                               

3.                               

4.                               

5.                               

Задание

Ø200

Ø25

Ø50

Ø80

Ø10

Варианты

6.                               

7.                               

8.                               

9.                               

10.                           

Задание

Ø20

Ø15

Ø12

Ø25

Ø175

 

Таблица 4.2. Варианты задания 2

Варианты

1.                               

2.                               

3.                               

4.                               

5.                               

Задание

Ø25

Ø 75

Ø 50

Ø 34

Ø 65

Варианты

6.                               

7.                               

8.                               

9.                               

10.                           

Задание

Ø 67

Ø 28

Ø 37

Ø 45

Ø 175

hello_html_1ce33bd0.jpg

Рисунок 1. Эскиз соединения 1

hello_html_1ce33bd0.jpg

Рисунок 2. Эскиз соединения 2

 



Пример 1

Задание: Определить характер соединения

1.                              Укажите обозначение сопрягаемого размера на чертеже

2.                              Укажите номинальный размер сопрягаемых;

3.                              Укажите верхнее и нижнее предельные отклонения;

4.                              Определите предельные размеры;

5.                              Начертите графическое изображение посадки;

6.                              Определите характер соединения;

7.                              Рассчитайте основные параметры посадки.

 



Сопрягаемый размер

Деталь 1

(Отверстие)

 

Деталь 2

(Вал)

 

Номинальный размер соединения

D = 48мм

d = 48 мм

Верхнее отклонения

ES = +0,064 мм

es = 0 мм

Нижнее отклонения

EI = +0,025 мм

ei = -0,016 мм

Верхний предельный размер

Dmax = D + ES = 48,064 мм

dmax = d + es = 48,000 мм;

Нижний предельный размер

Dmjn = D + EI = 48,025 мм;

dmin = d + ei = 47,984 мм;

Допуск размера

ТD = Dmax - Dmin = 0,039 мм

или

TD = ES - EI = 0,039 мм;

Тd= dmax- dmm = 0,016 мм;

или

Td = es - ei = 0,016 мм;

Графическое изображение посадки:

 

Характер соединения: посадка с зазором.

* Дальнейшее выполнение задания зависит от характера соединения (смотри справочный материал «Соединение 1» для посадки с зазором, «Соединение 2» для посадки с натягом, «Соединение 3» для посадки переходной).

Основные параметры посадки:

Наибольший зазор

Smax=Dmax-dmin=0,080 мм или Smax = ES - ei = 0,080 мм.

 

Наименьший зазор

Smin = Dmin - dmax = 0,025 мм или Smin = EI - es = 0,025 мм

 

Диапазон посадки с зазором

TS = Smax - Smin = 0,055 мм или TS = TD + Td = 0,055 мм



Пример 2

Задание: Определить характер соединения

1.                              Укажите обозначение сопрягаемого размера на чертеже

2.                              Укажите номинальный размер сопрягаемых размеров;

3.                              Определите верхнее и нижнее предельные отклонения;

4.                              Определите предельные размеры;

5.                              Начертите графическое изображение посадки;

6.                              Определите характер соединения;

7.                              Рассчитайте основные параметры посадки.

Деталь _____ Деталь _____

Сопрягаемый размер

(Отверстие)

48F8

(Вал)

48h6

Номинальный размер соединения

D = 48мм

d = 48 мм

Верхнее отклонения (определяем по справочнику)

ES = +0,064 мм

es = 0 мм

Нижнее отклонения (определяем по справочнику)

EI = +0,025 мм

ei = -0,016 мм

Верхний предельный размер

Dmax = D + ES = 48,064 мм

dmax = d + es = 48,000 мм;

Нижний предельный размер

Dmjn = D + EI = 48,025 мм;

dmin = d + ei = 47,984 мм;

Допуск

ТD = Dmax - Dmin = 0,039 мм

или

TD = ES - EI = 0,039 мм;

Тd= dmax- dmm = 0,016 мм;

или

Td = es - ei = 0,016 мм;

Графическое изображение посадки:

 

Характер соединения: посадка с зазором.

* Дальнейшее выполнение задания зависит от характера соединения (смотри справочный материал «Соединение 1» для посадки с зазором, «Соединение 2» для посадки с натягом, «Соединение 3» для посадки переходной).

Основные параметры посадки:

Наибольший зазор

Smax=Dmax-dmin=0,080 мм или Smax = ES - ei = 0,080 мм.

 

Наименьший зазор

Smin = Dmin - dmax = 0,025 мм или Smin = EI - es = 0,025 мм

 

Диапазон посадки с зазором

TS = Smax - Smin = 0,055 мм или TS = TD + Td = 0,055 мм





СПРАВОЧНЫЙ ЛИСТ

 

Посадкой - характер соединения двух деталей, определяемый величиной получающихся в нем зазоров или натягов. Различают посадки трех типовс зазором, с натягом и переходные.

ОПРЕДЕЛЕНИЕ ОСНОВНЫХ ЭЛЕМЕНТОВ ПОСАДОК

hello_html_7f1a2948.jpgРисунок 3 а,б- Поршневая группа в сборе;

 

Соединение 1 –

посадка с зазором

 

 

(Отверстие)

 

(Вал)

 

Номинальный размер соединения

D = 48мм

d = 48 мм

Верхнее отклонения

ES = +0,064 мм

es = 0 мм

Нижнее отклонения

EI = +0,025 мм

ei = -0,016 мм

Верхний предельный размер

Dmax = D + ES = 48,064 мм

dmax = d + es = 48,000 мм;

Нижний предельный размер

Dmjn = D + EI = 48,025 мм;

dmin = d + ei = 47,984 мм;

Допуск

ТD = Dmax - Dmin = 0,039 мм

или

TD = ES - EI = 0,039 мм;

Тd= dmax- dmm = 0,016 мм;

или

Td = es - ei = 0,016 мм;

Графическое изображение посадки:

 

Характер соединения: посадка с зазором.

Основные параметры посадки:

Наибольший зазор

Smax=Dmax-dmin=0,080 мм или Smax = ES - ei = 0,080 мм.

 

Наименьший зазор

Smin = Dmin - dmax = 0,025 мм или Smin = EI - es = 0,025 мм

 

Диапазон посадки с зазором

TS = Smax - Smin = 0,055 мм или TS = TD + Td = 0,055 мм

Соединение 2-

посадка с натягом

 

 

 

(Отверстие)

 

 

(Вал)

Номинальный размер соединения

D = 53 мм;

d = 53 мм;

Верхнее отклонения

ES = +0,030 мм;

es = + 0,083мм;

Нижнее отклонения

EI = 0 мм;

ei = + 0,053 мм;

Верхний предельный размер

Dmax = D + ES = 53,03 мм;

dmax = d + es = 53,083 мм;

Нижний предельный размер

Dmin = D + EI = 53,00 мм;

dmin = d + ei = 53,053 мм;

Допуск

ТD = Dmax - Dmin = 0,03 мм

или TD = ES - EI = 0,03 мм.

Тd = dmax - dmm = 0,03 мм;

или Td = es - ei = 0,03 мм;

Графическое изображение посадки:

 

Характер соединения: посадка с натягом.

Основные параметры посадки:

Наибольший натяг

Nmax = dmax - Dmin = 0,083 мм; или Nmax es - EI = 0,083 мм.

Наименьший натяг

Nmin = dmin - Dmax = 0,023 мм; или Nmin ei - ES = 0,023 мм.

Диапазон посадки с натягом

ТN = Nmax - Nmin = 0,06 мм; или TN = TD + Td = 0,06 мм.

 

 

Соединение 3-

посадка переходная

 

 



(Отверстие)

 



(Вал)

 

Номинальный размер соединения

D = 48 мм

d = 48 мм

Верхнее отклонения

ES = -0,012 мм

es = 0

Нижнее отклонения

EI = -0,028 мм

ei = -0,016 мм

Верхний предельный размер

Dmax = D + ES = 47,988 мм

dmax = d + es = 48,000 мм

Нижний предельный размер

Dmin = D + EI = 47,972 мм

d min = d + ei = 47,984 мм

Допуск

TD.= Dmax -Dmin = 0,016 мм

или ТD = ES - EI = 0,16 мм

Td = dmax - dmin = 0,016 мм

или Td = es - ei = 0,016 мм

Графическое изображение посадки:

 

Характер соединения: посадка переходная.

Основные параметры посадки:

Наибольший зазор Smax = Dmax - dmin = 0,004 мм или max = ES - ei = 0,004 мм.

Наибольший натяг Nmax = dmax - Dmin = 0,028 мм или Nmax = es - EI = 0,028 мм.

Диапазон переходной посадки Т(SN) = Smax + Nmax = 0,032 мм или Т(SN) = TD + Td = 0,032 мм.

 

Практическая работа №2

Цель: Формирование умений и навыков при выполнении ремонтных работ с применением полимерных материалов

Оборудование: автомобиль ВАЗ — 2106комплект водительского инструмента. Комплект рихтовочного инструмента

Методические указания: используются учебные пособия:

1. Ламака Ф. И, Лабораторно-практические работы по устройству грузовых автомобилей: учебное пособие для н.п.о., М.: издательский центр Академия, 2007, 2008

2.Ремонт автомобиля ВАЗ (DVD)

3.Шестопалов С.К. Устройство, техническое обслуживание и ремонт легковых автомобилей Учебник для начального профессионального образования, М.: ИРПО; Изд. центр «Академия», 2007

4. Руководство по эксплуатации и ремонту автомобиля ВАЗ — 2106.

Содержание работы

 

Заделка трещин и пробоин в деталях. Технологический процесс заделки трещин и пробоин в деталях или устранения повреждений кавитационного характера в корпусных деталях, таких, как корпуса турбокомпрессоров и воздуходувок, блоки цилиндров и др., отличается от процесса наращивания деталей пастами, главным образом, предварительной разделкой мест повреждения.

По концам трещин при их разделке сверлят отверстия диаметром 3...4 мм, а кромки трещин раскрывают под углом 60...90°. Если необходима высокая прочность детали, шов по длине усиливают скобами, через каждые 20... 30 мм сверлят технологические отверстия для клеевых заклепок  или на шов накладывают 2... 3 слоя стеклоткани. При разделке пробоин острые кромки у краев притупляют, а у корпусных деталей с толстыми стенками по периферии пробоины через каждые 20...30 мм сверлят отверстия диаметром 2...3 мм для клеевых заклепок. Зону вокруг разделки трещины или пробоины на расстоянии 15...20 мм зачищают до металлического блеска. Поверхности для наращивания и нанесения пасты подготавливают как обычно.

При заделывании трещин наносят два слоя пасты. Второй слой наносят с таким расчетом, чтобы он заполнил всю трещину и перекрыл ее по обеим сторонам на 10... 15 мм при толщине слоя 2... 3 мм. Стеклоткань толщиной 0,1... 0,3 мм накладывают между слоями пасты и обязательно прокатывают роликом.

Заделывание пробоин начинают с заполнения пастой просверленных отверстий и намазывания пасты вокруг пробоины. Пробоину закрывают стальной накладкой, слегка ее прижимают и наносят поочередно 2...3 слоя пасты, каждый из которых покрывают любой сетчатой тканью и прокатывают роликом. При заделке пробоины заподлицо, чтобы паста не проваливалась, снизу к детали приклеивают или удерживают на проволоке

 

 

https://fsd.multiurok.ru/html/2017/11/20/s_5a12b79dbdaa1/748545_1.png

 

 

 

 

Защита отчета по лабораторной работе.

 

 

Практическая работа №3

Цели занятия:

Образовательная: Изучить  технологический процесс очистки и мойки деталей.

Воспитательная: воспитывать и прививать интерес к будущей профессии.  

Развивающая: Научиться применять знания на практике

Литература: Бахолдин В.И. Технология ремонта тепловозов и дизель-поездов/ В.И. Бахолдин  (1-е изд.) учебник2013г., - 352 с. Допущено Экспертным советом по профессиональному образованию в качестве учебного пособия для использования в учебном процессе образовательных учреждений, реализующих программы среднего профессионального образования.

Теоретическая часть.

          На заводах локомотивы очищают и моют на технологических тележках в камере. Сначала секцию подготавливают к очистке. Подготовка заключается в следующем: уплотняют щели размером более 1 мм, а также места возможного попадания обмывочной жидкости. Концы проводов, находящихся под рамой, изолируют резиновыми чехлами. Секция подается конвейерной установкой, при помощи которой она 3—4 раза совершает возвратно-поступательное движение со скоростью 0,7 м/мин. Для очистки наружной поверхности кузова и рамы используют 3-процентный раствор каустической соды (NaOH), нагретый до 85—90° С. Затем обмывают чистой горячей водой и сушат подогретым воздухом. Обмытая секция выводится из камеры. После этого удаляют съемную часть кузова, демонтируют оборудование и в этой же камере обмывают внутренние поверхности и сушат в той же последовательности, как и при наружной обмывке, предварительно покрыв части, подверженные коррозии, раствором нитрита натрия.
        На некоторых заводах обмывку рамы подвижного состава производят снизу струями воды давлением 100 кГ/см2, подаваемой во вращающуюся конструкцию, снабженную соплами.

        В моечных машинах различного типа блоки дизелей, тележки и др. обмывают 2—3-процентным раствором каустической соды в первой камере, а затем горячей водой во второй. Между камерами установлена промежуточная камера, предупреждающая смешивание моечного раствора и воды. По концам машины расположены рольганги, служащие для размещения очищаемых узлов и деталей. Лента конвейера у машины, применяемой на заводах, имеет две скорости движения (0,211 и 0,323 м/мин). Щелочной раствор и вода циркулируют по замкнутому циклу. Напор в обмывочных соплах создается насосами, работающими на каждую из камер. Пары удаляются вытяжными установками, а грязь—при помощи фильтров, отстойников и гидроциклонов. Температура воды и раствора контролируется дистанционными термометрами. Управление машинами дистанционное. В депо используются моечные машины типа ММД6 и ММД12Б.

         Накипь имеет различный состав, поэтому и средства для ее удаления разнообразны. Накипь различают: карбонатную с большим содержанием углекислого кальция, гипсовую, характеризующуюся содержанием сернокислого кальция, силикатную, где преобладает окись хрома, и смешанную. Карбонатную и гипсовую накипи удаляют раствором соляной или хромовой кислоты. В раствор добавляют пассиваторы, замедляющие действие кислоты на металл (костный клей, фурфурол и др.). Для растворения силикатной накипи применяют 2—3-процентный раствор каустической соды, подогретый до 30° С.
        Чтобы удалить накипь любого состава, применяют 3—5%-процентный раствор тринатрийфосфата (Na3PO4). После разрыхления накипи ее удаляют проточной водой. Детали из алюминиевого сплава очищают от накипи раствором, состоящим из 100 г фосфорной кислоты (Н3РО4) и 50 г хромового ангидрида (СrО3) на 1 л воды.О

Выполнение задания

1.Описать процесс  подготовки секции локомотива к очистки.

___________________________________________________________________________________________________________________________________________________________________________________________________________________________________________________________________________________________________________________________________________________________________________________________________________________________________________________________________________________________________________________________________

2. Описать процесс очистки и мойки деталей.

__________________________________________________________________________________________________________________________________________________________________________________________________________________________________________________________________________________________________________________________________________________________________________________________________________________________________________________________________________________________________________________________________________________________________________________________________________________________

 

КОНТРОЛЬНЫЙ ВОПРОС

1. Описать правильное измерение износа и используемые средства измерения при этом.

______________________________________________________________________________________________________________________________________________________________________________________________________________________________________________________________________________________________________________________________________________________________________________________________________________________________________________________________________________________________________________________________________________________________________________________________________________________________________________________________________________________________________________________________________________________________________________________________________________________________________________________________________________________________________________________________________________________________________________________________________________________________________________________________________________________________________________________________________________________________

 

ОФОРМЛЕНИЕ РЕЗУЛЬТАТОВ ЗАНЯТИЯ

             Отчет по выполнению практического занятия на тему:_______________________________________________________________________________________________________________________________________________________________________

 

Выводы:__________________________________________________________________________________________________________________________________________________________________________________________________________________________________________________________

 

Оценка:_______________________                                             Преподаватель_______________

 

 

 

Практическая работа №4

Цель работ — закрепление и развитие знания, способов, средств и техники дефектации деталей, приобретение практических навыков определения дефектов и их сочетаний, использования средств контроля и руководства по капитальному ремонту автомобилей, уяснение характера работ, выполняемых дефектовщиком.

Теоретическая часть

После очистки от загрязнений и мойки детали подвергают дефектации с целью обнаружения в них дефектов и сортировки на годные для дальнейшего использования, требующие ремонта и не­годные. Разбраковку ведут в соответствии с техническими условия­ми на контроль и сортировку деталей, выполненными в виде карт.

В карту вносят следующие данные: общие сведения о детали; перечень возможных дефектов; способы обнаружения дефектов; указа­ния о допустимости дефектов и рекомендуемые способы их устра­нения.

К деталям, годным для дальнейшего использования, относят те, которые имеют допустимые размеры и шероховатость поверхности согласно чертежу и не имеют наружных и внутренних дефектов. Та­кие детали отправляют на склад запасных частей или в комплекто­вочное отделение.

Детали, износ которых больше допустимого, но годные к даль­нейшей эксплуатации, направляют на склад накопления деталей, а далее — в соответствующие ремонтные цехи для восстановления.

Негодные детали отправляют на металлолом, а вместо них со склада выписываются запасные детали.

В соответствии с техническими условиями процесс дефектации проводится в следующем порядке. Сначала внешним осмотром об­наруживают повреждения: видимые трещины, пробоины, задиры, риски, коррозию и т. п.; оценивают состояние трущихся поверхностей и соответствие их нормальному процессу эксплуатации. Далее дета­ли, прошедшие внешний осмотр, проверяются на соответствие их геометрических параметров и физико-механических свойств с задан­ными по чертежу. Из числа геометрических параметров устанавли­ваются действительные размеры деталей, погрешности формы (оваль­ность, конусность, прогиб), погрешности расположения (биение, несоосность, непараллельность и др.).

В процессе эксплуатации автомобиля происходят изменения фи­зико-механических свойств деталей. Контроль за изменением свойств осуществляется по величине твердости, измерение которой произво­дится твердомерами. Твердость детали должна быть не ниже ука­занной на чертеже или в технических условиях.

Потерю жесткости рессор и пружин оценивают по величине про­гиба при определенней нагрузке на специальных приспособлениях.

Окончательное заключение о годности деталей делается после контроля дефектов.

Под дефектом понимается недопустимая несплошность металла детали.

К числу дефектов, встречающихся в деталях автомобиля, относятся трещины различного происхождения (сварочные, усталостные, закалочные, шлифовочные, водородные и др.), коррозионные изъяз­вления, поры, неметаллические включения и др. По расположению дефекты бывают поверхностными и внутренними. Известно боль­шое разнообразие методов установления дефектов. Из них в авторе­монтном производстве наибольшее применение нашли такие мето­ды неразрушающего контроля, как магнитный, капиллярный и уль­тразвуковой.

Сущность магнитного метода контроля состоит в том, что при намагничивании контролируемой детали дефекты создают участок с неодинаковой магнитной проницаемостью, вызывающей изменение величины и направления магнитного потока. Магнитные силовые линии проходят через деталь и огибают дефект, как препятствие с малой магнитной проницаемостью.

Для выявления дефектных мест деталь сначала намагничивают, а затем наносят равномерный слой сухого магнитного порошка. Маг­нитный порошок под действием магнитного поля будет притянут краями дефекта и четко обрисует его границы.

Метод магнитной дефектоскопииобеспечивает высокую произ­водительность и дает возможность обнаружить трещины шириной до 0,001 мм на глубине до 6 мм. Применяется метод для контроля деталей, изготовленных из ферромагнитных материалов (сталь, чу­гун).

Для контроля деталей из цветных металлов и сплавов, пластмас­сы и других материалов применяют капиллярный метод дефекто­скопии.

Сущность капиллярной дефектоскопии заключается в том, что на контролируемую поверхность наносят слой специального цвето-контрастного жидкого индикаторного вещества.

Одним из способов капиллярного метода контроля является «ке­росиновая проба». На поверхность детали наносят слой керосина и выдерживают в течение 15—20 мин. Затем ветошью тщательно про­тирают поверхность насухо. Далее на поверхность наносят прояви­тель, представляющий собой водно-меловой раствор. При высыха­нии мел вытягивает керосин и на поверхности появляется керосиновое пятно. Способ весьма прост, но образующееся пятно не дает пол­ных сведений о форме и размерах дефекта.

Разновидностью капиллярного метода служит люминесцентный способ контроля дефектов, основанный на свойстве некоторых ве­ществ светиться при облучении их ультрафиолетовыми лучами.

Очищенные и обезжиренные детали помещают в ванну с флю­оресцирующей жидкостью. Жидкость проникает в дефекты и там задерживается. Остатки жидкости смывают холодной водой, де­таль сушат сжатым воздухом и припудривают порошком селикагеля. При освещении детали ультрафиолетовым излучением по­рошок селикагеля, пропитанный флюоресцирующей жидкостью, будет ярко светиться желто-зеленым светом. Трещины будут вид­ны в виде широких полос, поры — в виде пятен.

Люминесцентные дефектоскопы позволяют выявить трещины шириной 0,01 мм.

Ультразвуковой метод дефектоскопииоснован на свойстве уль­тразвука проходить через металлические изделия и отражаться от границы раздела двух сред, обладающих разными акустическими свойствами.

Метод ультразвуковой дефектоскопии позволяет установить лю­бые дефекты (трещины, поры, неметаллические включения и т. д.), залегающие на глубине 1—2500 мм.

Для обнаружения скрытых дефектов в полых деталях широко применяется метод гидравлических и пневматических испытаний.

Проводятся такие испытания на специальных стендах. Так, де­фекты в блоке и головке блока цилиндров устанавливают гидравли­ческим испытанием на стенде, обеспечивающим герметизацию всех отверстий. Блок заполняется горячей водой, и в нем создается давле­ние 0,3—0,4 МПа. Наличие дефектов определяют по подтеканию воды.

Пневматические испытания позволяют определить герметичность радиаторов, топливного бака и др. путем закачки в них сжатого воз­духа под давлением, согласно техническим условиям. Далее агрега­ты помещают в ванну с водой и по выделению пузырьков определяют место нахождения дефекта.

 

Содержание работы: подготовка исходных данных для дефектации деталей;

определение технического состояния деталей; сортировка деталей по результатам контроля;

назначение способа ремонта и содержания операций по подефектной технологии;

оформление отчета о результатах работы.

 

Практическая работа №5

Цель работы: ознакомиться с технологией нанесения гальванических покрытий, научиться рассчитывать режимы и нормы времени.

Теоретическая часть

Сущность процесса нанесения гальванических покрытий. В авторемонтном производстве при восстановлении деталей нашли широкое применение гальванические и химические процессы. Они применяются для компенсации износа рабочих поверхностей деталей, а также при нанесении на детали противокоррозионных и защитно-декоративных покрытий.
Гальванические и химические способы обработки предназначены для восстановления изношенных поверхностей деталей (хромирование, железнение, никелирование); для защиты деталей от коррозии (цинкование, бронзирование, оксидирование):для защитно-декоративных целей (хромирование, никелирование, цинкование, оксидировацие); ддя придания поверхностям дсталей специальных свойств, обеспечиваюших хорошую прирабаты ваемость (меднение, лужение, свинцование, фосфатирование), защиту от науглероживания при цементации (меднение), повышение электрической проводимости (меднение, серебрение), повышение отражательной способности (хромирование, никелирование), цодслоя под другое покрытие (медь, никель) или грунта под окраску

В основе восстановления деталей гальваническими покрытиями лежит процесс электролиза, т.е. прохождения постоянного тока через электролит, связанное с передвижением электрически заряженных частиц – ионов.

Электролиз — электрохимический процесс (электролиз металлов), протекающий между анодом и катодом (деталью) в электролите (водном растворе соли, кислоты или щелочи) и сопровождающийся выделением на катоде металла (рисунки 5.1и 5.2).

https://helpiks.org/helpiksorg/baza1/1351808689984.files/image233.jpg

Рисунок 5.1. Принципиальная схема процесса электролитического наращивания.

https://helpiks.org/helpiksorg/baza1/1351808689984.files/image234.jpg

 

Рисунок 5.2. Схема электрохимического осаждения металла:

1—ванна; 2 — Анодная штанга; 3 — Крюк (подвеска) для завешивания анода;

— катодная штанга; .5 —крюк подвеска для завешивания детали (катода);

6 — ионы металла (катионы); — покрытие; 8 — Анод; —- деталь (катод).

 

При прохождении постоянного тока через электролит на аноде 3 происходит растворение металла (переход его в электролит) и выделение кислорода, а на катоде 9 (деталь) — отложение металла и выделение водорода.

Из гальванических процессов наиболее широко применяются хроми­рование и железнение, а также никелирование, цинкование и меднение. Применяются также химические процессы; химическое никелирование, оксидирование и фосфатирование

Электролитические покрытия предпочтительнее наплавки, так как:

процессы гальванического осаждения металла не вызывают структурных изменений в деталях,

· позволяют устранять незначительные износы,

· легче поддаются механизации и автоматизации,

· можно получать равномерные по толщине покрытия с широким диапазоном твердости (от 1000 до 12000 МПа), что позволяет восстанавливать большую номенклатуру деталей, значительно от-личающихся конструктивно-технологическими характеристиками и условиями эксплуатации,

· одновременно можно восстанавливать значительное количество деталей,

· применяемые электролиты можно использовать многократно,

· технологический процесс легко поддается механизации и автоматизации.

Недостатки электролитического наращивания:

· сравнительно низкая производительность процесса,

· большой цикл подготовительных операций,

· значительное выделение вредных веществ (хлор, кислотные испарения и т, п.).

Наибольшее распространение получили осталивание (железнение), хромирование, никелирование, меднение, нанесение электролитических сплавов.

Железнение:

- высокая производительность наращивания (скорость осаждения металла 0,2…0,5 мм/ч),

- толстые осадки (до 2 мм и более),

- высокие физико-механические свойства,

- недорогие и недефицитные материалы,

- себестоимость восстановления – 30…50% от стоимости новой детали при одинаковой износостойкости.

Хромирование:

- высокая твердость, жаростойкость, износостойкость покрытий, низкий коэффициент трения;

- осадки хрома обладают повышенной хрупкостью и плохой прирабаты-ваемостью;

- низкий к. п.д.;

- процесс чувствителен к изменениям температуры электролита и плотности тока,

- электролит нестабилен по составу и требует корректировки в процессе электролиза.

Увеличивает износо - и коррозионную стойкость деталей, улучшает внешний вид.

Никелирование:

- высокая твердость, жаростойкость, износостойкость покрытий, низкий коэффициент трения;

- низкая производительность,

- дефицитные материалы,

- электролит нестабилен по составу и требует корректировки в процессе электролиза,

- высокая себестоимость восстановления.

Применятся для защитно-декоративных целей, как подстилающий слой при декоративном хромировании, а иногда для повышения изностойкости и восстановления деталей – поршневых колец, пальцев, плунжеров и т. п.

Безванные способы применяют для восстановления крупногабаритных деталей: коленчатых валов, отверстий корпусных деталей, цилиндров двигателей и др. К безванному осаждению металла относятся три способа: струйный, проточный, натиранием.

 

https://helpiks.org/helpiksorg/baza1/1351808689984.files/image236.jpg

Рисунок 5.3. Струйное хромирование.

В проточном электролите восстанавливают внутренние поверхности цилиндров двигателей (рисунок 5.4) и гидроцилиндров, которые образуют местную ванну для циркулирования электролита. Он нагнетается в полость детали насосом. Расстояние между зеркалом цилиндра (катодом) и стержнем (анодом) должно быть не менее 5...10 мм. При струйном и проточном способах восстановления деталей применяют плотность тока 180...220 А/дм2.

https://helpiks.org/helpiksorg/baza1/1351808689984.files/image238.jpg

Рис.5.4.Установка для безванного хромирования в проточном электролите.

Принципиальная схема наращивания металла электролитическим натиранием приведена на рис. 5.5

Восстанавливаемую деталь закрепляют в патроне станка и подключают к катоду источника постоянного тока 9Электролит из сосуда 1 с помощью капельницы с краном подается к войлочному тампону 4Закрепленному в тампонодержателе (анод).

В межэлектродном пространстве между деталью и стержнем (это собственно местная ванна) протекает электрохимическая реакция, в резуль-тате которой на детали наращивается металл.

Этим способом можно восстанавливать и внутренние поверхности (например, отверстия корпусных деталей), при этом применяют подвижный (вращающийся) анод.

Относительное перемещение анода (катода) препятствует росту зерен, структура осадка получается мелкозернистая и ненапряженная, а поверхность очень гладкая, что в отдельных случаях позволяет исключить механическую обработку покрытия.

Рабочая плотность тока при электронатирании — 150...180 А/дм2. Производительность этого способа в 3...4 раза выше, чем ванных.

https://helpiks.org/helpiksorg/baza1/1351808689984.files/image240.jpg

 

Рисунок 5.5.. Электролитическое осаждение металла натиранием: 1 — емкость для сбора электролита; 2 — деталь (катод); 3 — графитовый стержень (анод); 4 — тампон; 5 — пластмассовый колпачок; 6 — алюминиевый корпус; 7 — кран; 8 — резервуар с электролитом; 9 — источник тока; 10 — клемма; 11 — пластмассовая гайка; 12 — штеккер для подвода тока к аноду.

Хромирование. Технологический процесс хромирования включает следующие операции:

1) механическая обработка поверхности;

2) промывка органическими растворителями;

3) изоляция участков, не подлежащих покрытию;

4) монтаж на подвесные приспособления;

5) обезжиривание;

6) промывка в горячей и холодной воде;

7) декапирование;

8) электроосаждение покрытия.

При хромировании в качестве электролита используют водный раствор хромового ангидрида (CrО3) и серной кислоты (Н2SO4). При хромировании используют нерастворимые аноды, изготовленные из сплава свинца с сурьмой.

Изменяя режим электролиза (плотность тока, температуру электролита) можно получить различные хромовые покрытия:

- матовые (серые) отличаются высокой твердостью, хрупкостью и пониженной износостойкостью (некачественные);

- блестящие отличаются высокой твердостью, хрупкостью и износостойкостью. Имеется сетка пересекающихся трещин. Применяется для деталей, работающих на износ;

- молочные отличатся высокой износостойкостью, большой вязкостью и пониженной твердостью. Сетка трещин отсутствует. Для деталей воспринимающих большие удельные давления и знакопеременные нагрузки.

Железнение. Технологический процесс включает следующие операции:

1) механическая обработка поверхности;

2) защита поверхностей, не подлежащих покрытию;

3) обезжиривание в бензине или щелочном растворе;

4) промывка в горячей и холодной воде;

5) монтаж на подвесные приспособления;

6) анодная обработка (травление);

7) промывка в холодной воде;

8) железнение;

9) промывка в горячей воде;

10) нейтрализация;

11) механическая обработка.

В качестве электролита применяют водный раствор хлористого железа (FeCl2×4H2O) и соляную кислоту (НСl) и некоторые другие компоненты. Железнение производят с применением растворимых анодов из малоуглеродистой стали 08 или 10, помещенных в чехлы из стеклоткани для сбора шлама.

Технологические приемы получения износостойких покрытий.Применение ассиметричного периодического тока позволяет путем изменения параметров обратного импульса управлять свойствами покрытий (износостойкость, микротвердость, усталостная прочность), а также увеличить производительность процесса.

Ассиметричный периодический ток получают применением схемы, изображенной на рисунке 5.6.

https://helpiks.org/helpiksorg/baza1/1351808689984.files/image242.gif

Рисунок 5.6 – Принципиальная схема установки

для получения ассиметричного периодического тока

 

Практическая работа №6

Цель занятия: закрепить знания по устройству и обслуживанию системы охлаждения дизельного двигателя; получить практические навыки.

Оборудование:

– монтажные двигатели, установленные на стендах и укомплектованные составными частями изучаемых систем (с двигателей снято всё, что мешает доступу к системам, кроме генераторов, имеющих общий привод с вентиляторами; головки цилиндров, крышки распределительных шестерён закреплены лишь в двух-трёх точках; агрегаты изучаемых систем комплектные, включая детали уплотнений);

– разрезы двигателей и отдельных частей;

– оборудование и инструмент для разборочно-сборочных работ: верстаки с тисками, наборы инструмента, универсальные и специальные съёмники, выколотки с латунными наконечниками, деревянные наставки и накладки цилиндрические пустотелые оправки, молотки с медными бойками;

– контрольно-измерительные приборы (масштабные металлические линейки, пластинчатые щупы, индикатор, ртутный термометр, приспособление КИ-8920 для контроля натяжения ремней;

– сосуд с водой и нагреватель для проверки термостатов.



СОДЕРЖАНИЕ И ПОСЛЕДОВАТЕЛЬНОСТЬ ВЫПОЛНЕНИЯ РАБОТЫ:



1. Рассмотрите расположение и взаимосвязь составных частей системы охлаждения: водяных рубашек дизеля и пускового двигателя, водяного насоса, вентилятора и радиатора, места присоединения предпускового обогревателя, блока отопления и охлаждения кабины. Выясните, где наливается и сливается охлаждающая жидкость.

2. Изучите радиатор: крепление его на раме, устройство сердцевины, боковин и их соединение, устройство механизма управления шторкой радиатора.

3. Снимите водяной насос в сборе с вентилятором, частично разберите его, изучите устройство деталей. Заложите крыльчатку насоса в полость корпуса и по конфигурации полости определите направление вращения крыльчатки; проверьте правильность решения, исходя из направления вращения шкива вентилятора. Выясните, где в полости насоса расположены зоны низкого и высокого давления. Приложите корпус насоса к месту крепления его на двигателе и уясните связь нагнетательной полости насоса с водораспределительным каналом блок-картера. Соберите насос и вентилятор.

4. Снимите головку цилиндров, рассмотрите водяные рубашки блок-картера дизеля, головки цилиндров и пускового двигателя и соединение их между собой.

5. Снимите термостат, изучите его устройство и работу. Помещая термостат поочерёдно в горячую и холодную воду, проследите его действие. Постепенно нагревая воду, определите температуры, при которых основной клапан начнёт открываться и будет, открыт полностью.

6. Используя плакат и детали, изучите, как циркулирует вода в системе, при работе пускового двигателя, при работе прогретого и непрогретого дизеля.

7. Установите на место снятые детали и сборочные единицы. Выясните нормальное значение температуры охлаждающей жидкости при работе двигателя, рассмотрите места установки датчика и указателя температуры.

8. Снимите и вновь наденьте ремень привода вентилятора и отрегулируйте натяжение, используя приспособление КИ-8920 (рис. 51, учебника). Опробуйте натяжение ремня рукой, ослабьте его и вновь отрегулируйте, но уже без приспособления. Проверьте натяжение приспособлением и при необходимости уточните регулировку. Проверьте натяжение рукой и последующую регулировку повторите по два-три раза.

9. Выясните, какие признаки характеризуют работоспособное состояние системы охлаждения, изучите правила технического обслуживания, обеспечивающие её работоспособность.

СОДЕРЖАНИЕ ОТЧЕТА:

1. Тема.

2. Цель.

3. Описать устройство системы охлаждения дизельного ДВС.

4. Ответы на контрольные вопросы. (Устно или письменно, по указанию преподавателя в зависимости от качества усвоения материала во время занятия).

КОНТРОЛЬНЫЕ ВОПРОСЫ:

1. Какую температуру должна иметь охлаждающая жидкость при работе дизеля?

2. Как обеспечивается циркуляция охлаждающей жидкости при работе дизеля и пускового двигателя?

3. Как смазываются подшипники водяного насоса?

4. Как устроен термостат?



Практическая работа №7

Цель: Сформировать практические навыки по разборке и сборке масляного насоса и фильтров. Ознакомиться с общими схемами системы смазки дизельных двигателей. Изучить конструкции изучаемых узлов.

 

Теоретическая часть

Поддоны картеров являются резервуарами для масел, заливаемых в двигатель. Для слива масел в нижнюю часть поддона ввернута пробка. Поддоны изготовляют штамповкой из стали (двигатель ЗИЛ-130 и др.) или литьем из алюминиевого сплава (двигатель Д-240 и др.). Между поддоном и картером ставят уплотнительную прокладку. Уровень масла в поддоне проверяют масломерной линейкой.

При помощи масляных насосов создают давление нагнетаемого масла в систему смазки двигателя. Работа насоса основана на том, что шестерни 1 и 2 (рис, 1) вращаются, засасывая масло через входной канал 6, которое заполняет впадины между зубьями шестерен и переносится ими в нагнетательный канал 3. При увеличении давления масла в канале 3 открывается редукционный клапан 4 и масло сливается в поддон картера.

hello_html_2aae55d.png

Рис. 36. Схема действия масляного насоса:

1— ведущая шестерня; 2 —ведомая шестерня; 3 — нагнетательный канал; 4 — редукционный клапан; 5 — корпус насоса; 6 — входной канал; 7 — фильтрующая сетка.

 

Производительность основной секции насоса СМД-62 составляет 70 л/мин, радиаторной— 18,5 л/мин. Насосы испытывают на стендах УСИН-2 или УСИН-3. Для подачи масла в систему" смазки перед запуском на двигателе СМД-62 предусмотрен насос предпусковой подкачки масла.

Масляные радиаторы применяют для поддержания температуры масла при работе двигателя в необходимых пределах (70... ...80°С). Масляные радиаторы расположены впереди радиатора системы охлаждения, на пути воздушного потока. Включение масляных радиаторов осуществляется специальным краником или автоматически клапаном-термостатом.

Работа фильтров заключается в следующем. В роторе 7, свободно вращающемся под действием реакций струй масла, которые выходят под давлением через два жиклера 3, тяжелые частицы, загрязняющие масло, отбрасываются на стенки колпака 8. Далее масло проходит через сетку 10, жиклеры 3 и стекает в поддон картера двигателя.

 

Задание.

1. Пользуясь плакатами, повторите устройство системы смазки.

2. Разобрать и собрать масляный насос и фильтр.

3. Ответить на контрольные вопросы.

 

Порядок выполнения работы.

1. Пользуясь схемами и плакатами, непосредственно на двигателях ознакомьтесь с размещением приборов и агрегатов систем смазки.

2. Внимательно прочитайте название всех приборов и агрегатов, указанные на схемах систем смазки, проследите пути масла из поддона к трущимся поверхностям сопрягаемых деталей.

Пользуясь рисунком , рассмотрите детали масляных насосов. На двигателях СМД-62 и ЗИЛ-130 установлены двухсекционные насосы шестеренчатого типа. Отверните болты и снимите всасывающую трубку с маслозаборником 12. Отверните болты и снимите корпус 9 радиаторной секции, осмотрите шестерни радиаторной секции (ведущую 10 и ведомую 13). Если снять шестерни радиаторной секции и проставку 8, можно увидеть ведущую 7 и ведомую 14 шестерни основной секции насоса. Разберите предохранительный клапан 5 радиаторной секции (он регулируется на давление 0,25 МПа) и редукционный клапан 4 нагнетательной секции (он регулируется на давление 0,95 МПа). Обратите внимание на отверстия в корпусе 6 насоса. Проведите сборку насоса,

 

hello_html_10555273.png

Рис. 2. Масляный насос:

1 — шестерня привода; 2 — валик; 3 —шпонка; 4— редукционный клапан нагнетательной секции; 5 — предохранительный клапан радиаторной секции; 6 — корпус насоса; 7—ведущая шестерня; 8 — проставка; 9 — корпус радиаторной секции; 10 — ведущая шестерня радиаторной секции; 11—трубка; 12 — маслозаборник; 13 — ведомая шестерня радиаторной секции; 14—ведомая шестерня; 15 —втулка; 16 — валик ведомой шестерни.

 

Рассмотрите рисунок 3, изучите детали фильтров. Обратите внимание на то, что конструктивно фильтры дизеля СМД-62 и карбюраторного двигателя ЗМЗ-53 подобны и представляют собой полнопоточные центрифуги.

hello_html_1f09bb09.png

Рис. 3. Фильтр центробежной очистки масла;

а—автомобильный; б — тракторный; 1—ось ротора; 2 — подшипник; 3 — жиклер; 4 — поддон; 5, 11 — прокладки; 6 — уплотнитель; 7 — ротор; 8 — колпак; 9 — кожух; 10 — сетка; 12, 13, 14 — гайки; 15 — перепускной клапан.

Разберите фильтр. Эффективность очистки масла зависит от частоты вращения ротора, поэтому разбирать фильтр следует осторожно.

Разберите центрифугу в такой последовательности. Отверните гайку-барашек 14 и снимите кожух 9. Отверните гайку 12, удерживая колпак 8 от вращения, и осторожно снимите колпак. Рассмотрите отверстия в корпусе и оси фильтра, а также состояние отверстий жиклеров 3. Соберите центрифугу в обратном порядке.

 

Контрольные вопросы

1. Каково назначение системы смазки двигателей внутреннего сгорания?

2.Назовите приборы и агрегаты системы смазки карбюраторного и дизельного двигателей.

3. Покажите на общих схемах системы смазки двигателей пути масла к трущимся поверхностям сопрягаемых деталей. Какие детали смазываются под давлением и какие разбрызгиванием?

4. Как устроен масляный насос двигателя СМД-62 (ЗИЛ-130)?

5. Каково назначение масляных радиаторов?

6. Каково назначение масляных фильтров?

7. Как устроена и работает полнопоточная центрифуга?

 




Практическая работа №8

 

Цель работы: научиться правильно и в полном объеме проводить разборку двигателя легкового автомобиля, закрепить основные особенности разборки двигателя.

 

Оборудование: двигатель ВАЗ-21011, комплект съемников, инструмента и приспособлений, стенд для разбрки-сборки двигателя.

 

Порядок выполнения работы


1. Вымойте двигатель на моечной установке, установите его на стенд для разборки и слейте из картера масло;
2. Снимите карбюратор, отсоединив от него шланги и тягу привода дроссельной заслонки;
3. Снимите топливный насос, распределитель зажигания, ключом 67.7812.9514 выверните свечи и датчик указателя температуры охлаждающей жидкости;
4. Снимите ремень привода генератора и насоса охлаждающей жидкости, снимите генератор и кронштейн генератора;
5. Снимите насос охлаждающей жидкости, отсоединив от насоса и выпускного коллектора трубопровод подвода жидкости из отопителя;
6. Снимите с головки цилиндров выпускной патрубок охлаждающей жидкости и трубопровод отвода жидкости к отопителю;
7. Приспособлением А.60312 отверните и снимите масляный фильтр с прокладкой; (см. рис. 1)

hello_html_m51ecbaf1.jpg

 

Рис. 1 Снятие масляного фильтра


8. Выверните датчик контрольной лампы давления масла, снимите крышку сапуна вентиляции картера, картер и масляный насос. Снимите фиксатор сливной трубки маслоотделителя и выньте маслоотделитель вентиляции картера;
9. Снимите шкив коленчатого вала, закрепив маховик фиксатором А.60330/R и отвернув ключом А.50121 храповик; (см. рис 2)

 

hello_html_38dd3a5a.jpg

Рис. 2 Снятие храповика

 

 



 

 

10. Снимите крышку головки цилиндров и крышку цепного привода распределительного вала. Отверните болты крепления звездочек распределительного вала и вала привода масляного насоса;
11. Ослабьте колпачковую гайку 6 натяжителя цепи, отверните гайки 4 крепления его к головке цилиндров, снимите натяжитель и, отвернув болт 2, снимите башмак 3 натяжителя цепи; (см. рис. 3)

hello_html_m11eb1d31.jpg



1 – цепь привода распределительного вала; 2 – болт крепления башмака; 3 – башмак натяжителя; 4 – гайка крепления натяжителя; 5 – корпус натяжителя; 6 – колпачковая гайка натяжителя; 7 – болты крепления успокоителя; 8 – болт крепления звездочки валика привода масляного насоса


Рис. 3 Снятие натяжителя цепи
12. Отверните ограничительный палец цепи, снимите звездочки привода масляного насоса и распределительного вала и выньте цепь;

13. Ослабьте гайки шпилек 4. Отвернув гайки крепления, снимите корпус подшипников шпилек 4 и, удалив упорный фланец 1, осторожно, чтобы не повредить поверхность опор корпуса 3 подшипников, выньте распределительный вал 2; (см. рис 4)

hello_html_81b7e49.jpg


Рис. 4 Снятие распределительного вала


14. Отверните болты крепления головки цилиндров и снимите ее вместе с выпускным коллектором и впускным трубопроводом;
15. Снимите упорный фланец 1 валика привода масляного насоса и выньте валик 3 из блока цилиндров (2 – болт крепления фланца, 4 – ключ);(см. рис 5)

hello_html_3e47d12.jpg

 

Рис. 5 Снятие упорного фланца валика привода масляного насоса

16. Универсальным съемником А.40005/1/7 из комплекта А.40005 снимите звездочку с коленчатого вала; (см рис. 6)



hello_html_684e4e56.jpg

 

 

Рис. 6 Снятие звездочки коленчатого вала



17. Отверните гайки шатунных болтов, снимите крышки шатунов и осторожно выньте через цилиндры поршни с шатунами;
ВНИМАНИЕ: При разборке двигателя пометьте поршень, шатун, вкладыши коренных и шатунных подшипников, чтобы при сборке установить их на прежнее место.
18. Установите фиксатор 5 (см. рис. Снятие маховика ), отверните болты 3, снимите шайбу 4 и маховик с коленчатого вала, снимите переднюю крышку картера сцепления; (см. рис. 7)

 

hello_html_4d43593c.jpg

 

1 – ключ; 2 – маховик; 3 – болт крепления маховика; 4 – шайба; 5 – фиксатор А.60330/R для удержания маховика от проворачивания; 6 – передняя крышка картера сцепления

Рис. 7 Снятие маховика двигателя


19. Выталкивателем А.40006 выньте подшипник первичного вала коробки передач из гнезда в коленчатом валу; (см. рис. 8)

hello_html_16112bc2.jpg


Рис. 8 Снятие подшипника первичного вала коробки передач


20. Снимите держатель сальника коленчатого вала.
21. Отверните болты крышек коренных подшипников. Снимите их вместе с нижними вкладышами, снимите коленчатый вал, верхние вкладыши и упорные полукольца на задней опоре.

Контрольные вопросы

1. Перечислить основные детали двигателя.

2. Перечислить порядок разборки двигателя.

3. Как правильно маркируются крышки коренных подшипников коленчатого вала?

4. Какие особенности разборки шатунов двигателя?



Практическая работа №9


Цель занятия: Обучение практическим приемам сборки двигателя.

 

Инструкция по сборке двигателя автомобиля

1. Перед началом сборки необходимо осмотреть все детали и тщательно их промыть.

2. При укладке коленчатого вала двигателя следует проследить, чтобы посадка подшипников в гнезда блока картера была произведена без ударов. После установки коленчатый вал обязательно должен свободно вращаться, без заклинивания шариков. Маслоотражательное кольцо не должно при вращении касаться уплотняющей крышки. Все болты должны быть равномерно затянуты и законтрены. При установке маслоподающей шайбы на вал надо проследить, чтобы телескопическая труба вошла в отверстие маслоподающей шайбы.

3. При укладке коленчатого вала двигателя следует тщательно проверить соответствие вкладышей своим местам. Гайки подшипников надо затягивать равномерно, чтобы при всех затянутых подшипниках вал свободно вращался. Убедившись в этом, следует зашплинтовать гайки. Прокладки под кожух маховика и поддон должны быть без дефектов, иначе в работе двигателя всегда будет подтекать машинное масло.

4. Распределительный вал двигателя после установки фиксируется от продольного перемещения гайкой, которая контрится отгибной шайбой. Вал должен вращаться легко и свободно, без заклинивания шариков.

5. После установки распределительного вала, шестерен распределения и привода водяного насоса надо проверить щупом зазоры между зубьями сцепляющихся шестерен. Зазоры не должны превышать допустимые нормы. Паразитная шестерня должна быть сцеплена с шестернями коленчатого и распределительного валов по меткам. После установки шестерен надлежит провернуть вал и проверить свободное, без заеданий, вращение шестерен.

6. Маховик двигателя должен быть посажен па конус, все забоины и заусенцы должны быть предварительно зачищены. Маховик должен быть плотно, без качаний, посажен на шпонку. Затяжка гайки крепления маховика должна быть произведена полным усилием. После затяжки гайка должна быть законтрена. После сборки обязательно нужно проконтролировать индикатором биение данного маховика, которое не может превышать допустимой величины.

7. Перед установкой маховика на двигатель, следует убедиться в отсутствии забоин и заусенцев на фланце коленчатого вала и ступице маховика. После закрепления маховика на фланце вала надо проверить биение его по индикатору и отсутствие задевания за головки болтов крепления кожуха маховика. Убедившись в правильности установки, зашплинтовать болты крепящие маховик.

8. Поршень вместе с шатуном, со снятой крышкой нижней из головок шатуна, обязан устанавливаться во втулке цилиндра сверху. Сжатие поршневых колец для установки поршня в цилиндр должно производиться специальным приспособлением. Необходимо проследить за тем, чтобы при постановке не поцарапать зеркало цилиндра. Шатунную шейку на коленчатом валу при сборке следует смазать чистым маслом. Крышка на нижней головке данного шатуна устанавливается через нижние люки блока картера. Шатунные болты после затяжки необходимо отдать обратно на одну грань для разгрузки от скручивания и законтрить проволокой. Диаметральный зазор меж шейкой вала и вкладышем в эксплуатационных условиях может быть проверен путем обжима свинцовой проволоки и должен находиться в допустимых пределах.

9. Перед установкой уже собранной головки цилиндров необходимо проверить метку ВМТ, которая должна совпадать с визиром. До установки головки должна быть проверена герметичность клапанов, керосином, а головка тщательно осмотрена и промыта. Асбостальная прокладка под головку не должна быть повреждена во избежание пропуска газов или воды. Высота камеры сжатия, определяющая величину степени сжатия, проверяется свинцовой выжимкой. Гайки крепления на головках цилиндров затягиваются равномерно.

https://fsd.videouroki.net/html/2018/12/01/v_5c01f92584dd3/99724579_1.jpeg

Инструкция по сборке двигателя автомобиля

10. В двигателе должны быть проверены щупом зазоры между паразитной шестерней и шестерней коленчатого вала, между паразитной шестерней и шестерней на распределительном валу и между шестерёнкой распределительного вала и следующей за ней шестерней регулятора. Паразитная шестерня должна быть сцеплена с шестерней коленчатого и распределительного валов по меткам. После установки шестерен следует совершить оборот коленчатым валом и убедиться в свободном, без заеданий вращении шестерен.

11. При установке масляных трубопроводов, трубопроводы и масляный насос должны быть заполнены маслом.

12. Перед установкой на двигатель, топливный и масляный фильтры необходимо тщательно промыть. Установка масляного фильтра должна обеспечить герметичность стыка плоскости корпуса фильтра с блоком картера.

16. Установка выхлопного коллектора должна обеспечить герметичность соединения водяной и газовой полостей головки и коллектора. Герметичность достигается постановкой прокладки.

17. После окончания сборки двигателя коленчатый вал следует несколько раз провернуть для проверки отсутствия заеданий при вращении.

 

III. Вводный инструктаж 

1.                    Сообщить тему программы и тему занятия, назвать ее учебное значение.

2.                    Объяснить новый материал:

·                                 Рассказать о значении техники проведения сборки двигателя;

·                                 Разобрать инструкционные карты, обратив внимание на технические требования и условия выполнения;

·                                 Опираясь на знания теоретических дисциплин, разобрать со студентами порядок проведения сборки двигателя;

·                                 Рассмотреть применяемые инструменты, оборудование, приспособления; разобрать специфику проведения сборки двигателя;

·                                 Показать приемы работы; предупредить о возможных ошибках при выполнении работы. Обратить внимание на приемы самоконтроля;

·                                 Разобрать вопросы рациональной организации рабочего места;

·                                 Провести инструктаж по правилам техники безопасности, обратить внимание студентов на опасные зоны, требующие особой собранности при работе;

·                                 Предложить 2 студентам провести рабочие приемы сборки двигателя; убедиться в понимании;

·                                 Сообщить студентам критерии оценок.



IV. Текущий инструктаж 

Самостоятельная работа студентов – целевые обходы рабочих мест студентов:

·                                 Первый обход: проверить содержание рабочих мест, их организацию;

·                                 Второй обход: обратить внимание на правильность выполнения сборки двигателя, указать на допущенные ошибки и разобрать причины, их вызывающие;

·                                 Третий обход: проверить правильность соблюдения последовательности сборки двигателя;

·                                 Четвертый обход: проверить правильность ведения самоконтроля; соблюдение технических условий работы;

·                                 Пятый обход: провести приемку и оценку выполненных работ.



IV. Заключительный инструктаж 

1.                    Подвести итоги занятия.

2.                    Указать на допущенные ошибки и разобрать причины, их вызывающие.

3.                    Сообщить и прокомментировать оценку обучающимся за работу.

4.                    Выдать домашнее задание, объяснив его важность для усовершенствования навыков работы.




Практическая работа №10

Цели работы: изучить устройство, работу сцепления и его привода; приобрести навыки в разборке, сборке и регулировке сцепления, научиться определять его техническое состояние.

 

Оборудование: сцепления и детали привода различных автомобилей; приспособления для разборки и сборки сцеплений; наборы рожковых, накидных и торцевых ключей; тиски.

 

Содержание работы: с помощью учебных плакатов, альбомов и настоящего пособия изучить устройство сцеплений различных автомобилей, научиться их разбирать и собирать.

 

Описание устройства. Однодисковое сухое сцепление с периферийным расположением пружин и механическим приводом выключения (рис. 1). Кожух 3 сцепления стальной штампованный с четырьмя лапами. В каждой лапе имеется по два отверстия для крепления кожуха к маховику 2. Нажимной диск 7 отлит из чугуна. Поверхность, обращенная к маховику, тщательно обработана. На другой стороне имеется четыре проушины для присоединения с помощью пальцев и игольчатых подшипников 17 рычагов, к которым с помощью пальцев 18 присоединены вилки. Стержни вилок проходят через отверстия в кожухе, па вилки навернуты полусферические гайки. Между пальцами присоединения вилок и рычагами установлены игольчатые подшипники 17. Внутренние концы всех четырех рычагов 22 должны находиться в одной плоскости, в противном случае работа сцепления будет нарушена. Для равномерного распределения нажимных пружин 9 по всему нажимному диску 7 имеются установочные пальцы, на которые сначала надевают теплоизолирующие шайбы, а затем пружины. Для предотвращения перекосов пружин кожух сцепления в соответствии с установочными пальцами имеет проштампованные отверстия с внутренними буртиками, которые предотвращают перекос пружин. Для более надежной передачи вращения с кожуха на нажимной диск установлены пружинные пластины 5.

Ведомый диск состоит из тонкого стального диска, к которому с обеих сторон приклепаны фрикционные накладки из прессованной асбестовой крошки, на которых для предотвращения коробления выполнены разрезы.

При резком изменении частоты вращения коленчатого вала возникают крутильные колебания в трансмиссии автомобиля, которые ослабляют крепления, повышают изнашивание отдельных деталей и являются причиной выхода из строя зубчатых колес.

 

 

hello_html_m63223345.jpg

16 3 17 18 19 20 21

 

 

 

 

 

 

 

 

 

 

 

 

 

 

 

 

Рис. 1. Сцепление автомобиля ЗИЛ-431410:

 

1-крышка картера сцепления; — маховик; — кожух сцепления; 4 — заклепка крепления пружинных пластин к кожуху сцепления; 5 — пружинные пластины,

— болт крепления пружинных пластин к нажимному диску; 7 — нажимной диск; — ведомый диск; — нажимная пружина; 10 — болт крепления маховика к фланцу коленчатого вала; 11— передний подшипник ведущего вала коробки передач; 12 — коленчатый вал; 13 — масленка переднего подшипника ведущего вала; 14 — пружина гасителя крутильных колебаний; 15 — балансировочный грузик; 16— картер маховика и сцепления; 17 — игольчатый подшипник;

18- палец крепления рычага к опорной вилке; 19 — опорная вилка рычага включения; 20 — гайка со сферической поверхностью; 21 — упорная пластина; 22- рычаг выключения; 23 — упорный шариковый подшипник муфты выключения; 24 — ведущий вал коробки передач; 25 — вилка выключения сцепления.

 

 

Поэтому на ведомых дисках сцеплений устанавливают гасители крутильных колебаний.

Ступица ведомого диска не связана жестко с диском, имеем внутренние шлицы для соединения со шлицами ведущего вала коробки передач и установлена внутри стального ведомого диска. С одной стороны ведомого диска установлено кольцо гасителя. На кольце гасителя и на ведомом диске с помощью заклепок закреплены фрикционные пластины. По обе стороны фланца ступицы и ведомого диска установлены диски гасителя и маслоотражатели. Маслоотражатели, диски гасителя и фланец ступицы соединены с помощью заклепок. При этом ведомый диск может поворачиваться на некоторый угол относительно ступицы. В дисках гасителя, кольце гасителя и ведомом диске имеются окна, в которые вставлены пружины с опорными пластинами. Пружины находятся в сжатом, но не до конца, состоянии.

При работе двигателя вращение маховика передается через болты на кожух сцепления, а с него через заклепки на пружинные пластины и через болты па нажимной диск.

С нажимного диска вращение вследствие трения передается на фрикционные накладки и на стальной диск, далее через пружины гасителя крутильных колебаний на диски гасителя, затем через заклепки на фланец ступицы, на ступицу и через шлицы на ведущий вал коробки передач.

При резком изменении частоты вращения коленчатого вала пружины гасителя сжимаются и крутильные колебания уменьшаются.

Педаль выключения сцепления с помощью стяжного болта закреплена на конце вала, который поворачивается в кронштейне. Для уменьшения изнашивания вала и кронштейна на валу установлена масленка. На другом конце вала закреплен рычаг, к которому с помощью пальца присоединена тяга к рычагу вала вилки. Изменение длины тяги осуществляется с помощью шаровой гайки при регулировке свободного хода педали. В исходном положении педаль удерживается пружиной. Вилка выключения сцепления опирается на плечики муфты выключения сцепления. И исходное положение муфта возвращается пружиной. На муфте напрессован упорный шариковый подшипник. Между этим подшипником и внутренними концами рычагов сцепления для свободного хода педали оставляют зазор 3...4 мм.

 

Сцепление диафрагменного типа состоит из кожуха 16 (рис. 2) нажимного 8 и ведомого 4 дисков. Отличие этого сцепления от сцепления рычажного типа заключается в устройстве нажимных пружин и нажимного диска, который соединяется с кожухом гибкими соединительными пластинами 19. Нажимная пружина 9 диафрагменного типа по наружному диаметру опирается на края нажимного диска, а по внутреннему — на подшипник 10 муфты 14 выключения сцепления. В кожухе пружина опирается на опорные кольца 5 и 7.

Привод выключения сцепления гидравлический, состоит из главного цилиндра 1, педали и рабочего цилиндра 20.

Главный цилиндр привода выключения сцепления состоит из корпуса 5 (рис. 3), внутри которого имеется фигурный поршень 2.

Для предотвращения утечки жидкости задняя часть поршня уплотнена резиновой манжетой 1. От выхода из цилиндра поршень удерживается стопорным кольцом 19. От попадания пыли и грязи цилиндр защищен резиновым чехлом 17. Одна сторона чехла надета на цилиндр, другая — на толкатель 16, который имеет проушину 14 для соединения с педалью.

Толкатель и проушина имеют резьбовое соединение и удерживаются от самопроизвольного вращения контргайкой 15.Между толкателем и поршнем должен быть зазор 0,3...0,9 мм, который регулируется изменением длины толкателя.

В головке поршня имеются сквозные каналы, прикрытые пластинкой 3.

Клапан поджимается к головке поршня пружиной 6 через резиновую манжету 4. Для выхода рабочей жидкости из главного цилиндра имеется клапан 8.

Запас рабочей жидкости содержится в резервуаре 12 главного цилиндра, закрытом крышкой 10, в которой есть вентиляционные отверстия для поддержания в резервуаре атмосферного давления. Для того чтобы рабочая жидкость не выплескивалась через вентиляционные отверстия, на крышке имеется отражатель 11.

 

 

 

hello_html_dbef2b3.jpg

 

 

 

 

 

 

 

 

 

 

 

 

 

 

 

 

 

 

 

Рис. 2. Сцепление диафрагменного типа и привод выключения сцепления:

 

1 - главный цилиндр; — картер; 3 — маховик; 4 — ведомый диск; 5 и 7 — опорные

кольца; — оттяжная пружина педали; 8 — нажимной диск; 9 — нажимная

диафрагменная пружина; 10 — подшипник муфты выключения сцепления; 11 —

толкатель главного цилиндра; 12 — педаль; 13 — защитные поролоновые кольца;

14 — муфта выключения сцепления; 15 — шаровая опора; 16 — кожух; 17 — вилка

выключения сцепления; 18 — толкатель рабочего цилиндра; 19 — соединительные

пластины; 20 — рабочий цилиндр

 

 

 

hello_html_m24537db5.jpg




Рис: 3. Главный цилиндр привода выключения сцепления:

1 и 4 — манжеты; 2 — поршень; 3 — пластинка; 5 — корпус главного цилиндра; 6-пружина; 7 — упорное кольцо; 8 — клапан; 9 — обойма клапана; 10 — крышка;

11—отражатель; 12 — резервуар главного цилиндра; 13 — штуцер; 14 — проушина; 15 — контргайка; 16 — толкатель рабочего цилиндра; 17 — чехол; I8- упорная шайба; 19 — стопорное кольцо; А — компенсационное отверстие; Б - перепускное отверстие.

 

 

Резервуар крепится к корпусу с помощью штуцера 13. Рабочая жидкость из резервуара в цилиндр поступает через компенсационное А и перепускное Б отверстия.

При выключении сцепления педаль через проушину 14 и толкатель 16 воздействует на поршень 2. Сдвигаясь, поршень резиновой манжетой закрывает компенсационное отверстие и вытесняет жидкость через клапан 8 в рабочий цилиндр.

Рабочий цилиндр привода выключения сцепления состоит из корпуса, внутри которого находится поршень с уплотнительными резиновыми манжетами. Толкатель соприкасается с поршнем. Цилиндр от загрязнения защищен резиновым гофрированным чехлом и защитным кольцом. Чехол удерживается на корпусе пружинным кольцом. Для удаления случайно попавшего в цилиндр воздуха имеется клапан прокачки, закрытый защитным колпачком.

В привод выключения сцепления входят вилка 17 выключения сцепления (см. рис. .2), качающаяся на шаровой опоре 15, и муфта 14 выключения сцепления с упорным шариковым подшипником 10.

Пружина рабочего цилиндра постоянно отжимает поршень, толкатель и наружный конец вилки в положение, при котором упорный шариковый подшипник муфты выключения сцепления прижимается с небольшим усилием к внутренним концам рычагов выключения сцепления. Наружное кольцо упорного подшипника постоянно вращается с рычагами нажимного диска сцепления.

При включенном сцеплении поршень рабочего цилиндра под действием нажимных пружин отжат к днищу цилиндра.

При нажатии на педаль выключения сцепления рабочая жидкость из главного цилиндра поступает в рабочий цилиндр.

Под давлением поршень рабочего цилиндра толкателем перемещает наружный конец вилки 17 назад. Вилка поворачивается на шаровой опоре 15 и, нажимая на плечики муфты выключения сцепления, передвигает муфту вперед, нажимая на внутренние концы рычагов выключения сцепления. Нажимной диск при этом отходит назад, и сцепление выключается.

Свободный ход педали 12...28 мм обеспечивается конструкцией и не регулируется.

На автомобилях Hyundai Santa Fe, Kia Rio, УАЗ-469 и некоторых других необходимо проверять и регулировать свободный ход педали, который должен быть в пределах 6... 13 мм. На автомобилях ГАЗ-З110, Hyundai Accent, Chevrolet Niva, ГАЗель-33021, -2705 и некоторых других установлены беззазорные механизмы сцепления и регулировка свободного хода педали не производится. На автомобилях Lada Priora и Lada Kalina за счет храпового механизма регулировка троса производится автоматически, а у Renaull Logan и BA3-2113, -2114, -2115 для регулировки зазора имеются специальные гайки.

В подшипник и муфту выключения сцепления заложен смазочный материал, который не требует замены в течение всего срока эксплуатации.

На некоторых легковых автомобилях ВАЗ установлен тросовый привод выключения сцепления.

Педаль 5 сцепления (рис. 4) установлена на оси шарнирно в кронштейне 4 педалей сцепления и тормозного механизма. С помощью стопорной скобы 2 к педали присоединен верхний наконечник 1 троса, который находится в оболочке 8. Второй конец троса 15 с помощью нижнего наконечника 16 соединяется с поводком 17 троса. Длина троса регулируется с помощью регулировочной гайки 18 и контргайки 19. Поводок троса фиксатором 20 соединяется с вилкой 21 выключения сцепления. Тросовый привод выключения сцепления применяется на некоторых легковых автомобилях семейства ВАЗ (Lada Priora, Lada Kalina), Renault Logan и др.

 

 

hello_html_m475642ed.jpg

 

 

 


 

 

Рис. 4. Тросовый привод выключения сцепления:

 

I- верхний наконечник троса; — стопорная скоба; — возвратная пружина подали сцепления; 4 — кронштейн педалей сцепления и тормозного механизма; 5 — педаль сцепления; — верхний наконечник оболочки троса; 7 — скоба, 8 - оболочка троса; - задняя крышка коробки передач; 10 — нижний наконечник оболочки троса; 11 — упорная шайба; 12 — резиновая втулка демпфера; 13 — гайка; 14 — защитный колпачок; 15 — трос; 16 — нижний наконечник троса; 17 — поводок троса; 18 — регулировочная гайка; 19 — контргайка; 20 — фиксатор поводка; 21 — вилка выключения сцепления; 22 — уплотнитель оболочки троса.

 

Двухдисковые сцепления установлены на автомобилях марок КамАЗ и МАЗ. На автомобилях КамАЗ устанавливают фрикционное сухое двухдисковое сцепление с автоматической регулировкой положения среднего диска и периферийным расположением нажимных пружин (рис. 5). На маховике 2 закреплен кожух 17

сцепления, к которому с помощью вилок 8 и оттяжных рычагов 9 присоединен нажимной диск 7. Для надежной передачи крутящего момента путем увеличения площади трения установлен средний ведущий диск 6. Между маховиком, ведущим и нажимным дисками установлены ведомые диски 3 и 5, которые имеют фрикционные накладки и гасители крутильных колебаний — демпферы. Средний ведущий диск имеет механизм 4 автоматической установки, а нажимной диск 7 — приливы для присоединения оттяжных рычагов 9. В отверстие для пальцев установлены игольчатые подшипники. Для выключения сцепления имеется муфта 12 с упорным шариковым подшипником 11 и упорным кольцом 14с пружиной 10. Муфта имеет возвратную пружину. Приводится в движение муфта вилкой 13, установленной на валике 15, выключения сцепления. Для установки в вырезы маховика средний ведущий диск имеет четыре шипа.

Нажимной диск кроме четырех шипов имеет проушины для присоединения оттяжных рычагов и бобышки для нажимных пружин. Бобышки расположены группами — по три между проушинами рычагов. В средней бобышке каждой группы имеется резьбовое отверстие для установки стяжных болтов. Стяжные болты устанавливают при монтаже и демонтаже нажимного диска с кожухом в сборе для облегчения сборки и разборки сцепления. После прикрепления кожуха к маховику стяжные болты выворачиваются.

Нa каждом шипе нажимного диска со стороны среднего ведущего диска имеются закаленные токами высокой частоты площадки, предназначенные для упора лапок оттяжных рычагов среднего диска.

Стальной штампованный кожух сцепления устанавливается на маховике на двух трубчатых штифтах и 12 болтах. Между кожухом сцепления и нажимным диском установлено 12 нажимных пружин, посредством которых ведомые и средний ведущий диски сжаты между нажимным диском и маховиком.

Для обеспечения правильной установки нажимных пружин в кожухе имеется 12 выштамповок. Пружины опираются на бобышки нажимного диска через шайбы и прокладки из термоизоляционного материала.

В кожухе имеется четыре отверстия для стержней вилок оттяжных рычагов. Крепятся вилки гайками с конической полкой, обеспечивающей качание вилки в радиальном направлении при выключении сцепления. Гайка опирается на опорную пластину с волнистым профилем и фиксируется на кожухе запорной пластиной.

Опорная и запорная пластины крепятся к кожуху двумя болтами.

 

Нa другом конце вилки на оси установлен оттяжной рычаг нажимного диска. На оси рычага установлена пружина упорного кольца, которая одним усиком упирается в кожух, а другим через петлю постоянно прижимает упорное кольцо к лапкам оттяжных рычагов, обеспечивая тем самым зазор между упорным подшипником и упорным кольцом, который составляет при включении сцепления (3,6±0,4) мм.

 

 

 

hello_html_1a983eaa.jpg

 

 

 

 

 

 

 

 

 

 

 

 

 

 

 

 

 

 

 

 

 

Рис. 5. Сцепление автомобилей марки КамАЗ:

 

1 —ведущий вал: 2 — маховик: 3 и 5 — ведомые диски; 4 —механизм автоматической установки среднего ведущего диска; 6 — средний ведущий диск; 7— нажимной диск; 8 — вилка оттяжного рычага; 9 — оттяжной рычаг; 10— пружина упорного кольца; 11 — упорный шариковый подшипник; 12 — муфта выключения сцепления; 13 — вилка выключения сцепления; 14 —упорное кольцо; 15валик вилки; 16 — нажимная пружина; 17 — кожух; 18 — теплоизоляционная шайба; 19 — болт крепления кожуха; 20 —картер

 

 

На автомобилях марок КамАЗ и ЗИЛ устанавливают гидравлический привод выключения сцепления с пневматическим усилителем, который предназначен для дистанционного управления сцеплением и уменьшения усилия на педаль сцепления и включает в себя педаль сцепления главного цилиндра, пневмогидравлический усилитель, систему трубопроводов, шлангов и пружин.

Привод состоит из переднего и заднего корпусов, поршня выключения сцепления с толкателем, пневматического поршня, следящего поршня, мембраны редуктора и клапана редуктора.

В переднем корпусе имеется цилиндр, в котором установлены поршень и клапан. Седло клапана вмонтировано в мембрану, нагруженную пружиной. Полость клапана редуктора верхнего отверстия и полость над поршнем пневматического поршня нижнего отверстия соединены каналом. Верхнее отверстие со стороны клапана редуктора закрыто крышкой подвода сжатого воздуха. В задней стенке цилиндра имеется резьбовое отверстие для слива конденсата, закрытое пробкой.

В заднем корпусе имеется два отверстия: нижнее выполняет роль цилиндра поршня выключения сцепления. Шток поршня уплотнен манжетой. Поршень имеет возвратную пружину. С наружной стороны поршень выключения сцепления имеет сферическое углубление для установки толкателя. Верхнее отверстие служит для установки корпуса поршня следящего действия. Полость поршня следящего действия и полость поршня выключения сцепления соединены каналом.

В исходном положении (сцепление включено) толкатель под действием пружины прижимается к поршню, который штоком упирается в пяту пневматического поршня. Поршень занимает крайнее правое положение, пружина поршня разжата.

Следящий поршень под действием пружины мембраны находится в крайнем левом положении. Седло мембраны отсоединено от клапана редуктора, полость над пневматическим поршнем через открытый клапан и отверстие в седле мембраны сообщается с окружающей средой посредством отверстия, защищенного от попадания грязи крышкой.

Клапан редуктора пружиной прижат к седлу крышки подвода воздуха и предотвращает попадание сжатого воздуха из системы в полость над поршнем.

При нажатии на педаль сцепления рабочая жидкость под давлением поступает в полость цилиндра поршня выключения сцепления и далее по каналу в заднем корпусе подводится к следящему поршню, который перемещается, сжимая пружину мембраны и перемещая седло. Седло мембраны, перемещаясь, закрывает выпускной клапан редуктора, сжимает пружину клапана и отодвигает впускной клапан от седла крышки подвода воздуха. Сжатый воздух из системы поступает в полость над поршнем. Поршень под давлением перемещается, сжимая пружину и перемещая поршень выключения сцепления. В это время часть сжатого воздуха через отверстие в переднем корпусе подводится в полость мембраны.

На следящий поршень действуют две направленные в разные стороны силы: одна со стороны рабочей жидкости стремится переместить поршень и открыть впускной клапан, другая со стороны пружины и сжатого воздуха стремится вернуть поршень в исходное положение. При увеличении давления в рабочей жидкости увеличивается и усилие, действующее на мембрану, что и обусловливает следящее действие пневмогидроусилителя. Пневматический и следящий поршни, мембрана и пружина подобраны таким образом, что снижается усилие на педаль сцепления до 200 Н.

При выходе из строя пневмосистемы или отсутствии в ней воздуха перемещение поршня выключения сцепления осуществляется только под давлением рабочей жидкости, при этом усилие на педаль достигает 600 Н.

При отпускании педали сцепления давление рабочей жидкости уменьшается, следящий поршень перемещается в левое положение, мембрана под действием пружины и давления сжатого воздуха изгибается, перемещая седло мембраны. Впускной клапан редуктора под действием пружины садится на седло крышки подвода воздуха, прекращая подачу сжатого воздуха. При дальнейшем перемещении седла мембраны выпускной клапан редуктора отрывается от него, полость над поршнем сообщается с окружающей средой. Поршень под действием пружины перемещается в правое положение и сначала под действием нажимных пружин сцепления, а затем под действием пружины занимает исходное положение.

 

Порядок разборки сцепления:

1.                              уложить сцепление на приспособление;

2.                              установить на фасонную крышку сцепления специальный кронштейн и сжать нажимные пружины винтом пресса;

3.                              отвернуть гайки крепления вильчатых опорных кронштейнов рычагов;

4.                              отвернуть винт пресса, снять специальный кронштейн и фасонную крышку сцепления, конические пружины;

5.                              снять нажимные пружины и теплоизоляционные шайбы;

6.                              расшплинтовать рычаг выключения, вынуть ось и отсоединить рычаг от нажимного диска;

7.                              расшплинтовать ось рычага выключения, вынуть ось и ролик из кронштейна.

 

Порядок сборки сцепления:

1.                              вставить ось и ролик в кронштейн рычага выключения, вставить шплинт и зашплинтовать ось рычага выключения сцепления;

2.                              соединить рычаг выключения с нажимным диском, вставить ось, зашплинтовать шплинт оси рычага выключения;

3.                              установить теплоизоляционные шайбы и нажимные пружины;

4.                              установить фасонную крышку сцепления, конические пружины и специальный кронштейн, завернуть винт пресса и сжать нажимные пружины;

5.                              завернуть гайки крепления вильчатых опорных кронштейнов рычагов;

6.                              отвернуть винт пресса, снять специальный кронштейн и механизм сцепления.

 

Порядок разборки главного цилиндра сцепления:

1.                              снять крышку и сетчатый фильтр наполнительного бачка главного цилиндра;

2.                              вывернуть штуцер крепления бачка к корпусу, снять бачок и прокладку штуцера;

3.                              снять с корпуса и сдвинуть к проушине толкателя резиновый защитный чехол;

4.                              вынуть из корпуса главного цилиндра стопорное кольцо упорной шайбы;

5.                              вынуть из корпуса главного цилиндра упорную шайбу и толкатель;

6.                              вынуть из корпуса поршень с уплотнительными манжетами, клапан поршня, возвратную пружину с держателем. Чтобы не повредить уплотнительные манжеты, необходимо подвести сжатый воздух в отверстие трубопровода.

 

Порядок разборки рабочего цилиндра сцепления:

1.                              отсоединить от рабочего цилиндра резиновый защитный чехол и вынуть толкатель вместе с чехлом;

2.                              снять чехол с толкателя;

3.                              вынуть из корпуса рабочего цилиндра стопорное кольцо;

4.                              вынуть поршень с уплотнительной манжетой, используя сжатый воздух, подведенный в цилиндр через отверстие трубопровода для присоединения к цилиндру;

5.                              снять с поршня уплотнительную манжету;

6.                              вынуть из цилиндра пружину;

7.                              вывернуть клапан прокачки и снять с него защитный колпачок;

8.                              перед сборкой все детали тщательно промыть в тормозной жидкости или спирте, продуть сжатым воздухом и осмотреть.

Все резиновые манжеты должны быть мягкими и эластичными без повреждений. На зеркалах цилиндров не должно быть рисок, раковин, задиров и значительного износа.

При установке ведомого диска следует проверить биение плоскости трения, которое должно быть не более 0,7 мм. При большом биении диск необходимо править, используя приспособления.

Собирают ведущий диск сцепления в последовательности, обратной разборке. После сборки нужно проверить и при необходимости отрегулировать рычаги выключения сцепления.

В сцеплениях с мембранной пружиной следует проверить и при необходимости отрегулировать положение концов лепестков пружины и нажимное усилие.

Собранные диски установить на маховик с помощью специальной оправки и по меткам на кожухе сцепления и маховике. Если меток нет, то их необходимо нанести перед разборкой, для этого следует убедиться в том, что нажимные пружины центрируются по отбортовкам кожуха.

После регулировки зачеканить (раскернить) металл хвостовика каждой сферической гайки в прорезь опорной вилки, для того чтобы не нарушать балансировку при работе двигателя.

КОНТРОЛЬНЫЕ ВОПРОСЫ

1.                              Каково назначение трансмиссии автомобиля?

2.                              Перечислите основные механизмы трансмиссии и объясните их назначение.

3.                              Опишите назначение, устройство и работу однодискового сцепления.

4.                              Опишите назначение, устройство и работу двухдискового сцепления.

5.                              Опишите назначение, устройство и работу механического привода выключения сцепления.

 

 


Практическая работа №11

 

ЦЕЛЬ РАБОТЫ: По результатам изучения устройства коробок передач и дополнительной информации, полученным от мастера производственного обучения, дать заключение о назначении, работе, возможных неисправностях и ТО коробок передач.

ЗАДАНИЕ:

1. Изучить устройство коробок передач;

2. Разобрать коробку передач, найти и озвучить основные детали коробки передач;

3. Определить возможные неисправности и рассказать о технологических процессах

восстановления деталей коробок передач;

ОБЩИЕ ПОНЯТИЯ:

https://fsd.kopilkaurokov.ru/up/html/2020/03/12/k_5e69b4edecf17/542467_2.pngКоробка передач


Коробка передач предназначена для изменения по величине и направлению крутящего момента и передачи его от двигателя к ведущим колесам. Также она обеспечивает длительное разобщение двигателя и ведущих колес, причем на неограниченный срок и без усилий со стороны водителя (по сравнению со сцеплением).

https://fsd.kopilkaurokov.ru/up/html/2020/03/12/k_5e69b4edecf17/542467_3.png

Схема работы коробки передач.
1 - первичный вал; 2 - рычаг переключения передач;

3 - механизм переключения передач; 4 - вторичный вал;

5 - сливная пробка; 6 - промежуточный вал;

7 - картер коробки передач

Коробка передач состоит из :

  • картера,
  • первичного, вторичного и промежуточного валов с шестернями,
  • дополнительного вала и шестерни заднего хода
  • синхронизаторов,
  • механизма переключения передач с замковым и блокировочным устройствами
  • рычага переключения.

Картер содержит в себе все основные узлы и детали коробки передач. Он крепится к картеру сцепления, который, в свою очередь, закреплен на двигателе. Так как при работе, шестерни коробки передач испытывают большие нагрузки, то они должны хорошо смазываться. Поэтому картер наполовину своего

объема залит трансмиссионным маслом (в некоторых моделях автомобилей применяется моторное масло).


Валы коробки передач вращаются в подшипниках, установленных в картере, и имеют наборы шестерен с различным числом зубьев.

Синхронизаторы необходимы для плавного, бесшумного и безударного включения передач, путем уравнивания угловых скоростей вращающихся шестерен (наши руки на поручне вагона поезда в примере с работой сцепления).

Механизм переключения передач служит для смены передач в коробке и управляется водителем с помощью рычага из салона автомобиля. При этом замковое устройство не позволяет включаться одновременно двум передачам, а блокировочное устройство удерживает передачи от самопроизвольного выключения.

Как же происходит изменение величины крутящего момента (числа оборотов) на различных передачах? Давайте с этим разберемся на примере. рис. 1 (а,б)

https://fsd.kopilkaurokov.ru/up/html/2020/03/12/k_5e69b4edecf17/542467_4.png

Рис. 1. Передаточное отношение
а) одной пары шестерен


Возьмем две шестерни, не поленимся и сосчитаем число их зубьев. Первая шестеренка имеет 20 зубьев, а вторая 40. Значит при двух оборотах первой шестерни, вторая сделает только один оборот (передаточное число равно 2).

 

https://fsd.kopilkaurokov.ru/up/html/2020/03/12/k_5e69b4edecf17/542467_5.png

Рис. 1. Передаточное отношение
б) двух шестерен


На рисунке 1. б у первой шестерни («А») 20 зубьев, у второй («Б») 40, у третьей («В») снова 20, у четвертой («Г») опять 40.

 


А дальше очень простая арифметика. Первичный вал коробки передач и шестерня «А» вращаются со скоростью, допустим 2000 об/мин. Шестерня «Б» вращается в 2 раза медленнее, то есть она имеет 1000 об/мин, а так как шестерни «Б» и «В» закреплены на одном валу, то и третья шестеренка делает 1000 об/мин. Тогда шестерня «Г» будет вращаться еще в 2 раза медленнее - 500 об/мин.
От двигателя на первичный вал коробки передач приходит - 2000 об/мин, а выходит - 500 об/мин. На промежуточном валу коробки передач в это время - 1000 об/мин.
В данном примере передаточное число первой пары шестерен равно двум, второй пары шестерен тоже - двум. Общее передаточное число этой схемы 2х2=4. То есть в 4 раза уменьшается число оборотов на

вторичном валу коробки перемены передач, по сравнению с первичным. Обратите внимание на то, что если мы выведем из зацепления шестерни «В» и «Г», то вторичный вал коробки вращаться не будет. При этом прекращается передача крутящего момента и на ведущие колеса автомобиля, что и соответствует нейтральной передаче в коробке.
Задняя передача, то есть вращение вторичного вала коробки передач в другую сторону, обеспечивается дополнительным, четвертым валом с шестерней заднего хода. Дополнительный вал необходим для того, чтобы получилось нечетное число пар шестерен, тогда крутящий момент меняет свое направление (рис. 2).

https://fsd.kopilkaurokov.ru/up/html/2020/03/12/k_5e69b4edecf17/542467_6.png

Рис. 2. Схема передачи крутящего момента при включении задней передачи
1 - первичный вал; 2 - шестерня первичного вала; 3 - промежуточный вал; 4 - шестерня и вал передачи заднего хода; 5 - вторичный вал



 

 

Поскольку в коробке передач реального автомобиля имеется большой набор шестерен, то, вводя в зацепление различные их пары, мы имеем возможность менять и общее передаточное отношение коробки.

Первая передача необходима для начала движения автомобиля, для того чтобы двигатель смог сдвинуть с места тяжелое железное «чудовище». Далее, увеличив скорость движения и сделав некоторый запас инерции, вы можете переключиться на вторую передачу, более «слабую», но более «быструю», затем на третью, четвертую и пятую передачи. Все ступеньки переключения передач вверх - с первой по пятую, следует проходить последовательно. Переключение передач в нисходящем порядке можно производить «прыгая через ступеньку» и даже через несколько - две, три и так далее. Обычный режим движения автомобиля – на четвертой или пятой передачах, потому что они самые скоростные и экономичные.

 

Основные неисправности коробки передач.

Подтекание масла может быть из-за повреждения уплотнительных прокладок, сальников и ослабления крепления крышек картера. Для устранения неисправности необходимо поменять прокладки, сальники и подтянуть крепления крышек.

Шум при работе коробки передач может возникнуть из-за несправного синхронизатора, износа подшипников, шестерен и шлицевых соединений.
Для устранения неисправности необходимо заменить вышедшие из строя детали и узлы.

 

Затрудненное включение передач может происходить из-за поломок деталей механизма переключения, износа синхронизаторов или шестерен.
Для устранения неисправности необходимо заменить вышедшие из строя детали и узлы.

Самовыключение передач случается из-за неисправности блокировочного устройства, а также при сильном износе шестерен или синхронизаторов.
Для устранения неисправности необходимо заменить блокировочное устройство, вышедшие из строя

шестерни, синхронизаторы.


Правила пользования автоматической коробкой передач

Переключатель режимов работы автоматической коробки передач называется рычагом селектора и имеет следующие основные положения P, R, N, D. Есть также положения D2 (или L) и D3 (или S). Могут быть и дополнительные режимы, например W (winter - зима). Давайте разберемся с этими буквами, одновременно поглядывая на схему переключения рычага селектора.

 

https://fsd.kopilkaurokov.ru/up/html/2020/03/12/k_5e69b4edecf17/542467_7.png

Рис. 3. Схема положения рычага селектора
автоматической коробки передач


Р (парковка) - в это положение рычаг можно переводить только после полной остановки автомобиля и фиксации его ручным тормозом. Именно в этом положении следует осуществлять запуск двигателя.

R (задний ход) – можно включать, удерживая педаль тормоза нажатой и только после полной остановки автомобиля (иначе не избежать поломок).

N (нейтральное положение) – означает, что крутящий момент от двигателя не передается ведущим колесам. При этом положении рычага разрешается запуск двигателя. Во время движения автомобиля «N» не включать - возможна поломка!

D (движение) – именно при этом положении рычага селектора обеспечивается движение автомобиля в нормальных условиях. В этом режиме, по мере увеличения или уменьшения скорости движения автомобиля, автоматически, без участия водителя, последовательно меняются несколько передач.

D3 (S) - диапазон пониженных передач. Обычно включается на дороге с небольшими подъемами и спусками. Торможение двигателем более эффективно, чем в положении «D».

D2 (L) - второй диапазон пониженных передач. Включается водителем в тяжелых дорожных условиях (горы, бездорожье и тому подобное). Торможение двигателем более эффективное, чем в положении «S».

 

Перевод рычага селектора автоматической коробки передач из положения D в положение D2 или D3 и обратно может производиться во время движения автомобиля.


Автоматические коробки передач последних лет выпуска могут дополнительно оборудоваться переключателями режимов разгона:

  • N – нормальный,
  • Е – экономичный,
  • S – спортивный.

Для начала движения автомобиля следует, нажав правой ногой на педаль тормоза, рукой перевести рычаг селектора из положения Р, R или N в положение D (движение), и затем выключить стояночный тормоз. При отпускании педали тормоза (правой ногой) - автомобиль начинает движение! Левая нога в управлении автомобилем вообще не принимает участия!


Для увеличения скорости движения вам достаточно лишь перенести правую ногу на педаль газа и плавно на нее нажимать, а передачи сами будут мягко переключаться от первой до последней по мере увеличения скорости. Для снижения скорости движения достаточно ослабить усилие на педали газа или вообще ее отпустить, а передачи, опять же самостоятельно, будут переключаться в нисходящем порядке.

Если же вам надо более активно снизить скорость или остановиться, то вы должны перенести правую ногу на педаль тормоза, и всего лишь мягко с ней поработать. Для начала движения после кратковременной остановки (или после снижения скорости), снова переносим правую ногу с педали тормоза на педаль газа и автомобиль начинает движение. Причем рычаг селектора постоянно остается в положении D (движение). Перемещать его не надо, кроме как при длительных остановках.

Таким образом, при городском цикле движения, водителю достаточно один раз перевести рычаг селектора автоматической коробки передач в положение D (движение), а затем правой ногой, нажимая на педаль газа или тормоза регулировать скорость движения. Остается лишь работа рулевым колесом, сигналами поворота и, конечно, головой.
Для тех, кто усмотрел в вышеизложенном явную «халяву», можно добавить, что учиться водить машину лучше с обычной коробкой передач. Научившись водить автомобиль с автоматической коробкой, в дальнейшем вы будете «обречены» управлять машинами только с «автоматом», так как не сможете правильно работать педалью сцепления. А переучиваться всегда трудно!

 

 

 

КПП - основные неисправности

Коробки передач - наиболее устойчивая часть автомобиля. Поломки КПП встречаются гораздо реже, чем других частей автомобиля. Главный фактор поддержания коробки передач в исправном состоянии - хорошее состояние масла в коробке.

Все коробки передач очень близки по своим показателям. Переднеприводные машины с цилиндрическими главными передачами, расположенными прямо в картере коробки, позволяют использовать в трансмиссии автомобиля то же масло, что используется и в двигателе.

Коробки передач заднеприводных автомобилей заправляются специальным трансмиссионным маслом.

Механические коробки передач более просты в конструкции, не требуют специальных гидравлических жидкостей для работы гидротрансформатора, менее склонны к отказам.

Автоматические коробки выигрывают удобством эксплуатации, их ремонт хорошо освоен на СТО. Автоматическая коробка передач на подержанной иномарке - это необходимый элемент риска. Такая трансмиссия накладывает на водителя повышенные обязательства по соблюдению правил эксплуатации.

Замена трансмиссионного масла

Как и моторное масло трансмиссионное масло подвержено процессу старения. Оно теряет свои первоначальные свойства и перестает выполнять свои функции.

Поддержание трансмиссионного масла в хорошем состоянии значительно продлевает срок жизни КПП. Для этого необходимо выполнение как минимум 2 пунктов:

  • Своевременная замена трансмиссионного масла
  • Применение промывки масла перед его заменой

Промывка - важная часть процесса замены масла

При смене масла без применения промывки значительная часть загрязнений остается в КПП.

Промывка:

  • Размягчает и удаляет нагар, продукты износа, углеродистые отложения
  • Прочищает масляные каналы, улучшая циркуляцию масла
  • Обеспечивает более полный слив старого масла
  • Безопасна для резиновых уплотнителей, сальников

Выход из строя автоматической трансмиссии обычно происходит по одной из следующих причин или их комбинаций:

  • закисание или неправильная регулировка троса управления;
  • неисправности гидравлической части системы управления;
  • неисправности механической части;
  • износ одного или нескольких фрикционных элементов управления;
  • неисправности гидротрансформатора;
  • неисправности электрической части системы управления;
  • выхода из строя электронного блока управления;
  •  
  • нарушения регулировок.

Нижеприведенная методика значительно облегчает процедуру поиска неисправности и ее устранение. Однако окончательный успех в значительной мере зависит от точности собранной информации и правильной ее интерпретации.

Диагностические процедуры

Поиск неисправности АКП рекомендуется проводить в следующей последовательности:

  • Проверка уровня масла и его состояния (в главе "Трансмиссионное масло").
  • Беглая проверка работы двигателя на режиме холостого хода, вакуума, мест соединений электропроводки, троса механизма выбора диапазона и троса управления клапаном-дросселем (для АКП с гидравлической системой управления).
  •  
  • Определение кодов неисправностей блоков управления двигателем и коробкой передач.
  • Проверка на полностью заторможенном автомобиле.
  • Проверка в движении (дорожные испытания).
  • Проверка давлений в системе управления.

Проверка троса управления клапаном-дросселем

Важная деталь в управлении АКП - трос управления клапаном-дросселем. Он соединяет механизм управления АКП с сектором дроссельной заслонки двигателя или рычагом управления ТНВД. Трос состоит из металлической основы, помещенной в пластиковую оболочку, жестко закрепленную с обеих сторон. При долгой эксплуатации, оболочка троса, высыхая, укорачивается и вылезает из точек крепления.

Разрегулированный трос может быть причиной переключения передач на повышенных или пониженных оборотах. Переключение передач на повышенных оборотах ведет к преждевременному износу деталей

коробки и может быть причиной повышенного расхода топлива. Некоторые неисправности АКП можно устранить, отрегулировав трос управления клапаном-дросселем.

Регулировка троса

  1. · Перед регулировкой проверьте легкость хода троса. Если необходимо смажьте трос. Ослабьте регулировочные гайки.
  2. Полностью выжмите педаль управления дроссельной заслонкой и отрегулируйте трос таким образом, чтобы упор выходил из оболочки троса не более, чем на 1,5 мм. Если трос новый, то он не имеет упора. В этом случае вытягивайте внутренний трос до тех пор, пока не почувствуете небольшое сопротивление, затем, удерживая его в этом положении, установите упор на расстоянии 0,8 - 1,5 мм от чехла троса.
  3. Затяните гайки и отпустите педаль.

Проверка двигателя и его блока управления

Двигатель и коробка передач - две части одной системы и поэтому их совместная работа должна быть согласована. В связи с этим проверка работы двигателя представляет собой важный этап диагностики АКП. Если двигатель неисправен, то это может приводить к формированию неправильных сигналов, используемых системой управления трансмиссии. Система управления не имеет возможности определять достоверность поступившей информации, и в любом случае реагирует на ее изменение. В зависимости от трансмиссии, это может приводить к резким переключениям передач или комбинации резких с поздними



переключениями. Также могут иметь место и беспорядочные переключения. Кроме того, сложности могут усугубляться способностью электронной системы управления двигателем адаптироваться к его неправильной работе, что также вносит путаницу в работу системы управления АКП.

Любая неисправность двигателя должна быть устранена. Вы можете быть введены в заблуждение, и начать ремонтировать коробку передачу только для того, чтобы определить, что она исправна, а проблема заключается в неправильной работе двигателя.

ОФОРМЛЕНИЕ ОТЧЁТА: В отчёте должны быть предоставлены:

1. Описать о назначении коробок передач, дать описание устройству коробки передач и

принципу работы;

2. Определить возможные неисправности коробок передач;

 

Наименование работ

Результаты практической работы

Определить возможные неисправности коробок передач

 

Описать о назначении коробок передач, дать описание устройству коробки передач и принципу работы;



Практическая работа №12

Цели работы: изучить устройство и работу карданов и карданных передач; приобрести навыки в разборке и сборке карданных передач.

 

Оборудование: карданы и карданные передачи различных моделей автомобилей; приспособления для разборки и сборки карданных передач и карданов; тиски; наборы инструментов.

 

Содержание работы: по плакатам и учебным пособиям изучить устройство карданной передачи и отдельных карданов.

 

Описание устройства. Карданная передача состоит из карданных шарниров (карданов), карданных валов и промежуточной опоры.

В некоторых автомобилях передний кардан (рис. 1) состоит из фланца, установленного на шлицах заднего конца ведомого вала коробки передач, к которому тремя болтами 1 крепится эластичная муфта 2. Эта же муфта тремя болтами крепится к фланцу 3, установленному на шлицах переднего конца переднего карданного вала. Болты крепления эластичной муфты установлены через один: один болт крепит муфту к фланцу вторичного вала коробки передач, другой — к фланцу карданного вала. В таком же порядке установлены все шесть болтов. Наконечник переднего карданного вала 5 имеет шлицы, с помощью которых изменяется длина карданной передачи.

Если карданная передача имеет два или более карданных валов, то для подвески валов имеются промежуточные опоры 12, которые крепятся на поперечине 20. В упругой эластичной опоре установлен шариковый подшипник 13, на который опирается карданный вал. Для балансировки валов привариваются балансировочные пластины 6.

Задний карданный вал соединяется с передним карданным валом с помощью жесткого карданного шарнира, состоящего из двух вилок.

В проушины вилок вставляются шипы крестовины. На тщательно обработанных шипах крестовины установлены стальные стаканы с игольчатыми подшипниками, уплотненные изнутри уплотнительными манжетами. Крестовина со стаканами закреплена в ушках вилок крышками и стопорными пластинами. На стопорных пластинах имеются усики, которые после завертывания болтов загибаются на головки болтов для предотвращения их самопроизвольного отворачивания. Вместо крышек и стопорных пластин крестовина и стаканы могут крепиться стопорными кольцами. На стопорных пластинах может иметься надпись: «Смазывать только маслом».

 

 

hello_html_m1bc10055.jpg

 

 

 

 

 

 

 

 

 

 

 

 

 

 

 

 

 

Рис. 1. Продольный разрез [а] карданного вала по эластичной муфте и поперечный разрез [б] по промежуточной опоре:

 

— болт с гайкой крепления эластичной муфты к коробке передач; 2 — эластичная муфта; 3 — фланец переднего карданного вала; 4 — пробка смазочного отверстия; 5 — передний карданный вал; 6 — балансировочная пластина; 7 — уплотнительная манжета фланца переднего карданного вала; 8 — обойма уплотнительной манжеты; 9 — установочные метки; 10 — центрирующая втулка; 11— центрирующее кольцо; 12 — упругая промежуточная опора; 13 — шариковый подшипник; 14 — стопорное кольцо; 15 — пылеотражатели; 16 — гайка крепления вилки переднего карданного вала; 17 — задний карданный вал; 18 — вилка переднего карданного вала; 19 — гайки крепления поперечины опоры к кузову; 20 — поперечина; 21 — шайба; 22 — резиновая втулка; 23 — болт крепления упругой опоры к поперечине.

 

 

Для подачи масла на крестовине установлена масленка, а в крестовине выполнены продольные каналы. Ограничивается нагнетание масла предохранительным клапаном.

На многих легковых и грузовых автомобилях в игольчатые подшипники закладывается так называемая вечная смазка, например смазка 158 (ТУ-101320-77). Каналы в этом случае делаются несквозными.

Карданы на автомобилях ГАЗель выполнены с прокачиваемой смазочной системой. Крестовина имеет продольные каналы. В центральной части в каналы ввернута пресс-масленка, закрываемая резиновым колпачком. Для смазывания игольчатых подшипников снимается резиновый колпачок и на пресс-масленку надевается наконечник нагнетателя масла. Масло под давлением проходит по каналам к игольчатым подшипникам. Излишки масла под давлением сжимают края манжеты и выходят наружу. Смазочный материал, находящийся между грязеотражателем крестовины и уплотнительной манжетой, служит масляным фильтром, защищающим рабочую кромку манжеты от пыли и грязи.

Жесткие карданы передают момент вращения с вилки неравномерно. Для равномерной передачи момента вращения на грузовых автомобилях чаще всего устанавливают шариковые карданы с делительными канавками и кулачковые карданы.

Шариковые карданы состоят из двух кулаков, в которых имеются делительные канавки для установки ведущих шариков. Для центрирования кулаков служит центральный шарик, устанавливаемый в гнезде с помощью штифта и шпильки. Форма делительных канавок такова, что при любом повороте кулаков относительно друг друга ведущие шарики будут перекатываться и занимать положение в плоскости, делящей угол между ними пополам (биссекторная плоскость).

Реже встречается кардан равных угловых скоростей кулачкового типа, который состоит из двух вилок со вставленными полуцилиндрическими кулаками, между которыми расположен центральный диск.

Шариковые шарниры могут быть с делительными канавками и делительным рычажком. Рассмотрим принцип их действия. При передаче момента вращения под углом рабочие шарики шарниров располагаются в биссекторной плоскости, делящей угол между ведущим и ведомым валами пополам, что обеспечивает синхронность их вращения.

В шариковом шарнире с делительными канавками вилки имеют по четыре делительные канавки, средние линии которых представляют собой окружности с одинаковыми радиусами, равноудаленными от центра шарнира. При сборке шарнира сначала между вилками устанавливают центрирующий шарик на штифте, который входит в отверстия шарика и одной из вилок. От осевых перемещений штифт удерживается другим штифтом. Затем в канавки вилок, расположенных под прямым углом, закладывают четыре рабочих шарика. Так как шарики расположены симметрично относительно центра, то при передаче момента вращения под углом центры шариков описывают окружности, находящиеся в биссекторной плоскости, причем в передаче момента участвуют только два шарика. Шарнир может передавать момент под углом 40...32°.

Достоинства шарнира: простота изготовления и сравнительно небольшая стоимость: недостатки: ускоренное изнашивание шарнира из-за скольжения шариков относительно канавок и высокого давления, вызванного, в частности, тем, что момент вращения передают только два шарика.

В шариковом шарнире с делительным рычажком в передаче момента вращения участвуют все шарики, что уменьшает усилие и увеличивает срок службы шарнира.

Детали шарнира расположены в чашке, имеющей на внутренней поверхности шесть сферических канавок для установки шести шариков. Такие же канавки имеет и сферический кулак, в шлицевое отверстие которого входит ведущий вал. Делительное устройство, устанавливающее шарики в биссекторной плоскости, состоит из сепаратора, в котором они расположены, сферической чашки и делительного рычажка. Делительный рычажок своими сферическими поверхностями входит в гнезда ведущего и ведомою валов и в отверстие чашки и пружиной прижат к ведущему налу. Плечи рычажка подобраны так, что при передаче момента вращения под углом он поворачивает сепаратор па угол, равный половине угла между осями ведущего и ведомого валов, т. е. устанавливает шарики в биссекторной плоскости. Шарнир может передавать момент вращения под углом 35...38°.

Достоинство шарнира — передача момента вращения всеми шариками, что повышает долговечность шарнира и позволяет уменьшить его габаритные размеры. Однако он сложнее и дороже шарнира с делительными канавками.

Сдвоенный кулачковый карданный шарнир равных угловых скоростей привода ведущих управляемых колес автомобиля «Урал- 375» состоит из ведущей и ведомой вилок, связанных с ведущим и ведомым валами, и вставленных в вилки кулаков, в пазы которых вставляется центральный диск. Этот диск является промежуточным звеном шарнира. Кулачковый карданный шарнир может передавать момент вращения под углом 45...50°. Устройство этого шарнира проще, он дешевле шариковых карданов, но его КПД ниже.

 

Порядок разборки промежуточного вала:

1.                              снятый карданный вал с промежуточной опорой укрепить на стенде;

2.                              ослабить передний хомут грязезащитного чехла и сдвинуть чехол;

3.                              отогнуть замочную шайбу;

4.                              отвернуть гайку уплотнительной манжеты шлицевого соединения и сдвинуть ее в сторону отодвинутого чехла;

5.                              снять со шлицевого конца карданного вала скользящую вилку, уплотнительные манжеты и упорное кольцо;

6.                              вынуть карданный вал из промежуточной опоры.

 

Порядок сборки промежуточного вала:

1.                              вставить карданный вал в промежуточную опору;

2.                              на шлицевой конец карданного вала поставить упорное кольцо и уплотнительные манжеты;

3.                              вставить шлицевой конец скользящей вилки так, чтобы ее пружины находились в одной плоскости с проушинами вилки переднего конца карданного вала;

4.                              завернуть гайку уплотнительной манжеты шлицевого соединения и отогнуть замочную шайбу;

5.                              надвинуть грязезащитный чехол и затянуть его хомутами.

 

Порядок разборки карданных шарниров:

1.                              отогнуть концы стопорных пластин;

2.                              отвернуть болты крышек крепления подшипников и снять две стопорные пластины и две крышки подшипников;

3.                              выпрессовать оба подшипника: один — наружу, другой — внутрь вилки;

4.                              снять наружный выпрессованный подшипник, внутренний снова вставить в гнездо вилки, выпрессовать наружу, а затем снять с крестовины;

5.                              вывернуть из крестовины пресс-масленку и предохранительный клапан;

6.                              снять с шипов крестовины четыре обоймы уплотнительных манжет и пробки колец.

 

Порядок сборки карданных шарниров:

1.                              завернуть в крестовину пресс-масленку и предохранительный клапан;

2.                              напрессовать на каждый шип крестовины обоймы уплотнительных манжет и пробковые кольца;

3.                              завести крестовину в проушины вилки кардана;

4.                              запрессовать два подшипника в проушины вилки, причем паз на торце подшипника должен лежать на оси двух резьбовых отверстий в проушине;

5.                              установить крышки подшипников в стопорные пластины и привернуть;

6.                              завести проушины фланца на два других шипа крестовины;

7.                              отогнуть концы стопорных пластин и прижать их к граням болтов.

 

Порядок разборки промежуточной опоры карданного вала:

1.                              освободить и сдвинуть грязезащитный чехол;

2.                              отвернуть гайку уплотнительной манжеты шлицевого соединения;

3.                              разъединить промежуточный и главный карданные валы и снять промежуточную опору;

4.                              снять с промежуточной опоры кронштейн в сборе с накладкой, резиновую подушку, передние и задние отражатели и шариковый подшипник опоры.

 

Порядок сборки промежуточной опоры карданного вала:

1.                              запрессовать шариковый подшипник в резиновую подушку;

2.                              поставить отражатели (перед установкой отражателей посадочные поверхности следует обжать, обеспечив тугую посадку);

3.                              надеть кронштейн промежуточной опоры и напрессовать опору на промежуточный вал, соединить главный и промежуточный карданные валы;

4.                              завернуть гайку уплотнительной манжеты шлицевого соединения;

5.                              надеть грязезащитный чехол.

 

КОНТРОЛЬНЫЕ ВОПРОСЫ

1.                              Опишите назначение и основные части карданной передачи.

2.                              Опишите назначение, устройство и работу карданов неравной угловой скорости.

3.                              Опишите назначение, устройство и работу промежуточной опоры карданных передач.

4.                              Опишите назначение, устройство и работу карданов равной угловой скорости.

5.                              Опишите назначение, устройство и работу шариковых карданов равной угловой скорости.

6.                              Опишите назначение, устройство и работу кулачковых карданов равной угловой скорости.

7.                              Опишите назначение, устройство и работу «мягкого» кардана.

 

Практическая работа №13

Цели работы: изучить устройство и работу главных передач и дифференциалов; приобрести навыки в разборке и сборке механизмов ведущих мостов.

 

Оборудование: ведущие мосты автомобилей; приспособления для разборочных работ; наборы рожковых, накидных и торцевых ключей; тиски; круглогубцы; выколотки.

 

Содержание работы: по плакатам и учебным пособиям изучить устройство и работу главных передач и дифференциалов; научиться разбирать и собирать механизмы ведущих мостов.

 

Описание устройства. Ведущий мост воспринимает силы, действующие между колесами и подвеской, передает момент вращения от карданной передачи к ведущим колесам автомобиля и воспринимает вертикальные, продольные и поперечные усилия.

 

Главная передача предназначена для увеличения момента вращения в передаточное число раз и для передачи его под углом 90° на ведущие колеса автомобиля.

Одинарные главные передачи с гипоидным зацеплением устанавливают на легковые и грузовые автомобили малой и средней грузоподъемности. Такие передачи состоят из ведущего зубчатого колеса 1 (рис. 1), изготовленного как одно целое с ведущим валом. Вал вращается в двух конических роликоподшипниках 4. Передний конец вала имеет шлицы для установки фланца 2, к которому присоединяется карданная передача. Фланец на валу крепится гайкой, которая шплинтуется. Ведущее зубчатое колесо 1 находится в постоянном зацеплении с ведомым зубчатым колесом 17, которое болтами крепится к корпусу дифференциала и вместе с ним вращается в двух конических роликоподшипниках. В грузовом автомобиле ЗИЛ-433100 ведущее зубчатое колесо установлено в стакане на двух конических роликоподшипниках и одном цилиндрическом, расположенном в перегородке картера главной передачи. На заводе подшипники ведущего зубчатого колеса устанавливают с предварительным натягом. Между внутренними кольцами конических подшипников ведущего вала имеется распорная регулировочная втулка, толщину которой подбирают так, чтобы обеспечить требуемый предварительный натяг подшипников. Между фланцем стакана подшипников и картером главной передачи установлены регулировочные прокладки. Регулированием их толщины устанавливают положение ведущего зубчатого колеса в осевом направлении.

 

hello_html_m25e705e2.jpg

 

 

 

 

 

 

 

 

 

 

 

 

 

 

 

 

 

 

 

 

 

 

 

 

Рис. 1. Главная передача заднего моста автомобиля ЗИЛ-433100:

1 — ведущее зубчатое колесо; 2 — фланец; 3 — уплотнительная манжета; 4 — конические роликоподшипники; 5 — цилиндрический роликоподшипник; 6 — шток муфты блокировки дифференциала; 7 — включатель сигнальной лампы (токировки дифференциала; 8 — камера механизма блокировки; 9— мембрана; 10 — вилка включения блокировки; 11— муфта включения блокировки дифференциала; 12 — регулировочная гайка подшипника дифференциала; 13—корпус муфты блокировки; 14 — муфта блокировки дифференциала; 7 5 — зубчатое колесо полуоси; 16 — крестовина сателлитов; 17 — ведомое зубчатое колесо; 18 — картер главной передачи; 19 — опорный болт; 20 —регулировочные прокладки

 

 

На автомобилях марки ВАЗ картер главной передачи объединен с картером коробки передач. Ведущее зубчатое колесо главной передачи выполнено как одно целое с ведомым валом коробки передач и находится в постоянном зацеплении с ведомым зубчатым колесом главной передачи, закрепленным болтами на корпусе дифференциала. Вращаются они на конических роликовых подшипниках.

На грузовых автомобилях средней и большой грузоподъемности устанавливают двухступенчатые главные передачи, что обусловлено необходимостью передавать больший момент вращения, поэтому для уменьшения нагрузки на зубья применяется две пары зубчатых колес: одна — коническая, другая — цилиндрическая.

 

Ведущее коническое зубчатое колесо 11 (рис. 2) изготовлено как единое целое с ведущим валом, который вращается в двух конических роликоподшипниках 6 и 9, установленных в стакане 7, который болтами крепится к картеру заднего моста. Между стаканом 7 и картером моста установлены регулировочные прокладки 10. С их помощью регулируется затяжка роликоподшипника 9. Роликоподшипник 6 имеет регулировочные шайбы 8. На переднем конце ведущего вала имеются шлицы для установки фланца, к которому присоединяется карданная передача. Фланец на валу крепится коронной гайкой. Гайка шплинтуется.

Ведущее коническое зубчатое колесо 11 находится в зацеплении с ведомым зубчатым колесом 12, которое с помощью заклепок закреплено на фланце промежуточного вала, который вращается на двух роликоподшипниках 14 и 31, установленных на валу. Подшипники закрыты крышками 15 и 32. Под крышками находятся регулировочные прокладки 13. Регулируют затяжку подшипников изменением толщины прокладок.

 

hello_html_m5f470234.jpg

 

 


Рис. 2. Ведущий мост автомобиля ЗИЛ – 431410:



1 — фланец; 2 — манжета; 3, 15, 18 и 32 — крышки; 4 —шайба; 5 — уплотни- м:пьная прокладка; 6, 9, 14, 24 и 31 — роликоподшипники; 7 — стакан; 8 — регулировочная шайба; 10 и 13 — регулировочные прокладки; 11— ведущее коническое зубчатое колесо; 12 — ведомое коническое зубчатое колесо; 16 — цилиндрическое зубчатое колесо; 17 — картер главной передачи; 19 и 29 —опорные шайбы; 20 и 23 — правая и левая чашки дифференциала; 21 —ведомое цилиндрическое зубчатое колесо; 22 — полуосевое зубчатое колесо; 25 — регулировочная гайка; 26 — полуось; 27 — картер моста; 28 —сателлит; 30 — крестовина; .43 — распорная втулка.

 

В средней части промежуточного вала имеется ведущее цилиндрическое зубчатое колесо 16, находящееся в постоянном зацеплении с ведомым цилиндрическим зубчатым колесом 21, которое болтами закреплено между правой 20 и левой 23 чашками дифференциала. Чашки дифференциала вращаются в двух конических роликоподшипниках 24.Затяжка этих подшипников регулируется гайками 25.

На автомобилях марок МАЗ и БелАЗ устанавливают разнесенные двойные главные передачи, у которых центральный редуктор расположен в картере заднего моста. Вторая часть главной передачи — колесная (бортовая) находится внугри ступицы задних колес, которые соединяются полуосями.

Центральный редуктор — одноступенчатая передача, состоящая из двух зубчатых колес со спиральными зубьями. Все детали главной передачи смонтированы в картере из ковкого чугуна.

Ведущее коническое зубчатое колесо изготовлено как одно целое с ведущим валом и вращается на трех подшипниках.

Наружные кольца конических роликоподшипников расположены в картере. Между ними установлено распорное кольцо и регулировочная шайба, изменением толщины которой можно регулировать необходимый натяг в конических роликоподшипниках.

На передней части ведущего вала имеются шлицы для установки фланца карданного вала. Все детали, расположенные на ведущем валу, затянуты коронной гайкой. Гайка зашплинтована.

Ведомое коническое зубчатое колесо закреплено с помощью заклепок на правой чашке дифференциала.

Колесная передача является второй ступенью главной передачи и состоит из планетарной передачи, в которую входят зубчатые колеса, сателлиты внешнего зацепления и зубчатые колеса с внутренним зацеплением.

Подвижные зубчатые колеса на эвольвентных шлицах устанавливаются на концах полуосей. Противоположные концы полуосей с помощью шлицов соединены с полуосевыми зубчатыми колесами дифференциала. Осевое перемещение подвижного зубчатого колеса на шлицах полуоси ограничивается стопорным кольцом. Осевое перемещение полуоси в сторону центрального редуктора ограничивается зубчатым колесом, а в обратную сторону — упорным сухарем. Сателлиты вращаются на осях, закрепленных в разъемном водиле, которое состоит из внутренней и наружной чашек. Чашки водила соединены между собой тремя болтами. Оси сателлитов в наружной чашке водила фиксируются стопорными болтами.

Момент вращения от ведущего конического зубчатого колеса главной передачи передается на ведомое губчатое колесо, корпус дифференциала, через крестовину и сателлиты на полуосевые зубчатые колеса, на полуоси, а с них на подвижные зубчатые колеса колесной передачи и далее через три сателлита на зубчатое колесо с внутренним зацеплением, а с него на ступицу заднего ведущего колеса автомобиля.

Дифференциал предназначен для распределения момента вращения между ведущими колесами автомобиля, что обеспечивает колесам возможность вращаться с разными угловыми скоростями.

Дифференциал состоит из двух чашек, на одной из которых или между обеими закреплено заклепками или болтами ведомое зубчатое колесо главной передачи. В чашках имеются гнезда для установки крестовины с четырьмя шипами, на которые надеваются сателлиты. С ними в постоянном зацеплении находится два молуосевых зубчатых колеса, имеющие внутри шлицы для присоединения приводных валов (полуосей) колес. Для уменьшения трения и регулировки зазоров между чашками и полуосевыми зубчатыми колесами и сателлитами установлены шайбы. Шайбы со стороны зубчатых колес и сателлитов для удержания масла имеют густую накерненную сетку. Чашки дифференциала стягиваются болтами. Вращается дифференциал на двух конических роликоподшипниках, установленных в гнездах картера главной передачи. Регулировка затяжки подшипников осуществляется специальными гайками.

При повороте автомобиля колеса, катящиеся по внутренней колее, замедляются относительно ведомого зубчатого колеса главной передачи, так как двигаются по короткой дуге, что заставляет сателлиты вращаться вокруг своей оси. Зубья сателлитов выполняют роль рычагов, воздействующих на зубья полуосевых зубчатых колес, которые передают усилия равномерно на полуосевые зубчатые колеса. Поэтому на сколько одно полуосевое зубчатое колесо отстанет от ведомого зубчатого колеса главной передачи, на столько другое полуосевое зубчатое колесо должно его обогнать. Частота вращения ведомого зубчатого колеса главной передачи и чашек дифференциала всегда равна полусумме частот вращения правого и левого полуосевых зубчатых колес, а следовательно, правого и левого ведущих колес автомобиля.

В автомобиле ЗИЛ-433100 механизм блокировки симметричного дифференциала установлен на главных передачах заднего моста. Он состоит из камеры механизма блокировки, закрытой крышкой, под которой закреплена мембрана. Под мембраной установлена возвратная пружина и шток муфты блокировки дифференциала. На штоке закреплена вилка включения блокировки, которая входит в кольцевую проточку на муфте блокировки. Для контроля за включением блокировки имеется сигнальная лампа, вмонтированная в клавишу включения механизма блокировки на панели приборов.

При включении механизма блокировки электропневматический клапан подает сжатый воздух пневмосистемы автомобиля в камеру механизма блокировки. Под действием воздуха мембрана, передвигаясь, вилкой передвигает муфту блокировки. Муфта включения торцевыми зубьями соединяется с муфтой блокировки, правый приводной вал (полуось) блокируется с чашкой дифференциала. Таким образом, две полуоси создают единый вал, что и обеспечивает движение автомобиля. При выезде на дорогу с хорошим покрытием блокировку следует выключить.

Кроме дифференциалов с принудительной блокировкой применяют самоблокирующиеся дифференциалы. Если самоблокировка осуществляется вследствие внутреннего трения между деталями, дифференциал называется дифференциалом повышенного трения. Такие дифференциалы устанавливают на автомобилях ГАЗ-66-11.

Основной деталью кулачкового дифференциала является сепаратор, выполненный вместе с чашкой дифференциала. К чашке болтами крепится ведомое зубчатое колесо главной передачи. В сепараторе имеется два ряда отверстий в шахматном порядке для установки сухарей. В каждом ряду по 12 отверстий. Сухари на сторонах, обращенных друг к другу, имеют срезы. Срезы делаются с обоих концов, но в средней части оставлен поясок, ширина которого меньше толщины сепаратора. Между рядами сухарей снаружи и внутри сепаратора установлены стопорные кольца, предотвращающие проворачивание сухарей вокруг своих осей и удерживающие их от выпадения из сепаратора при разборке и сборке дифференциала. Между рядами сухарей внутри сепаратора установлена внутренняя звездочка с двумя рядами кулачков. В каждом ряду имеется по шесть кулачков, расположенных в шахматном порядке. Снаружи сепаратор охватывается наружной звездочкой. Внутри она имеет один ряд (шесть штук) кулачков. Закрываются звездочки второй чашкой дифференциала. Обе звездочки имеют внутренние шлицы для соединения с полуосями.

Передача момента вращения происходит следующим образом. С ведущего зубчатого колеса главной передачи вращение передается на ведомое зубчатое колесо главной передачи, а затем через болты на чашки дифференциала и сепаратор. При вращении сепаратора сухари упираются в выступы кулачков внутренней и наружной звездочек, заставляя их вращаться, вращать полуоси и ведущие колеса автомобиля.

Если одна из звездочек испытывает большее сопротивление, чем другая, то она будет вращаться медленнее сепаратора. В этом случае эта звездочка будет толкать своими кулачками сухари в сторону другой звездочки, ускоряя ее вращение.

На автомобилях с колесной формулой 6x4 (автомобили марки КамАЗ) устанавливают два ведущих моста: средний и задний. Конструктивно они изготовлены одинаково. Основное отличие включается в том, что средний мост имеет межосевой блокируемый дифференциал.

Крутящий момент к среднему мосту от коробки передач подводится карданной передачей на вал чашки межосевого дифференциала, далее через крестовину и сателлиты передается на вал заднего моста, а через другое коническое зубчатое колесо вращение передается на ведущее коническое зубчатое колесо среднего моста. Главные передачи на обоих ведущих мостах — двухступенчатые с проходным валом, имеют пары спиральных конических зубчатых колес и пары косозубых цилиндрических колес.

Межосевой дифференциал, установленный на среднем мосту, предназначен и для распределения момента вращения между задним и средним ведущими мостами. Это необходимо в том случае, если радиусы качения колес разных мостов будут отличаться.

Межосевой дифференциал состоит из картера 32 дифференциала (рис. 3), левой и правой чашек, соединенных болтами. Между чашками находится крестовина 23, на шипах которой установлены сателлиты 35, находящиеся в постоянном зацеплении с зубчатым колесом 24 привода заднего моста и зубчатым колесом 21 привода среднего моста. Механизм блокировки состоит из корпуса 15 с крышкой 14. Между корпусом и крышкой находится мембрана 10 с возвратной пружиной 7 и нажимной пружиной 8. На стержне 6 механизма блокировки закреплена установочным винтом 4 и гайкой 3 вилка муфты 16, которая вставлена в кольцевую проточку муфты 19 блокировки межосевого дифференциала.

Дифференциал в сборе установлен на двух опорах: одной из них является шариковый подшипник 27, а другой — два конических роликоподшипника ведущего конического зубчатого колеса среднего моста.

 

 

К крану включения

hello_html_586ff51d.jpg











Рис. 3. Межосевой дифференциал с механизмом блокировки и кран включения механизма блокировки:

 

1.                              — заглушка: 2 — микровыключатель; 3 и 29 —гайки; 4 — установочный винт; 5 — заливная пробка; 6 —стержень механизма блокировки; 7 — возвратная пружина; 8— нажимная пружина; 9 — стакан стержня; 10 — мембрана; 11 — шланг; 12 —крышка стакана; 13 и 17 —стопорные кольца; 14 — крышка корпуса; 15 — корпус механизма блокировки; 16 — вилка муфты; 18 — муфта зубчатого колеса привода среднего моста; 19 — муфта блокировки межосевого дифференциала; 20 — сливная пробка; 21 —зубчатое колесо привода среднего моста; 22, 26, 33 и 36 — опорные шайбы; 23 —крестовина; 24 — зубчатое колесо привода заднего моста; 25 — самоконтрящийся болт; 27 — шариковый подшипник; 28 — крышка подшипника; 30 — шайба; 37 — фланец; 32 — картер межосевого дифференциала; 34 — чашка дифференциала в сборе [комплект]; 35 — сателлит

 

Порядок разборки главной передачи:

1.                              снять шплинтовочную проволоку болтов крышки подшипников;

2.                              отвернуть два болта и снять стопорные пластины;

3.                              отвернуть болты крепления крышек подшипников дифференциала в сборе и снять крышки;

4.                              отвернуть регулировочные гайки и снять наружные кольца подшипников;

5.                              ослабить контргайку и отвернуть регулировочный винт;

6) снять дифференциал в сборе;

7.                              отвернуть болты крепления ведущего зубчатого колеса к картеру редуктора;

8.                              выпрессовать ведущее зубчатое колесо в сборе из картера редуктора и снять регулировочные прокладки, спрессовать с ведущего вала подшипник;

9.                              отогнуть концы стопорной пластины, отвернуть болт крепления масляной трубки и снять стопорную пластину;

10.                          снять пружину, тарелку маслоприемной трубки, маслоприемную трубку;

11.                          отвернуть маслоналивную и маслосливную пробки.

 

Порядок сборки главной передачи:

1.                              наложить прокладки на плоскость горловины картера редуктора в сборе так, чтобы совпали шесть отверстий и отверстие для масла с отверстиями на плоскости горловины картера редуктора н сборе;

2.                              два фиксатора вставить в отверстие крышки уплотнительной манжеты, прокладки, муфты подшипников ведущего зубчатого колеса главной передачи в сборе. Один из фиксаторов должен находиться около выступа для масла в крышке уплотнительной манжеты. Весь комплект положить на ранее уложенные прокладки I а к, чтобы выступ для масла совпал с отверстием для масла в прокладках и горловины картера редуктора в сборе;

3.                              запрессовать ведущее зубчатое колесо в сборе и снять фиксаторы;

4.                              вставить шесть болтов с шайбами и завернуть;

5) дифференциал в сборе установить в гнездо для подшипников дифференциала в картере редуктора, установить ведомое зубчатое колесо в зацепление с ведущим зубчатым колесом;

6.                              установить крышки подшипников, завернуть болты;

7.                              надеть наружные кольца подшипников дифференциала;

8.                              завернуть регулировочные гайки подшипников дифференциала;

9.                              завернуть болты крышки подшипников, проверяя свободное вращение регулировочных гаек и зашплинтовать проволокой;

10.                          проверить осевой зазор в подшипниках дифференциала;

11) завернуть регулировочный винт с гайкой в картер редуктора до отказа, затем отвернуть на 1/оборота, проверить вращение ведомого зубчатого колеса и, убедившись в том, что нет задевания, завернуть винт с гайкой, проверить биение «затылка» ведомого зубчатого колеса;

7.                              вставить в масляный канал маслоприемную трубку так, чтобы боковое отверстие совпало с каналом картера редуктора в сборе;

8.                              завернуть стопорный болт со стопорной пластиной, убедиться в правильном зацеплении ведущего и ведомого зубчатых колес по пятну контакта;

9.                              вложить в тарелку маслоприемной трубки пружину, ввернуть и затянуть трубку маслоприемного отверстия;

10.                          завернуть и затянуть пробки маслосливного и маслоналивного отверстий.

 

Порядок разборки дифференциала (использовать приспособления):

1.                              поставить коробку дифференциала с ведомым зубчатым колесом главной передачи так, чтобы зубья зубчатого колеса были направлены вверх, и выпрессовать подшипник левой части коробки;

2.                              снять проволоку и отвернуть болты крепления коробки дифференциала, разъединить коробку, вынуть опорные шайбы, зубчатые колеса полуосей и сателлитов, крестовину;

3.                              расшплинтовать болты и отвернуть гайки болтов крепления ведомого зубчатого колеса главной передачи;

4.                              отвернуть болт крепления маслоуловителя, предварительно отогнув концы стопорного кольца (для снятия стопорного кольца оси сателлитов воспользоваться круглогубцами);

5.                              снять маслоуловитель и шайбу;

6.                              выпрессовать подшипник правой части дифференциала;

7.                              вынуть болты крепления ведомого зубчатого колеса главной передачи;

8.                              разъединить левую часть коробки дифференциала с ведомым зубчатым колесом главной передачи;

9.                              промыть детали разобранного дифференциала, зачистить забоины и протереть.

 

Порядок сборки дифференциала:

1.                              два подшипника в сборе наложить на правую и левую части коробки дифференциала, запрессовать подшипники на коробку дифференциала до упора, проверить биение дифференциала;

2.                              поставить левую часть коробки дифференциала в приспособление на гидропресс и запрессовать в нее ведомое зубчатое колесо главной передачи;

3.                              взяв левую часть коробки дифференциала в сборе с напрессованным ведомым зубчатым колесом главной передачи и подшипниками, вставить 12 болтов в отверстия для крепления ведомого зубчатого колеса к левой части коробки;

4.                              завернуть на болтах гайки и зашплинтовать;

5.                              вставить в отверстие левой части коробки маслоуловитель и навернуть болт крепления маслоуловителя, предварительно надев на него стопорную шайбу, согнуть концы шайбы;

6.                              вставить опорные шайбы в правую и левую части коробки дифференциала лунками на торце наружу и зубчатые колеса полуосей, проверить зазор;

7.                              надеть на крестовину четыре зубчатых колеса сателлитов и четыре опорные шайбы, поставить крестовину с зубчатыми колесами в левую часть коробки дифференциала;

8.                              вставить восемь болтов в отверстия коробки дифференциала, болты завернуть и зашплинтовать.



Порядок снятия полуоси:

1.                              отвернуть гайки крепления полуоси к ступице;

2.                              отвернуть контргайки съемных болтов полуоси и, ввертывая болты, снять полуось и прокладку.

 

Порядок установки полуоси:

1.                              надеть прокладку на шпильки;

2.                              вставить полуось в отверстие картера, надев фланец полуоси на шпильки ступицы;

3.                              надеть на шпильки разжимные втулки, шайбы и навернуть гайки до отказа;

4.                              завернуть болты с контргайками.

 

Установка колес автомобиля. Колеса автомобиля должны быть установлены так, чтобы обеспечить:

1.                              легкое управление;

2.                              наименьший износ шин и деталей;

3.                              устойчивость (стабилизацию) передних управляемых колес, и среднем положении, соответствующем прямолинейному движению.

 

Развал колес. При эксплуатации автомобиля шкворни поворотных цапф и их втулки постепенно изнашиваютя. В результате увеличения зазора между ними происходит отклонение плоскости колеса от вертикальной плоскости (рис. 4, а), что отрицательно влияет на износ шин и управляемость автомобилем. В качестве меры борьбы с этим применяют установку поворотных цапф с наклоном вниз. Отклонение верхней части колеса от вертикальной плоскости наружу называется положительным развалом. За счет этого появляется осевая сила, прижимающая ступицу к внутреннему большому подшипнику, разгружая наружный маленький подшипник. При развале колес уменьшается расстояние между точкой пересечения продолжения оси шкворня с дорогой и точкой контакта колеса с дорогой, что и облегчает поворот колес. Угол развала у разных моделей автомобилей находится в пределах 0... 2°.

Для облегчения управления автомобилем передние управляемые колеса имеют развал в вертикальной плоскости и схождение в горизонтальной плоскости. Для возврата колес в среднее исходное положение шкворни поворотных цапф наклонены в продольной и поперечной плоскостях. Задние подвески переднеприводных легковых автомобилей в настоящее время выполняются с развалом и схождением

Износ шин. Развал колес оказывает влияние на износ шин. Наименьший износ будет при отсутствии развала. При развале до 2° износ шин будет не очень большим. При эксплуатации автомобиля за счет износа шкворней, втулок и усталостного износа балки передней оси положительный развал постепенно уменьшается до нуля, а затем отклонение колес переходит в сторону отрицательного развала и износ шин возрастает.

У грузовых автомобилей изменение развала устраняется заменой изношенных деталей, а у легковых автомобилей — регулировкой.

 

Схождение колес. В результате наклона колес при развале возникают силы, стремящиеся развернуть их в разные стороны при движении. Появляется поперечное проскальзывание колес, что способствует износу шин и затрудняет управление автомобилем. Для устранения вредных последствий развала колеса устанавливают со схождением. При этом расстояние между ободами колес на уровне передней оси спереди на несколько миллиметров меньше, чем сзади. Величина схождения находится в прямой зависимости от величины развала и делается в пределах 0...12 мм.

 

 

hello_html_me8e1316.jpghello_html_6323cf40.jpg




hello_html_57190618.jpg







Рис. 4. Схемы [а...в] установки управляемых колес:



1 — рулевая тяга; α — угол развала колес; β — угол поперечного наклона шкворня; γ — угол продольного наклона шкворня; А — расстояние между колесами на уровне оси сзади; В — расстояние между колесами на уровне оси спереди; С — расстояние от точки пересечения продолжения шкворня с дорогой до точки касания шины колеса

Схождение колес регулируется у грузовых автомобилей изменением длины поперечной рулевой тяги, а у легковых автомобилей — изменением длины боковых регулировочных трубок.

У легковых автомобилей Hyundai Santa Fe, Hyundai Santa Fe Classic, Nissan Pathfinder, Hyundai Accent, Lada Priora, KiaRio, Renault Logan и др. схождение и развал предусмотрены конструкцией и для передних, и для задних колес.

При этом на автомобилях Hyundai Santa Fe, Hyundai Santa Fe Classic, Nissan Pathfinder, Hyundai Accent конструкцией предусмотрено регулирование при необходимости схождения и развала передних и задних подвесок.

На автомобилях Lada Priora схождение и развал передней подвески подлежит проверке и регулировке, а для задней подвески схождение и развал предусмотрены конструкцией, но регулировке' не подлежат.

На автомобилях Renault Logan и Kia Rio схождение передних колес регулируют, регулировка развала не предусмотрена, а для задних колес схождение и развал предусмотрены конструкцией, но регулировке не подлежат.

Стабилизация колес в среднем положении достигается поперечным и продольным наклонами шкворней поворотных цапф за счет формы передней оси автомобиля.

Поперечный наклон на угол 6... 10° (рис. 4, 6) при повороте колес вынуждает переднюю ось опуститься к поверхности дороги, что невозможно, и тогда передняя часть автомобиля поднимается. После освобождения рулевого колеса сила тяжести заставляет переднюю ось опуститься, возвращая передние управляемые колеса в среднее исходное положение для прямолинейного движения. Сила тяжести способствует увеличению устойчивости колес в этом положении. Поперечный наклон шкворня способствует стабилизации колес на малых скоростях движения.

Продольный наклон шкворня предназначен (рис. 4, в) для обеспечения стабилизации управляемых колес в среднем положении на больших скоростях движения автомобиля при значительных центробежных силах. Продолжение оси шкворня пересекается с дорогой впереди точки касания шины колеса на некотором расстоянии. На больших скоростях во время поворота колес возникает центробежная сила, стремящаяся сдвинуть автомобиль по направлению от центра поворота. Между шинами и дорогой в точках их касания появляются силы трения, действующие с определенным плечом относительно оси шкворня и способствующие возврату колес в среднее положение для прямолинейного движения. Величина продольного наклона шкворня выдерживается в пределах 0...3,5°.

Установка шкворней с большими углами наклона затрудняет управление автомобилем, вследствие чего на легковых автомобилях эти углы делают очень малыми или равными нулю. На легковых автомобилях применяют эластичные шины и стабилизация колес в среднем положении обеспечивается углом увода упругих деформирующихся шин. Сама шина за счет своей упругости после окончания поворота стремится вернуть колеса в нейтральное положение.

Если передние колеса не только управляемые, но еще и ведущие, то углы продольного наклона шкворней также малы или равны нулю. Тяговое усилие ведущего переднего моста способствует улучшению стабилизации колес в среднем положении.

Измерителями стабилизации колес при выходе автомобиля из поворота служат стабилизирующий момент и угловая скорость поворота рулевого колеса при возвращении его в нейтральное положение. Стабилизирующий момент возникает благодаря продольному и поперечному наклонам шкворней, а также вследствие поперечной эластичности шин.

КОНТРОЛЬНЫЕ ВОПРОСЫ

1.                              Каково назначение главной передачи? Какие существуют типы главных передач? Укажите область их применения. Опишите устройство и работу гипоидной главной передачи автомобиля.

2.                              Опишите устройство и работу двойной главной передачи автомобилей марок ЗИЛ, КамАЗ.

3.                              Опишите устройство и работу разнесенной главной передачи автомобиля МАЗ-500А (устройство и работу главного редуктора и колесной передачи).

4.                              Опишите назначение, устройство и работу шестеренчатого кулачкового дифференциала.

5.                              Опишите назначение, устройство и работу межосевого дифференциала.

 

Практическая работа №14

Цели занятия:

Образовательная:

1. Ознакомить обучающихся с операциями выполняемыми при монтаже и демонтаже шин. Балансировки колес

2. Ознакомить обучающихся с инструментами и оборудованием применяемыми при монтаже и демонтаже шин. Балансировки колес.

3. Сформировать практический опыт выполнения операций по монтажу и демонтажу шин. Балансировки колес

Воспитательная:

1. Воспитать у обучающихся бережное отношение к технике, инструменту и материалам.

2. Воспитывать аккуратность и внимательность в работе, бдительности в соблюдении требований безопасности труда.

3.Воспитывать интерес к профессии, добросовестность, ответственность и культуру труда.

Развивающая:

1. Побуждать обучающихся к мышлению во время работы.

2. Развивать их творческие способности, сообразительность, производственную смекалку.

3. Развивать техническое мышление, самостоятельность, умение использовать в речи профессиональную терминологию.



Ход занятия

1.Организационная часть

1.1Контроль посещаемости и наличие спецодежды.

2.Вводный инструктаж

2.1 Сообщение целевой установки: мастер сообщает тему занятия, разъясняет цели.

2.2 Актуализация знаний: 1) Что обозначает первая цифра в обозначении протектора?

-Ширина профиля — это линейное расстояние между наружными сторонами

боковин. Именно это значение ставится в первом числе индекса.

2) Снижение давления в покрышке ниже рекомендуемого приводит:

- Уменьшение давления ниже требуемого неизбежно ухудшит управляемость и скажется на безопасности.

Еще один эффект "гипотонии в колесах" - повышенный износ покрышки.

3) Как часто необходимо выполнять балансировку колес?

- Выполнять балансировку нужно каждый сезон при смене резины, но не менее чем через 10 тыс. км.

4)При какой температуре воздуха стоит менять летние шины на зимние?

- Менять летнюю резину на зимнюю лучше при температуре +7С. При

этой температуре сцепление летней резины с асфальтом заметно ухудшается,

тем более в ночное время. Также не стоит забывать, что зимнюю резину нужно обкатывать,

так что, к тому времени, как температура опустится до 0 градусов по Цельсию, у вас уже

будут обкатанные покрышки.

5)Что означает, если края шины изнашиваются сильней центральной час?

- Это означает, что колесо не отбалансировано

2.3 Формирование ориентировочных основ действий:

-значение данной работы для освоения профессии: Эффективность использования автомобилей и их эксплуатационная надежность на прямую зависит от соблюдения сроков технических обслуживаний и квалификации рабочих выполняющих ТО и ремонт.

-ознакомление обучающихся с инструментом и материалами: На занятие вы будете использовать: Шиномонтажный станок; Балансировочный станок; Домкрат подкатной; Баллонный ключ; Монтажную лопатка. Динамометрический ключ; Очищающую жидкость; Грузы балансировочные; Колеса автомобильные; Ключ для снятия колес; Инструкционные карты; Ветошь;

-объяснение и показ приемов выполнения операций: Мастер объясняет и показывает последовательность выполнения операций согласно технологической карты.

-разбор типичных ошибок: Наиболее частые ошибки при монтаже и демонтаже шин. Балансировки колес. -это а)нарушение технологической последовательности; б)не правильное снятие показаний; в)нарушение безопасности труда.

-обсуждение безопасных условий труда: Мастер объясняет безопасные условия труда

1.Рабочий костюм должен быть застегнут во избежание захвата вращающимися частями станка.

2.Перед началом работы проверяйте техническое состояние стенда: целостность электрических кабелей, шлангов; протяжку резьбовых соединений; натяжку приводных ремней; чистоту рабочей поверхности стола. При появлении странных шумов или вибраций немедленно отключите стенд от компрессора и от сети. Не используйте его до тех пор, пока неисправность не будет устранена.

3.Рабочее место должно содержаться в чистоте, не загромождаться деталями, оборудованием, инструментом, приспособлениями, материалами.

4.На шиномонтажном станке может работать только один человек. В рабочей зоне не должны находиться посторонние люди.

5.Перед демонтажем шины полностью выпустите воздух из шины или камеры, вывернув золотник. Отбортовку шины, плотно прилипшей к ободу, осуществляйте при помощи специального стенда или съемника. В случае «прикипания» шины к ободу колеса пользуйтесь мыльным раствором.

6.Не располагайте ногу между лапой устройства демонтажа шин и резиновой подушкой.

7.Берегите руки при демонтаже и монтаже колеса, во избежание защемления пальцев рук или ладони между бортом диска и покрышкой, чтобы не допустить травмы.

8.Во время вращения монтажного стола следите за тем, чтобы детали одежды не находились близко с вращающимися поверхностями.

9. Следить, чтобы во время работы станка не возникали опасные условия. В случае возникновения неисправностей, необходимо сразу же остановить аппарат и сообщить мастеру производственного обучения.

10. Следить, чтобы рабочее место вокруг аппарата было свободным от потенциально опасных предметов (диски, шины, колеса в сборе и др.), свободным от нефтепродуктов (в том числе смазочных материалов), т.к. это опасно травмами.









-проверка усвоения материала: Мастер выбирает одного обучающегося и задает вопрос: перечислите операции по монтажу и демонтажу шин. Балансировки колес.

-сообщение критерий оценок: а)Отлично- без ошибок б)хорошо- незначительные нарушения последовательности операций; в)удовлетворительно- нарушение технических условий г) не удовлетворительно- нарушение безопасности труд.

3.Текущий инструктаж

-Обучающиеся самостоятельно, используя технологическую карту, выполняют операции по монтажу и демонтажу шин. Балансировки колес.

-Мастер проводит целевые обходы рабочих мест с целью:

А) Проверки организации рабочих мест;

Б) Проверки правильности выполнения операций;

В) Проверки соблюдения технических условий;

Г) Проверки соблюдения безопасных условий труда;

Д) Приемки и оценки выполненных операций.

4.Заключительный инструктаж

4.1 Мастер подводит итоги занятия.

4.2 Мастер указывает на допущенные ошибки и причины их возникновения.

4.3 Мастер сообщает оценки.

4.4 Выдача домашнего задания: составить Технологическую карту по замене масла в коробке передач ВАЗ-21073

 

 

 

Технологическая карта

По теме: «монтаж и демонтаж шин»

 

 

ОБЩИЕ СВЕДЕНИЯ

Шиномонтажный станок предназначен для монтажа и демонтажа шин легковых и грузовых автомобилей (посадочный диаметр – от 10 до 20 дюймов; максимальный внешний диаметр – 1000 мм). Любое иное использование станка рассматривается как не предусмотренное производителем и не допускается.

 

hello_html_6ae5d202.jpg

 

 

 

 

 

Рабочие части автоматического шиномонтажного стенда

1.                              Педаль управления вращением стола

2.                              Монтажный стол

3.                              Педаль устройства отбортовки шин

4.                              Лапа устройства отбортовки шин

5.                              Педаль сведения кулачков

6.                              Кулачок зажимной

7.                              Монтажка

8.                              Регулировочный винт

9.                              Механизм блокировки

10.                          Горизонтальная «рука»

11.                          Вертикальная направляющая

12.                          Головка монтажная

13.                          Резиновая подушка

14.                          Лопата отбортовки шин

 

 

 

 

 

 

 

 

 

ПОРЯДОК РАЗБОРКИ КОЛЕСАhello_html_2afbb956.png

1.                              Перед выполнением данного этапа полностью выпустите воздух из шины, вывернув золотник

2.                              Снимите балансировочные грузы с обеих сторон обода колеса.

3.                              Полностью сведите кулачки на монтажном столе.

4.                              Отведите лопату отжима покрышки наружу вручную. Установите колесо на пол напротив резинового буфера. Подведите лопату к покрышке на расстоянии 10 мм от края обода. Установите колесо рядом с устройством отрыва борта,hello_html_m264636f.png

5.                              Нажмите педаль отжима(3) до упора, для приведения в действие лопаты. Отпустите педаль, когда лопата переместиться до конца своего хода или когда покрышка сместиться во внутреннее углубление обода.hello_html_m6a4a0fa6.jpg

6.                              Немного поверните колесо и повторите процедуру по всей окружности обода, пока покрышка не отойдет полностью от обода.

7.                              Повторить данную операцию с другой стороны колеса.

ЗАКРЕПЛЕНИЕ ОБОДА КОЛЕСА ИЗНУТРИ

8.                              Проверьте, что зажимные кулачки сведены к центру монтажного стола.

9.                              Поместите колесо на монтажный стол и нажмите на педаль(5). При этом 4 зажимных кулачка разойдутся в стороны и зажмут обод.hello_html_3dc9aacd.png

ДЕМОНТАЖ ШИНЫ

10.                          Поместите монтажную головку (12) на бортовой закраине обода, перемещая горизонтальный кронштейн (11) и шестигранный шток(10).

11.                          Монтажную головку (12 ) блокируйте рычагом храповика (9). При этом монтажная головка автоматически поднимется на 2 мм над закраиной обода.hello_html_m154b0905.png

12.                          Вращая регулировочный винт (8), отодвиньте монтажную головку (12) примерно на 2 мм в сторону от закраины обода.

13.                          Заведите кромку монтажной лопатки (7) под верхний борт шины и, опираясь лопаткой сверху на палец монтажной головки (12), приподнимите борт шины над закраиной обода. Удерживайте монтажную лопатку в таком положении и, нажимая на педаль вращения вниз, вращайте монтажный (поворотный) стол (2) по часовой стрелке до тех пор, пока борт шины не окажется полностью над закраиной обода.

Примечание.

Чтобы при выполнении этой операции не защемило камеру, вентиль должен находиться примерно на 10 см правее монтажной головки.

14.                          Если шина имеет камеру, ее нужно вынуть.hello_html_m6c607c3f.png

15.                          Аналогичным образом, как описано в п. 13,снять с обода нижний борт шины.

16.                           

 

 

 

МОНТАЖ ШИНЫ

 

ВНИМАНИЕ! Контроль шины и обода очень важен, поскольку их дефекты могут привести к несчастному случаю при монтаже и подкачке. Перед монтажом необходимо убедиться в следующем:

- шина не имеет повреждений каркаса и иных дефектов. Если осмотр и проверка на ощупь выявили отслоение корда, разрывы или иные дефекты, шина монтажу не подлежит;

- обод не имеет вмятин и деформаций.

- посадочные диаметры обода и шины совпадают.

 

Примечание. На легкосплавных колесах вмятины часто вызывают появление внутренних микротрещин, невидимых невооруженным глазом, которые, однако, снижают прочность колеса и поэтому при накачивании шины представляют опасность.

Примечание. Диаметр обода указан на самом колесе. Диаметр шины указан на боковой стороне шины. Ни в коем случае не монтировать шины на колеса, точный диаметр которых нельзя установить.

1.                              Борта шины и бортовые закраины обода обработайте рекомендованной изготовителем

смазкой.

2.                              Если обод по каким-либо причинам был снят с поворотного стола, он должен быть вновь закреплен, как описано в разделе «Закрепление колеса».

3.                              Подведите монтажную головку (12) к бортовой закраине обода, как описано в п. 10 – 12 процедуры демонтажа.hello_html_d29d108.png

4.                              Поместите шину таким образом, чтобы ее борт оказался под пальцем монтажной головки (12) и вне ее опорного язычка (см. рис.)

ВНИМАНИЕ!

Если осуществляется монтаж бескамерной шины, начинать его следует с точки, в которой вентиль отстоит от монтажной головки на 180°.

5.                              Нажмите на педаль (1) вниз, чтобы стол вращался по часовой стрелке. При этом прижмите руками уже заведенную за закраину обода часть борта шины вниз, к середине обода.

6.                              Если осуществляется монтаж шины с камерой, вставьте камеру.

7.                              Операцию, изложенную в п. 4, повторите с верхним бортом шины.

8.                              Нажимая на педаль( 5), освободите колесо от зажимов.

Как при монтаже, так и при демонтаже стол должен двигаться по часовой стрелке. Вращение против часовой стрелки используется только при исправлении ошибок оператора станка.

 

НАКАЧИВАНИЕ ШИНЫ

ВНИМАНИЕ!

Трещина в ободе или разрыв шины под давлением может привести к взрыву, в результате которого колесо может с большой силой отскочить вверх или в сторону, чем вызвать серьезный ущерб и увечья вплоть до гибели оператора станка или окружающих! Несмотря на то, что этот станок оснащен клапаном безопасности, установленным на 3,5 бара, это не гарантирует полной безопасности и не устраняет риска взрыва в приведенных ниже случаях.

 

Порядок выполнения операции

1. Выверните золотник.

2. Наденьте клапан подкачки на штуцер и убедитесь, что он надежно зафиксирован.

3. Еще раз проверьте, совпадают ли посадочные диаметры шины и обода.

4. Убедитесь, что закраины обода и борта шины в достаточной степени обработаны смазкой. При необходимости провести дополнительную обработку.

5. Подавайте небольшие порции воздуха, постоянно контролируя давление с помощью манометра, до тех пор, пока борта шины не займут правильное положение на закраинах обода (обычно правильная посадка сопровождается громким хлопком). Проверьте плотность прилегания бортов шины к закраинам обода. При необходимости продолжите

процедуру накачивания.

6. Продолжайте закачивать воздух небольшими порциями, в перерывах между ними контролируя показания манометра, до тех пор, пока не будет достигнуто требуемое давление.(1,9атм.)



Технологическая карта

по теме: «Балансировка колес»

Общие сведения

Электронный балансировочный стенд с микропроцессором, способный балансировать колеса весом до 65 кг. Контрольные элементы и сигналы располагаются на передней панели. Система меню дает возможность работать с различными видами колес (автомобили и мотоциклы). Также доступны функции ALU для колес специальной формы или для предварительной настройки функций балансировочного стенда. Балансировочный стенд предназначен только для балансировки колес с максимальными размерами - 20¨.

Кнопки и индикаторы Контрольной панели

hello_html_1c4bf7a1.png

 

1. ИНДИКАТОР ДИСБАЛАНСА внутренней стороны.

2. ИНДИКАТОР ДИСБАЛАНСА внешней стороны.

3. Дисплеи, показывающие выбранную программу балансировки.

4. Кнопка ввода параметра РАСТОЯНИЕ

5. Кнопка ввода параметра ШИРИНА

6. Кнопка ввода параметра ДИАМЕТР

7. Кнопка запуска цикла

8. Кнопка аварийной остановки

9. Кнопка отмены

10. Кнопка оптимизации дисбаланса

11. Кнопка для выбора единиц измерения грамм/унция, мм/дюйм;

самокалибровка

12. Кнопка пересчёта

13. Кнопка выбора типа балансировки

14. Дисплей выбора программы «оптимизация дисбаланса»

15. Дисплей положения дисбаланса внешней стороны.

16. Дисплей положения дисбаланса внутренней стороны.

Примечание: нажимайте на кнопки только пальцами. Не используйте для этого

металлические и прочие твердые предметы.

 

Использование клавиатуры (пульта управления).

Используя клавиатуру пульта управления можно выполнять следующие действия:

1. Изменение параметра «расстояние» («а»)

2. Изменение параметра «ширина» («b»)

3. Изменение параметра «диаметр» («d»)

4. Отмена введенных параметров

5. Вывод на дисплей значений дисбалансов

6. Выбор программы балансировки (Dynamic, Static, ALUx)

7. Static Dynamic

8. ALUx Dynamic

9. Самокалибровка

10. Запуск закрытием кожуха

11. Единицы дисбаланса (грамм или унция)

12. Единицы ширины (мм или дюймы)

13. Единицы диаметра (мм или дюймы)hello_html_f92cca9.jpg

14. Оптимизация дисбаланса

15. Запуск измерительного цикла

16. Аварийная остановка двигателя

Перед балансировкой выставите правильные размеры для данного колеса

Ввод размеров колеса.

ДИАМЕТР: установите номинальный диаметр “d”, указанный на шине.

ШИРИНА: установите ширину диска, которая обычно указывается на ободе, или

используйте для ее измерения циркуль, входящий в комплектацию станка.

РАССТОЯНИЕ: определите расстояние между стендом и ободом колеса,

используя измеритель “a” (рис.4)

 

Балансировка колеса

- закройте кожух

- нажмите кнопку START

- колесо разгонится автоматически до нужной скорости, затем остановится.

Дисплеи 1 и 2 отобразят величину дисбалансов.

- дисплеи 3 и 4 отобразят положения измеренных дисбалансов. Если при свободном вращении колеса все светодиоды загорелись, балансировочный груз должен быть закреплен наверху вертикальной оси.

ВАЖНО: величина дисбаланса ниже 10-12 грамм (0,4-0,5 унций) – нормальная для хорошей балансировки. С данным балансировочным стендом легко проводить балансировку с допуском в 5 грамм (0,2 унции).

КНОПКА ВОЗВРАТА: <

Значение оставшегося дисбаланса ниже 5 грамм (0,2 унции) может быть отображено при необходимости нажатием этой кнопки.

Дисплеи 1 и 2 отображают «0» для дисбалансов ниже 5 грамм (0,2 унций) в нормальных условиях.

 

Самокалибровка.

- Закрепите на станке колесо среднего размера (желательно отбалансированное или с небольшим дисбалансом).

- введите с клавиатуры пульта управления параметры «расстояние», «ширина» и

«диаметр»

ВНИМАНИЕ: любая ошибка в настройке измерений будет означать, что калибровка проходит неверно, и все последующие действия будут неправильными пока не будут установлены правильные измерения.

- Нажмите одновременно кнопки «С» и START и удерживайте их нажатыми. На числовом дисплее внутреннего дисбаланса появится мигающая надпись CAL, которая указывает, что станок просит подтверждения проведения самокалибровки. Если Вы желаете продолжить процесс самокалибровки, нужно удерживать нажатыми кнопки «С» и START до тех пор, пока надпись CAL не

перестанет мигать. Если Вы отпустите одну из кнопок или обе кнопки во время входа в режим самокалибровки, операция не будет завершена. Когда надпись перестанет мигать, нажмите START чтобы начать первый цикл.

- По окончании первого цикла, на левом дисплее появится надпись “Add 100”. В то же время загорится дисплей внешнего дисбаланса. Теперь надо закрепить на внешней стороне обода балансировочный груз весом 100 г., в соответствии с индикатором 15.

- Нажмите START. Калибровка будет проводиться до конца цикла. После его окончания на дисплее будет показана надпись END CAL. Уберите вес в 100 грамм с колеса, он может быть использован для другого цикла. По окончании самокалибровки калибровочные данные автоматически сохраняются в памяти стенда до проведения следующей самокалибровки. Процесс самокалибровки может проводиться сколь угодно часто, но обязательно всякий раз, когда показания измерений вызывают сомнение в их правильности.

Функции “STATIC-ALU”.

(для балансировки колес мотоциклов, легкосплавных дисков или дисков

специальной формы).

1. NORMAL. Для балансировки колес с помощью пружинных грузов,

закрепляющихся на закраинах обода (обычно – для колес со стальными

штампованными дисками). При включении станка главным выключателем

компьютер автоматически выбирает эту программу балансировки.

hello_html_4a1ebc6d.png

2. ALU 1. Для балансировки колес с легкосплавными дисками с применением самоклеющихся грузов.

hello_html_m2e5071e5.png

3. ALU 2. Для «спрятанной» балансировки колес с легкосплавными дисками самоклеющимися грузами.

hello_html_7bb7a35e.png

4. ALU 3. Для комбинированной балансировки колес: пружинный груз на внешней стороне обода + самоклеящийся груз на внутренней стороне.

hello_html_42fad2e9.png

5. STATIC. Для балансировки узких (мотоциклетных) колес, или когда невозможно закрепить грузы ни в каком другом месте на ободе.

hello_html_m27d07733.png

ВНИМАНИЕ: ALU функции могут быть выбраны в любое время, чтобы определить веса балансировочных грузов по схемам, отличным от NORMAL. Нажатием кнопки А выбирается любая желаемая функция, при этом дисплеи дисбалансов отобразят реальные значения весов балансировочных грузов для каждой выбранной схемы балансировки.

МИНИМИЗАЦИЯ ДИСБАЛАНСА.

ВНИМАНИЕ:В процессе оптимизации дисбаланса «легкое» место шины совмещается с «тяжелым» местом диска, при этом уменьшается вес потребных балансировочных грузов.

1. Установить колесо на станок.

2. Ввести параметры «Расстояние», «Ширина» и «Диаметр».

3. Нажать кнопку «OPT» на пульте управления. На дисплее внутреннего дисбаланса «А» появится надпись «r s».

4. Закрыть защитный кожух – начнется измерительный цикл.

5. Когда измерительный цикл закончится дисплей внутреннего дисбаланса «A» выдаст инструкцию повернуть шину на ободе. Сделайте мелом метки на корпусе станка и на внутренней поверхности обода колеса напротив друг друга, это потребуется позже.

1. Снимите колесо со станка, сдуйте шину, разбортируйте ее, не снимая с диска, и поверните относительно диска на 180 градусов.

2. Накачайте шину и установите колесо на станок по меловым меткам на корпусе станка и на ободе так, как оно стояло прежде. Закройте защитный кожух, начнется второй измерительный цикл.

3. По окончании измерительного цикла на дисплее внешнего дисбаланса «В» отобразится процент изменения статического дисбаланса по сравнению с первоначальным (до поворота шины на диске). В то же время на дисплее внутреннего дисбаланса отобразится текущее значение статического

дисбаланса. Эта величина (вес) может быть изменен на процент, отображенный дисплеем внешнего дисбаланса.

4. Вручную поворачивайте колесо, пока не загорятся все светодиоды «О» положения внешнего дисбаланса. В этой позиции отметьте мелом вершину шины. Поворачивайте далее колесо, пока не загорятся все светодиоды «Р» положения внутреннего дисбаланса. В этой позиции отметьте мелом вершину обода

5. Нажать «STOP» чтобы закончить процесс оптимизации дисбаланса.

6. Оптимизация дисбаланса достигается совмещением меток на вершинах шины и обода (еще один поворот спущенной шины на ободе). Теперь провести стандартный процесс динамической балансировки.

 

Практическая работа №15

 

Цель: Сформировать практические навыки по регулировке гидроусилителя рулевого управления. Закрепить теоретические знания по устройству рулевых управлений с гидроусилителем тракторов и автомобилей.

 

 

Теоретическая часть

 

При повороте рулевого колеса, вправо золотник 14 (рис.1, б) отключает полость Б цилиндра от бака и соединяет с насосом 6, Под давлением жидкости перемещается поршень-рейка и зубчатый сектор с валом 16 рулевой сошки. Аналогично этому, когда рулевое колесо вращается влево, давление создается в полости А (рис. 1, в) цилиндра. При повороте для снижения противодавлений в противоположных полостях цилиндра они сообщаются с баком.

Гидроусилитель начинает действовать в тот момент, когда сопротивление колес повороту создает на гайке рулевого механизма реактивное усилие, превышающее силу предварительного сжатия пружин и давления масла на плунжеры 13, которые стремятся удержать винт 15 рулевого, механизма в нейтральном положении.

Источником энергии для гидроусилителя служит роторный насос 6. При неработающем насосе рулевой механизм работает без гидроусилителя, так как шариковый клапан 12 соединяет линии давления и слива.

Рулевой механизм соединен с рулевым валом карданным валом, с двумя шарнирами. Насос гидроусилителя приводится в действие при помощи клиновидного ремня от шкива коленчатого вала.

hello_html_7d9aa205.png

Рис. 1. Схема работы гидроусилителя рулевого привода автомобиля

а — нейтральное положение; б, в — перемещение цилиндра соответственно вправо и влево; 1 — сапун; 2, 3 — фильтры; 4, 7 — перепускные клапаны; 5 — коллектор; 6 — насос; 8 — предохранительный клапан; 9, 10, 11 — отверстия; 12 —шариковый клапан; 13 — реактивный плунжер; 14 —золотник; 15 — винт; 16 — вал сошки; 17 — картер; А, Б — полости.

На рисунке 2 показана схема гидроусилителя рулевого управления с приводом механизма автоматической блокировки дифференциала.

При движении трактора прямо (нейтральное положение золотника) усилие на червяке 13 (рис. 2; а) отсутствует или недостаточно для сжатия пружин 10 ползунов 9; тогда пружины 10 удерживают, золотник 6 в среднем, нейтральном положении и гидроусилитель отключен. Между рулевым колесом 16 и управляемыми колесами 14 существует только механическая связь. Масло из бака 2 насосом 3 подается через нагнетательный трубопровод 4 и выточки золотника 6 в обе полости А и Б цилиндра 24, а также по маслопроводу 25 через фильтр 1 в бак 2.

При повороте трактора вправо (рис. 2, б) и влево (рис. 2, в) на червяк 13 действует усилие, превышающее силу предварительного сжатия пружин 10, центрирующих золотник 6, поэтому внутренние шайбы упорных шарикоподшипников 5 перемещают золотник б в осевом направлении. Поток масла, поступающий из насоса 3 в нагнетательный маслопровод 4, может попасть только в одну из полостей гидравлического цилиндра 24; при правом повороте (рис. 2, б) через заднюю выточку золотника и в полость Б; при левом повороте (рис. 2, в) через переднюю выточку золотника и в полость А. Давление масла в цилиндре перемещает поршень 26 со штоком 23 и рейкой 22. Далее усилие передается через сектор 17, поворотный вал 19 и сошку 18 на рулевую трапецию и колеса 14. Предохранительный клапан 12 ограничивает давление в системе до 8 МПа.

Через золотник датчика 21 автоматической блокировки дифференциала жидкость из насоса 3 по масдопроводам 25 может поступать в камеру муфты сцепления механизма блокировки дифференциала. Это зависит от положения крана, находящегося слева на корпусе гидроусилителя. Кран имеет две позиции; «выкл» и «вкл» (блокировка выключена и включена). Дифференциал автоматически разблокируется при повороте колес на угол, больший 8°.

hello_html_514abf9d.png

Рис. 2. Схема работы гидроусилителя рулевого управления:

а — нейтральное положение; б, в — поворот соответственно вправо и влево; 1 — фильтр; 2—бак; 3 — насос; 4, 25 — маслопроводы; 5 — гайка; 6 — золотник; 7 — крышка; 8 — шарикоподшипник; 9 — ползун; 10—пружина; 11 — корпус; 12 — предохранительный клапан; 13 — червяк; 14 — управляемое колесо; 15 — регулировочная втулка; 16 — рулевое колесо; 17 — сектор; 18 — сошка;19 — вал; 20 — муфта механизма блокировки дифференциала; 21 — датчик; 22—рейка; 23 — шток; 24—цилиндр; 26 — поршень; 27 — корпус; А, Б — полости; К — камера.

Гидроусилитель состоит из гидронасоса 8 (рис. 3) марки НШ-50, клапана расхода 7 и предохранительного клапана 6, рулевого механизма 4, распределителя 13 с червяком 11 и золотником 12, запорных клапанов 2, гидравлических цилиндров 1, бака 9, тяги обратной связи 16, рулевого колеса 15 и трубопроводов для соединения гидравлических узлов. Масло, залитое в бак 9, засасывается насосом 8 и через клапан расхода 7 подается к распределителю 13 рулевого механизма 4. В зависимости от положения золотника 12 распределителя 13 масло сливается в бак или через запорные клапаны 2 поступает в одну, из полостей цилиндра 1.

hello_html_25619a28.png

Рис. 88. Схема рулевого управлений:

1— гидроцилиндр; 2 — запорный клапан; 3 —сошка; 4 — рулевой механизм; 5 — сектор; 6— предохранительный клапан; 7 — клапан расхода; 8—. гидронасос; 9 — бак; 10 — заливная горловина; 11 — червяк; 12 — золотник; 13 — распределитель? 14 — колонка; 15 — рулевое колесо; 16 — тяга обратной связи.

 

Задание.

1. На автомобиле рассмотрите место установки рулевого управления.

2.Проведите регулировки гидроусилителя.

2. Ответить на контрольные вопросы.

 

Порядок выполнения работы.

Непосредственно на автомобиле рассмотрите место установки агрегата рулевого управления. Пользуясь плакатами, разрезами и схемами, изучите устройство гидроусилителя рулевого управления (рис. 1).

Зазор в зацеплении поршень-рейка и сектора отрегулируйте смещением в осевом направлении вала сошки. При сборке рулевого мехнизма в винтовые канавки гайки, винта, и желобы заложите шарики.

Для золотника 14 отверстие в корпусе клапана управления выполнено большей длины, вследствие чего золотник 14 и винт 15 могут перемещаться в осевом направлении на 1 мм в каждую сторону от среднего положения. Шесть реактивных пружин и плунжеров 13 удерживают золотник 14 в среднем положении.

Когда автомобиль движется прямо, чему соответствует нейтральное положение золотника (рис. 1, а), полости А и Б цилиндра соединены с насосом 6 и его баком.

Проведите регулировки гидроусилителя.

При регулировке осевого разбега вала заверните до упора регулировочный винт, предварительно ослабив контргайку, затем отверните на 1/8...1/10 оборота и законтрите его контргайкой.

Для регулировки зацепления червяк 13 — сектор 17 ослабьте болты крепления регулировочной втулки 15, поверните втулку по часовой стрелке до упора и затем против часовой стрелки на 5...6 мм по наружному диаметру фланца втулки. Затяните болты крепления. Запустите двигатель и убедитесь в отсутствии заеданий при повороте рулевого колеса в обе стороны до упора. При необходимости увеличьте зазор в зацеплении, поворачивая втулку против часовой стрелки до тех пор, пока не будет исключено заедание.

Регулировку зацепления сектор 17 — рейка 22 проведите путем удаления регулировочных прокладок под фланцем упора до получения зазора между упором и рейкой, равного 0,1...0,3 мин.

Затяните сферическую гайку 5 (рис.2) червяка моментом 0,2 МПа, затем отверните на 1/10... 1/12 оборота и зашплинтуйте.

Изучите устройство гидроусилителей других тракторов.

 

Контрольные вопросы

1. Назовите детали рулевого управления автомобиля.

2. Расскажите об устройстве и принципе действия гидроусилителя рулевого привода автомобиля.

3. Для какой цели служит гидроусилитель рулевого управления?

4. Опишите устройство, работу и регулировки гидроусилителя.

 

 

Практическая работа №16

Цель: Сформировать практические навыки по регулировке тормозных систем с пневмоприводом. Закрепить теоретические знания по устройству тормозных систем с пневмоприводом.

 

Теоретическая часть

Тормозные механизмы системы барабанного типа с двумя внутренними колодками, диаметр тормозных барабанов 420 мм, ширина накладок 180 мм. Передние тормозные камеры — диафрагменные, типа 30, задние — типа 30/24.

Привод рабочих тормозных систем — пневматический, раздельный. Количество ресиверов 6, общим объемом 120 л.

Номинальное давление в пневмоприводе (6,5—8,0 кгс/ см2).

Давление сжатого воздуха в пневмоприводе регулируйте винтом 2 регулятора давления (см. рис. Регулятор давления). При вворачивании винта величина регулируемого давления увеличивается, при выворачивании — уменьшается.

hello_html_m5ab7708b.jpg   hello_html_505288db.jpg

В тормозной системе автомобиля установлен адсорбентный осушитель воздуха 3 (см. рис. Регулятор давления). Осушитель выполнен совместно с регу-лятором давления и предназначен для охлаждения, выделения конденсата и поддержания требуемого давления сжатого воздуха поступающего от компрессора. Подаваемый от компрессора в осушитель сжатый воздух проходит через фетровый диск и гранулант, очищается и попадает дальше в тормозную систему. После заполнения тормозной системы и срабатывания регулятора давления происходит очистка грануланта от влаги воздухом, выходящим в атмосферу, через атмосферный вывод осушителя. Техническое обслуживание осушителя заключается в периодической замене фильтрующего элемента по мере его загрязнения (примерно раз в два года).

Для накачки шин на регуляторе давления имеется клапан отбора воздуха, закрытый колпачком 1 (см. рис. Регулятор давления). При отборе воздуха шлангом ля накачки шин из комплекта инструментов подсоедините его вместо колпачка, навернув до упора гайку-барашек, и понизьте давление сжатого воздуха в пневмоприводе, потому что при холостом ходе компрессора отбора воздуха нет. Для снижения давления откройте кран слива конденсата на любом ресивере. Конденсат из ресиверов сливайте ежедневно по окончании работы. Давление сжатого воздуха в пневмоприводе при этом должно быть номинальным.

Краны слива конденсата откройте, отведя в сторону толкатель за кольцо (см. рисунок). Не тяните шток вниз и не нажимайте его вверх. После слива конденсата доведите давление сжатого воздуха в пневмоприводе до номинального.

Задание.

1. Отрегулировать свободный ход педали тормоза.

2.Проверить работу компрессора и тормозного крана.

2. Ответить на контрольные вопросы.

 

Порядок выполнения работы.

 

Управление рабочими тормозными системами автомобиля  осуществляется двухсекционным краном с приводом от педали.

Положение тормозной педали относительно пола кабины регулируйте согласно Схеме установки педали на тормозной кран. Регулировкой установочного и регулировочного болтов необходимо обеспечить положение площадки педали под углом 35о ±2о и свободный ход педали 10-15 мм. Установочный болт зафиксировать контргайкой, регулировочный болт перед регулировкой покрыть герметиком УГ7.

hello_html_m33984057.png

Обслуживание двухсекционного тормозного крана заключается в его периодическом осмотре, очистке от грязи, проверке герметичности и работы. Необходимо следить за состоянием защитного резинового чехла крана и плотностью прилегания его к корпусу, так как при попадании грязи на рычажную систему и трущиеся поверхности тормозной кран выходит из строя. Герметичность тормозного крана проверяют с помощью мыльной эмульсии в двух положениях: в заторможенном и расторможенном. Утечка воздуха через атмосферный вывод тормозного крана при этих положениях свидетельствует о том, что в одной из секций либо нарушилась герметичность впускного клапана, либо вышел из строя выпускной клапан. Кран с такими дефектами необходимо заменить. Тормозной кран срабатывает полностью при усилии на рычаге 80 кгс и ходе рычага 26 мм. Начальная нечувствительность крана примерно 15 кгс. Разность давления в секциях крана может составлять до 25 кгс/см2.

Обслуживание привода тормозного крана заключается в периодическом осмотре, очистке и смазке шарнирных соединений. Следует проверить состояние защитного чехла (он не должен иметь разрывов) и убедиться в том, что он плотно прилегает к корпусу тормозного крана по всему периметру. Необходимо следить за состоянием кронштейнов, а также тяг и рычагов, связывающих тормозную педаль с тормозным краном, периодически очищать их от грязи и посторонних предметов (веток, проволоки и т. д.). Полностью нажатая педаль тормоза не должна доходить до пола на 10—30 мм. Полный ход ее должен быть в пределах 100—130 мм, а свободный 20—30 м. В случае необходимости следует отрегулировать ход педали тормоза, изменяя с помощью регулировочной вилки длину тяги, соединяющей педаль с первым промежуточным рычагом привода. Если по каким-либо причинам разбирался привод тормозного крана, то при сборке надо добиться совмещения нижнего отверстия промежуточного рычага с осью опрокидывания кабины. Затем, изменяя длину тяги, идущей от педали к переднему рычагу, установить педаль в требуемое положение по отношению к полу кабины.

При обслуживании компрессора необходимо проверять затяжку гаек его крепления к двигателю, затяжку гаек шпилек, крепящих головку, и других крепежных деталей. Гайки шпилек, крепящих головку, следует затягивать равномерно, в два приема. Окончательный момент затяжки должен быть в пределах 1,2— 1,7 кгс-см2.Через 80 000—100 000 км пробега при сезонном обслуживании (весной) надо снимать головку компрессора для очистки поршней, клапанов и седел. Клапаны, не обеспечивающие герметичности, необходимо притереть к седлам, а сильно изношенные или поврежденные заменить новыми. Новые клапаны также следует притереть к седлам (до получения непрерывного кольцевого контакта при проверке «на краску»). Признаками неисправности компрессора являются появление шума и стука при его работе, увеличенное количество масла в конденсате, сливаемом из воздушных баллонов. Последнее обычно является следствием износа поршневых колец, масляного уплотнения заднего торца коленчатого вала или подшипников нижних головок шатунов.

Контрольные вопросы

1.Какой принцип работы тормозной системы с пневмоприводом?

2.Какие операции включает обслуживание привода тормозного крана?

3.Как определяют неисправности компрессора?

 

Практическая работа №17

Цель: Сформировать практические навыки по удалению воздуха из гидравлических систем. Закрепить теоретические знания по регулировке тормозных систем с гидроприводом.

 

Теоретическая часть

Тормозная система предназначена для управляемого изменения скорости автомобиля, его остановки, а также удержания на месте длительное время за счет использования тормозной силы между колесом и дорогой. Тормозная сила может создаваться колесным тормозным механизмом, двигателем автомобиля (торможение двигателем), гидравлическим или электрическим тормозом-замедлителем в трансмиссии. Для уменьшения скорости движения, остановки и удержания в неподвижном состоянии автомобили оборудуют тормозной системой, состоящей из тормозного механизма и привода.                   
Для торможения автомобиля во время движения, как правило, пользуются рабочим тормозом: при нажатии на педаль колодочные механизмы одновременно действуют на все четыре колеса. На стоянках или остановках пользуются стояночным тормозом.                     
Безопасность движения автомобилей с высокими скоростями в значительной степени определяется эффективностью действия и безопасностью тормозов.       
         Эксплуатация любого автомобиля допускается в том случае, если он имеет исправную тормозную систему. Тормозная система необходима на автомобиле для снижения его скорости, остановки и удерживания на месте.  Тормозная сила возникает между колесом и дорогой по направлению, препятствующему вращению колеса. Максимальное значение тормозной силы на колесе зависит от возможностей механизма, создающего силу торможения, от нагрузки, приходящейся на колесо, и от коэффициента сцепления с дорогой. При равенстве всех условий, определяющих силу торможения, эффективность тормозной системы будет зависеть в первую очередь от особенностей конструкции механизмов, производящих торможение автомобиля.

Тормозная система служит для снижения скорости автомобиля, его остановки и удержания на месте на стоянке. Тормозное управление является важнейшим средством обеспечения безопасности автомобиля. К нему предъявляют следующие требования: минимальный тормозной путь, сохранение устойчивости при торможении, стабильность тормозных свойств при неоднократных торможениях, минимальное время срабатывания тормозного привода, малое усилие на тормозной педали при ее ходе 80-180 мм, надежность всех элементов тормозной системы. Основные элементы должны иметь гарантированную прочность, не должны выходить из строя на протяжении гарантированного ресурса, время срабатывания тормозного привода должно быть минимальным, между усилием на педаль и приводным моментом должна быть пропорциональность, о неисправности тормозной системы должна оповещать сигнализация.
Тормозное управление автомобиля должно включать рабочую, запасную, стояночную и вспомогательную тормозные системы:
 - рабочую тормозную систему, которая приводится в действие нажатием ноги водителя на педаль ножного тормоза, служит для снижения скорости движения автомобиля вплоть до полной его остановки вне зависимости от его скорости, нагрузки и уклонов дороги. Рабочая тормозная система обладает наибольшей эффективностью из всех типов тормозных систем;
- запасная тормозная система предназначена для остановки автомобиля в случае отказа основной рабочей системы. Она обладает меньшим тормозящим действием, чем рабочая система;
- стояночная тормозная система служит для удержания остановленного автомобиля на месте, чтобы исключить его самопроизвольное движение. Управляется стояночная тормозная система через рычаг ручного тормоза; 
-вспомогательная система тормозов предназначена для поддержания постоянной скорости автомобиля, при движении его на затяжных спусках горных дорог, с целью снижения нагрузки на рабочею тормозную систему при длительном торможении.                             
 Каждая тормозная система состоит из тормозных механизмов, которые обеспечивают затормаживание колес или вал трансмиссий, и тормозного привода приводящего в действие тормозной механизм. Тормозной механизм может быть колесный, трансмиссионный, барабанный и дисковый. На автомобиле ГАЗ-3307 рабочая тормозная система - с барабанными механизмами диаметр 380мм, ширина передних накладок 80, задних - 100 мм, двухконтурным гидравлическим приводом (раздельный по осям), гидровакуумным усилителем. Стояночный тормоз - трансмиссионный барабанный (диаметр 220 мм, ширина накладок 60 мм), с механическим приводом. Запасной тормоз - любой из контуров рабочей тормозной системы.     

 

Задание.

1. Удалить воздух из гидравлической системы.

2. Ответить на контрольные вопросы.

 

Порядок выполнения работы.

Удаление воздуха из системы гидропривода. Удаление воздуха из системы гидропривода, ее прокачку производят при профилактической замене жидкости, при попадании воздуха в гидропривод, а также после проведения работ, связанных со сливом жидкости из тормозных узлов. Попавший в гидропривод воздух значительно снижает эффективность рабочей тормозной системы. На наличие воздуха в приводе тормозов указывает увеличенный ход педали тормоза и «мягкость» хода педали. Для прокачки гидропривода необходимо: 
-  очистить клапаны выпуска воздуха на узле, из которого будет удаляться воздух, от пыли и грязи; 

-  проверить, а при необходимости залить в бачок тормозную жидкость; 
-  снять колпачок с клапана выпуска воздуха и надеть на его головку резиновый или пластмассовый шланг для слива жидкости; 
-  погрузить конец шланга в частично заполненную тормозной жидкостью чистую прозрачную емкость; 

-  резко нажать 5 раз на тормозную педаль с интервалами 2 с, а затем, оставляя педаль нажатой, отвернуть на пол-оборота клапан выпуска воздуха; при этом в вытекающей из шланга жидкости будут видны пузырьки воздуха. 
После того как вытекание жидкости из шланга прекратится, завернуть плотно клапан выпуска воздуха и отпустить тормозную педаль. 
Удаляя воздух из гидропривода, необходимо добавлять тормозную жидкость в бачок, следя за тем, чтобы уровень в нем не опускался ниже минимальной отметки. Необходимо помнить, что каждый контур гидропривода в бачке имеет свою полость. Для доливки в бачок жидкости нельзя использовать жидкость, слитую при прокачке, без ее предварительного фильтрования. 
Далее нужно снять шланг, вытереть конец клапана и надеть на него колпачок. Эти операции повторяют на всех тормозных узлах, удаляя из них таким образом воздух. 

Контрольные вопросы

1.Для чего предназначены тормозные системы?

2.Какая принципиальная разница между гидравлическими и пневмотическими тормозными системами?

3.Как удалить воздух из гидросистемы?

 

Практическая работа №18

Цель работы:

Цель: Сформировать практические навыки по ремонту кузова, кабины и оперения.

 

Теоретическая часть

Кузова легковых автомобилей подразделяются по конструктивному исполнению на два основных вида: рамные и безрамные. В первом случае на жесткое основание - раму - крепятся двигатель, трансмиссия, подвеска и сам кузов. Кузов, таким образом, не является несущим. Второй тип кузова - безрамный - называют также модульным. Он состоит из коробчатых жестких конструкций, которые, в свою очередь, собираются из тонких листов металла (1-2 мм) с помощью контактной сварки. Соединение таких элементов с помощью той же сварки дает несущий кузов. Места крепления двигателя, подвески и других тяжелых агрегатов могут усиливаться наваркой пластин, ребер и штамповкой объемных профилей на самом листе.

Для перехода от рамных кузовов к несущим есть несколько причин. Среди них и облегчение конструкции в целом. Немаловажной причиной является необходимость повысить безопасность пассажиров на случай столкновения. Коробчатые конструкции кузова, прежде чем передать энергию удара дальше, сминаются сами и поглощают существенную часть этой энергии. Таким образом, безопасность пассажиров значительно повышается. Для обеспечения пассажирам "пространства выживания" внутри салона усиливают пол кабины, центральные стойки, устанавливают продольные штанги в дверях.

На рис. 1.1 вы можете видеть основу кузова современного легкового автомобиля. Видны элементы усиления в полу кабины, в зоне крепления двигателя и передней подвески, а также в зоне багажника и задней подвески. Кроме того, становится понятно, какие детали кузова входят в основу, а какие являются навесными: навесные на рисунке отсутствуют.

 

https://fsd.multiurok.ru/html/2020/11/02/s_5f9faa732663d/1554670_1.png




Рис. 1.1. Основа кузова легкового автомобиля

 

Как правило, даже при несущественной деформации усиленных элементов, а также при смещении опорных ниш стоек подвески, эксплуатационные характеристики автомобиля существенно ухудшаются - неравномерно и ускоренно изнашиваются покрышки, ухудшается управляемость. Элементы усиления крайне сложно вернуть в исходное положение, а если это удается, то с помощью замены поврежденных деталей на новые или с использованием дорогого и сложного оборудования.

В любом случае необходимо уметь установить смещение деталей и определить границу повреждения. Далее можно срезать поврежденные детали и приварить новые, но главное правильно выбрать места резки и последующего соединения - лучше делать это в местах заводского соединения деталей, а не на середине сплошного листа. На рис. 1.2а вы можете увидеть собранное соединение лонжерона с брызговиком и поперечной передней балкой, на рис. 1.2б эти детали показаны отдельно. На основе этого примера можно понять, как следует отделять поврежденные детали с наименьшим ущербом для кузова. Для подобного разделения достаточно высверлить или разрубить точки контактной сварки.

https://fsd.multiurok.ru/html/2020/11/02/s_5f9faa732663d/1554670_2.jpeg https://fsd.multiurok.ru/html/2020/11/02/s_5f9faa732663d/1554670_3.jpeg 

Так как форму деталей, составляющих кузов, невозможно описать, данная работа является хорошей возможностью для получения необходимых знаний о конструкции кузова.

 

Ход работы:

 

https://fsd.multiurok.ru/html/2020/11/02/s_5f9faa732663d/1554670_5.jpeg

Рис. 1.3. Элементы основы кузова

Работа проводится на основе кузова легкового автомобиля со снятыми навесными элементами. Задача состоит в том, чтобы, пользуясь рисунком 1.3, найти все элементы, из которых состоит основа кузова, дать им названия, и заполнить табл. 1.

 

- центральный брызговик;

- боковые панели крыши;

- скобка передка;

- рамка задней перегородки;

- нижняя панель передка;

- панель задка (стенка багажника);

- боковой брызговик бампера;

- нижняя поперечина задка;

- кожух фары;

- заднее крыло;

- верхняя поперечина передка;

- лонжероны багажника;

- усилитель поперечины;

- внутренние арки задних колес;

- площадка аккумулятора;

- пол багажника;

- щиток передка (задняя стенка капота);

- поперечина пола багажника;

- коробка воздухопритока;

- задний пол;

- боковины кузова;

- передний пол;

- рама ветрового стекла;

- усилитель боковины;

- нижняя поперечина приборной панели;

- стойки передней подвески;

- крыша;

- брызговики передних крыльев;

- панель заднего окна;

- передние крылья.

 

Следует учитывать, что не все детали, присутствующие на рисунке, есть в исследуемом кузове, и не все детали кузова есть на рис. 1.3. Для некоторых деталей можно подобрать более удачные названия, следуя логике примера.

 

Таблица 1

Название детали

Соединяется с номерами

Виды сварки на соединениях

 

 

В таблице следует заполнять сначала первые две колонки, то есть дать деталям названия и присвоить номера. Затем заполняются колонки соединений. Достаточно научиться отличать два вида соединений:

  • соединение объемных деталей с помощью точек контактной сварки на разных гранях детали. Такое соединение обычно проще разобрать путем высверливания точек сварки;
  • соединение плоских кромок с помощью ряда близкорасположенных точек сварки. При таком соединении обычно спиливают или срубают поврежденную деталь по линии около места наложения, а её остатки на соседней кромке стачивают точильным кругом.

 

Отчет включает общее описание кузова, цель работы и таблицу.

 

Контрольные вопросы:

  1. Материалы, используемые при изготовлении кузовов.
  2. Технология заводской сборки кузовов.
  3. Основные детали кузова и их назначение.
  4. Особенности строения современных кузовов.
  5. Способы соединения деталей кузова при сборке.
  6. Способы разборки соединений точечной сварки.
  7. Конструкция защитных элементов (бамперов).
  8. Шумовая изоляция современных кузовов.
  9. Классификация кузовов по замкнутым объемам.
  10. Антигравийные и антикоррозионные покрытия кузовов и места их нанесения.

 

Практическая работа №19

Цель: Сформировать практические навыки по испытанию автомобиля после ремонта.

 

Теоретическая часть

Автомобиль испытывают пробегом на рас­стояние 30—50 км со скоростью не более 40— 50 км/ч с грузом в 75% от номинальной грузо­подъемности. Перед пробегом двигатель про­гревают до температуры ОЖ в системе ох­лаждения не менее 60°С. Прогретый двигатель должен запускаться стартером, устойчиво ра­ботать на малых оборотах холостого хода и равномерно увеличивать обороты при открытии дросселя.

Во время испытания автомобиля наблюда­ют за работой всех его агрегатов. Двигатель при трогании с места должен работать без рез­кого повышения числа оборотов. Сцепление должно легко выключаться и полностью разъ­единять двигатель от ведущего вала коробки передач, обеспечивать бесшумное и плавное трогание автомобиля с места. Буксование сцепления во время разгона не допускается.

Легкое и бесшумное переключение передач после небольшой выдержки при выключенном сцеплении указывает на хорошее качество ре­монта. Самовыключения шестерен не должно быть.

Во время движения автомобиля температу­ра воды в радиаторе не должна превышать 80°С, а температура масла при включенном масляном радиаторе — 100°С. В коробке пере­дач и заднем мосту может наблюдаться равно­мерный шум, но без стуков. Температура мас­ла в коробке передач не должна превышать 70°С. Не допускаются вибрации и стуки кар­данных валов.

Проверяется работа рулевого механизма. Механизм должен действовать легко, без зае­даний, обеспечивая полный разворот в обе сто­роны. При этом покрышки не должны задевать за продольную рулевую тягу или раму автомо­биля.

Во время испытания проверяется тормозная система. Она должна обеспечить равномерное торможение при плавном приложении усилия к тормозной педали или рычагу ручного тор­моза. При полном торможении педаль или ры­чаг не должны доходить до упора. Тормоза должны работать без шума, тормозные бараба­ны и ступицы колес не должны нагреваться. Определяется путь торможения автомобиля, который должен составлять примерно 10 м на горизонтальном участке сухой дороги с твер­дым покрытием при скорости движения 30 км/ч. Ручной тормоз без дополнительных приспособлений должен удерживать автомо­биль на месте неограниченное время на укло­нах не менее 25% при сухом дорожном покры­тии.

Во время движения автомобиля не допус­кается самопроизвольное открытие дверей ка­бины, стекол, застежек капота, запоров бортов платформы, а также дребезжание крыльев ка­пота, глушителя и других деталей.

Не допускается подтекание смазки, топлива и воды, а также пропуск газов через все соеди­нения.

Должны безотказно и с надлежащей точ­ностью работать контрольные приборы: указа­тель давления масла, амперметр, указатель уровня топлива, спидометр, а также переклю­чатель света, сигнал, стеклоочиститель и т. п.

Испытание пробегом прекращают, если об­наружены неисправности, которые угрожают безопасности движения, сохранности агрега­тов или мешают проверке работы автомобиля. После устранения неисправностей автомобиль вновь испытывают пробегом. При замене дви­гателя испытания автомобиля полностью по­вторяют, а при замене коробки передач или заднего моста пробег автомобиля составляет 15 км с нагрузкой, равной 75% номинальной грузоподъемности.

Затем автомобиль тщательно осматривают. Все выявленные пробегом и осмотром неис­правности и дефекты должны быть устранены, а наружные крепления подтянуты. После этого автомобиль окончательно окрашивают и предъявляют работникам отдела технического контроля (ОТК) для проверки комплектности и качества ремонта.

Заполняется технический паспорт отремон­тированного автомобиля и двигателя, а также составляется акт технического состояния авто­мобиля. Качество выполненных работ по авто­мобилю и агрегатам должно соответствовать техническим условиям на капитальный ремонт.

Принятый ОТК автомобиль выдается из ре­монта представителем авторемонтного пред­приятия по приемо-сдаточному акту в соответ­ствии с техническими условиями на сдачу в капитальный ремонт и выдачу из капитального ремонта автомобиля. Автомобиль должен иметь все колеса (кроме запасного) с накачен­ными и годными к эксплуатации шинами.

Капитально отремонтированный автомо­биль должен обеспечить нормальную работу до следующего капитального ремонта, который устанавливается по пробегу не менее 80% от нормы для новых автомобилей и агрегатов. Нормы пробега автомобиля рассмотрены ра­нее

Контрольные вопросы

1. Расскажите об укрупненном технологическом про­цессе сборки автомобиля

2. Как испытывают отремонтированный автомобиль?

 

Практическая работа №20

Цель: Сформировать практические навыки по основам технологии сборки автомобиля.

 

Теоретическая часть

Сборка автомобилей из отремонтированных сборочных единиц ведется в следующем порядке.

1.                              Ставят раму на специальные подставки (или тележку), подкатывают под нее задний и передний мосты в сборе с рессорами и присоединяют к раме.

2.                              Устанавливают и закрепляют рулевой механизм (без рулевого колеса). Ставят тормозные воздухопроводы или гидропроводы.

3.                              Монтируют двигатель в сборе с коробкой передач и карданные валы. Соединяют выпускную трубу в сборе с глушителем, затем радиатор, бензобак, кабину, крылья.

4.                              Монтируют провода, приборы электроосвещения, зажигания и сигнализации, рулевое колесо.

5.                              Устанавливают ходовые колеса. Прокачивают и регулируют тормоза, регулируют рулевое управление, механизм управления двигателем.

6.                              Смазывают машину в соответствии с инструкцией, заправляют водой, топливом, обкатывают, испытывают, окрашивают и сдают заказчику.

7.                              процесс сборки несколько иной. На раму, перевернутую на 180° и установленную на подставках, монтируют задний мост в сборе, передний мост и рулевое управление, тормозные воздухопроводы или гидропроводы. После этого раму поднимают электротельфером и ставят в нормальное положение. Далее сборку продолжают в указанной выше последовательности.

8.                              Все поверхности сборочных единиц и деталей, поступающих на линию сборки тракторов или автомобилей, должны быть окрашены эмалевой краской, кроме впускных и выпускных коллекторов двигателей, полов кабины, рычагов, педалей, кронштейнов, фар, сеток облицовки, защелок и ручек, которые окрашивают в черный цвет другими красками. Наружную окраску всей машины проводят после обкатки и устранения дефектов. Качество ремонтных лакокрасочных покрытий не должно уступать новым.

 

Основы технологии сборки автомобиля

Технологический процесс сборки разных моделей автомобилей определяется их конструкцией, но общая последовательность сборки примерно одинакова.

Рассмотрим в укрупненном виде технологический процесс сборки грузового автомобиля ЗИЛ-130. Сборка заключается в установке на раму автомобиля в определенной последовательности собранных, испытанных и окрашенных узлов и агрегатов. Раму располагают так, чтобы горизонтальные нижние полки продольных балок находились в верхнем положении. Устанавливают и закрепляют обоймы и дополнительные буферы передних и задних рессор, тягу соединения двигателя с рамой в сборе с чашками и буферами.

Затем устанавливают передний и задний мосты в сборе с рессорами так, чтобы совместились отверстия передних ушек рессор и кронштейнов, вставляют и закрепляют пальцы. Задние концы рессор устанавливают накладками «а сухари кронштейнов, вставляют втулки, совмещают отверстия вкладышей, втулок, устанавливают стяжные болты, пружинные шайбы и закрепляют гайки.

Следующей операцией является установка и закрепление амортизаторов передней подвески. В гнездо кронштейнов задней опоры двигателя вставляют опорные подушки.

Устанавливают и закрепляют воздушные баллоны, тормозной кран, пневматические трубопроводы, соединяя их с тормозным краном, воздушными баллонами и тормозными камерами передних и задних колес.

Размещают и закрепляют карданную передачу (основной и промежуточный карданные валы), закрепляют на раме брызговики двигателя, глушитель, амортизатор и приемные трубы глушителя. Краном поднимают раму с установленными агрегатами и переворачивают, опуская на деревянные подкладки под передней и задний мосты. Устанавливают буксирный прибор в сборе, соединяют с трубопроводами гибкие шланги тормозных камер передних и задних колес. На поперечину рамы устанавливают и закрепляют разобщительный кран, соединив его с тормозным краном.

На раме устанавливают и закрепляют кронштейн вала педали сцепления, надевают рычаг и закрепляют его болтом, подложив под головку болта пружинную шайбу. На вал педали надевают рычаг управления тормозным краном и устанавливают вал в отверстие кронштейна. На наружный конец вала надевают педаль привода сцепления, предварительно вставив шпонку, и закрепляют болтом.

Устанавливают и закрепляют рулевой механизм с гидроусилителем, соединяют передний мост с рулевым механизмом продольной рулевой тягой, вставив в отверстие поворотного рычага и сошки вала рулевого механизма шаровые пальцы, и закрепляют их гайками. На передние концы продольных балок рамы устанавливают и закрепляют передний буфер, усилители и буксирные крюки, брызговики облицовки радиатора.

Затем прокладывают пучки проводов, закрепляют их скобами, а соединительную панель крепят к четвертой поперечине рамы. Аккумуляторную батарею устанавливают в гнездо, присоединяя соответствующие провода.

Устанавливают, закрепляют на раме двигатель в сборе со сцеплением и коробкой передач. К выпускному трубопроводу прикрепляют приемные трубы глушителя. Устанавливают и закрепляют первую трубку от крестовины тормозного крана к клапану регулятора давления и трубку от компрессора к первому воздушному баллону. Колено подводящего патрубка радиатора соединяют со шлангами и сливным краником, а затем устанавливают и соединяют хомутами с патрубком водяного насоса. Тягу от промежуточного рычага тормозного крана закрепляют с рычагом привода ручного тормоза, отрегулировав длину тяги и обеспечив зазор, равный 1,0 мм, между пальцем и скобой тяги. Рычаг педали сцепления соединяют с рычагом вилки включения сцепления, отрегулировав свободный ход педали 35—50 мм.

На переднем кронштейне топливного бака устанавливают и закрепляют фильтр-отстойник. Устанавливают в кронштейны и закрепляют топливный бак, фильтр-отстойник и топливный насос.

Отвернув пробки наливных отверстий, заливают трансмиссионное автомобильное масло (см. Приложения) в картеры заднего моста и коробки передач. Через пресс-масленки смазывают узлы головок рулевых тяг, подшипники вилки выключения сцепления, оси

педали сцепления, стебля крюка буксирного прибора, шкворней поворотных цапф, пальцев передней и задней подвесок, валов разжимных кулаков.

Устанавливают и закрепляют на передней поперечине рамы радиатор в сборе с рамкой подвески, кожухом вентилятора, жалюзи, масляным радиатором. С помощью хомутов соединяют шланги патрубка водяной рубашки и колена подводящего патрубка с патрубками радиатора. Также шлангами с трубками соединяют патрубки масляного радиатора с масляным картером двигателя и нижней секцией масляного насоса. Шлангами низкого и высокого давления соединяют бачок и корпус насоса с гидроусилителем рулевого механизма.

Затем устанавливают и закрепляют кабины в сборе с арматурой, электрооборудованием, отопителем, облицовкой радиатора, крыльями, подножками, капотом и колонкой рулевого управления. На рычаг переключения передач навертывают рукоятку, соединяют верхнюю и нижнюю части педали сцепления. Соединяют трубки пневматической системы с регулятором давления и воздушным манометром.

Далее соединяют провода с соответствующими узлами и датчиками автомобиля.

К полу кабины крепят передний и задний коврики. К ступицам крепят передние и задние колеса. Устанавливают подушки и спинки сидений пассажира и водителя.

Затем отсоединяют продольную рулевую тягу от рулевого механизма и заправляют маслом систему гидроусилителя, предварительно повернув рулевое колесо в крайнее левое положение. Масло доливают до тех пор, пока при вращении рулевого колеса от одного крайнего положения до другого не будет залито не менее 2,5 л. Затем включают двигатель и на режиме холостого хода доливают масло до уровня метки, вращая рулевое колесо от одного крайнего положения до другого и удерживая его в этих положениях в течение 2—3 с с усилием 100 Н. Заливку масла заканчивают при прекращении выхода пузырьков воздуха из системы через масло в бачке насоса гидроусилителя. После заправки маслом закрепляют крышку бачка насоса, устанавливают сошку на валу рулевого механизма, предварительно совместив их метки..

Далее готовят собранный автомобиль к испытанию. Перед испытанием автомобиль подвергают внешнему осмотру. При осмотре проверяют комплектность, качество сборки, исправное действие и правильность регулировки отдельных механизмов и приборов, а также готовность к испытательному пробегу.

Проверяют состояние дверей. Они должны легко открываться, плотно закрываться и не иметь перекосов. Стекла дверей должны плавно опускаться и подниматься подъемными механизмами. Проверяют качество сборки капота двигателя. Он должен плотно закрываться, легко подниматься, опускаться и удерживаться в поднятом положении.

Обращают внимание на монтаж передних колес, которые не

должны иметь ощутимого люфта при боковом качании. Проверяют работу приборов освещения и сигнализации, а также надежность крепления всех резьбовых соединений. Затем автомобиль полностью заправляют (водой, топливом, маслом), проверяют правильность подсоединения проводов зажигания и регулируют фары. Заправку осуществляют в соответствии с заводской инструкцией.

 

Практическая работа №21

Цель: Сформировать практические навыки по ремонту механизмов подъема платформы автомобиля-самосвала.

 

Теоретическая часть

Подъемный механизм состоит из телескопического цилиндра с плунжером, шестеренчатого масляного насоса, крана управления, коробки отбора мощности, масляного бака и трубопроводов. При опущенной грузовой платформе плунжер гидравлического подъемника находится у днища цилиндра, а масло — в полости под плунжером и в баке. Подъем платформ осуществляется при помощи масляного насоса, подающего масло в полость под плунжер. В результате давления, создаваемого насосом, плунжер перемещается и поднимает платформу. Опускается грузовая платформа под действием собственного веса, при этом из-под плунжера масло выдавливается и поступает в масляный бак. Масляный насос, нагнетающий масло в гидравлический подъемник, приводится в действие от одноступенчатой коробки отбора мощности и установлен на ее корпусе.

При перемещении рычага, находящегося в кабине автомобиля, вводится в зацепление промежуточная шестерня коробки отбора мощности с большой шестерней блока шестерен заднего хода коробки передач автомобиля. Подъем, опускание и удержание платформы в поднятом положении осуществляется при помощи крана управления, приводимого в действие тем же рычагом.

Чтобы поднять платформу автомобиля-самосвала, нужно: открыть запор заднего борта, рычаг коробки передач поставить в нейтральное положение, нажать на педаль сцепления и рычагом включить коробку отбора мощности, а затем, увеличив частоту вращения коленчатого вала двигателя до указанной в инструкции, плавно отпустить педаль сцепления. При необходимости прекращения подъема и фиксирования платформы в поднятом положении следует нажать на педаль сцепления и рычаг установить в положение «Стоп». Опускание платформы осуществляется переводом рычага в положение «Спуск».

Гидравлическая система подъемного механизма заполняется маслом, имеющим небольшую вязкость. Проверка уровня масла и доливка производятся ежедневно, а замена — при переходе с одного сезона эксплуатации на другой.

Безопасная работа и обслуживание подъемного механизма автомобиля-самосвала могут быть обеспечены при исправном ограничительном тросе подъема платформы и исправном упоре. Не допускается эксплуатация автомобиля-самосвала при неисправном ограничительном тросе и техническое обслуживание без надежно установленного упора. Запрещается передвижение автомобиля-самосвала с включенной коробкой отбора мощности, с поднятой платформой и открытым задним бортом, а также поднятие платформы при движении автомобиля.

Прицепы

Прицеп состоит из платформы, рамы, осей с колесами, рессор, поворотного и сцепного устройств. Прицепы могут быть одно-, двух- или многоосные. Прицепы, на которых перевозят особо тяжелые грузы, имеют, как правило, большое количество колес (катков). Поворотное устройство, которое состоит из поворотного круга и передней оси с подрамником, устанавливается на прицепах, имеющих больше одной оси. На поворотный круг подрамника опирается вторым кругом передняя часть рамы прицепа. Между двумя поворотными кругами уложены шарики или ролики, облегчающие поворот передней оси прицепа.

Полуприцеп своей передней частью опирается на седельное сцепное устройство, установленное на раме автомобиля-тягача, а сзади имеет одну или несколько осей в зависимости от грузоподъемности. В передней части полуприцепа установлены убирающиеся упоры, при помощи которых полуприцеп поддерживается в горизонтальном положении, когда он отцеплен от автомобиля-тягача. Полуприцеп не имеет поворотного приспособления, а поворот обеспечивается седельным сцепным устройством автомобиля-тягача.

Прицеп-роспуск, применяемый для перевозки длинномерных грузов, представляет собой небольшую раму, установленную на одной или двух осях с колесами и рессорной подвеской, к которой жестко прикреплено дышло.

Поворот обеспечивается установленным на раме повортным брусом, на который укладывают длинномерный груз. Такой же поворотный брус, на который опирается передний конец длинномерного груза, установлен на автомобиле-тягаче. Если длина перевозимого груза значительная, то прицеп-роспуск оборудуют устройством, обеспечивающим поворот его колес.

Все двухосные прицепы и полуприцепы имеют тормоза, приводимые в действие от тормозной системы автомобиля-тягача, габаритные фонари, указатели поворота и стоп-сигналы. Номерной знак прицепа освещается. Шланги пневматического привода тормозов после сцепки автомобиля с прицепом (полуприцепом) соединяют с соединительной головкой, а электропровод питания световых сигналов и габаритных фонарей подсоединяют к розетке. Для обеспечения поступления воздуха в тормозную систему прицепа надо открыть разобщительный кран.

 

 

 

 

 


Скачано с www.znanio.ru

Департамент образования Ямало-Ненецкого автономного округа государственное бюджетное профессиональное образовательное учреждение

Департамент образования Ямало-Ненецкого автономного округа государственное бюджетное профессиональное образовательное учреждение

Методические рекомендации для обучающихся по выполнению практических работ являются частью основной профессиональной программы

Методические рекомендации для обучающихся по выполнению практических работ являются частью основной профессиональной программы

ПЕРЕЧЕНЬ ПРАКТИЧЕСКИХ РАБОТ

ПЕРЕЧЕНЬ ПРАКТИЧЕСКИХ РАБОТ

Монтаж и демонтаж шин. 2.

Монтаж и демонтаж шин. 2.

УВАЖАЕМЫЙ СТУДЕНТ!

УВАЖАЕМЫЙ СТУДЕНТ!

Введение Содержанием данного документа являются методические указания и индивидуальные задания для выполнения практических работ по дисциплине профессионального цикла

Введение Содержанием данного документа являются методические указания и индивидуальные задания для выполнения практических работ по дисциплине профессионального цикла

Организация и порядок проведения практических работ

Организация и порядок проведения практических работ

Практические работы по МДК.01

Практические работы по МДК.01

Под столешницей расположены выдвижные ящики для хранения инструментов, мелких деталей и технической документации

Под столешницей расположены выдвижные ящики для хранения инструментов, мелких деталей и технической документации

Во время работы спецодежда работающего должна быть аккуратной и чистой

Во время работы спецодежда работающего должна быть аккуратной и чистой

Детали, которые имеют индивидуальную укладку, обычно располагаются в той таре, в которой они доставляются к рабочему месту

Детали, которые имеют индивидуальную укладку, обычно располагаются в той таре, в которой они доставляются к рабочему месту

Ящик с набором слесарных инструментов: 1 - щетка, 2 - скребок для очистки напильников, 3 - отвертка, 4 - крейцмейсель, 5 - зубило, 6 -…

Ящик с набором слесарных инструментов: 1 - щетка, 2 - скребок для очистки напильников, 3 - отвертка, 4 - крейцмейсель, 5 - зубило, 6 -…

Оформить отчет по прилагаемой форме

Оформить отчет по прилагаемой форме

Металл подвергается правке как в холодном, так и в нагретом состоянии

Металл подвергается правке как в холодном, так и в нагретом состоянии

Рис. 5. Правка металла: а — проверка на глаз, б — момент правки

Рис. 5. Правка металла: а — проверка на глаз, б — момент правки

Рис. 6. Приемы рихтовки заготовок из листового (а), полосового (б, в) и пруткового (г) материала

Рис. 6. Приемы рихтовки заготовок из листового (а), полосового (б, в) и пруткового (г) материала

Затем надеть на левую руку рукавицу и положить деталь на рихтовочную бабку, установленную на деревянном столе

Затем надеть на левую руку рукавицу и положить деталь на рихтовочную бабку, установленную на деревянном столе

Рубкой называется операция, при которой с помощью зубила и слесарного молотка с заготовки удаляют слои металла или разрубают заготовку

Рубкой называется операция, при которой с помощью зубила и слесарного молотка с заготовки удаляют слои металла или разрубают заготовку

Рис. 1 Зубило слесарное При рубке надо стоять у тисков устойчиво вполоборота к ним, корпус должен находиться левее от тисков, левая нога вперед, правая назад

Рис. 1 Зубило слесарное При рубке надо стоять у тисков устойчиво вполоборота к ним, корпус должен находиться левее от тисков, левая нога вперед, правая назад

При резке металла пользуются различными инструментами: кусачками, ножницами, ножовками, труборезами

При резке металла пользуются различными инструментами: кусачками, ножницами, ножовками, труборезами

Электрические ножницы И-31

Электрические ножницы И-31

Электромеханическая ножовка

Электромеханическая ножовка

Для резки труб применяется труборез

Для резки труб применяется труборез

Напильник ( а) представляет собой стальной брусок определенного профиля и длины, на поверхности которого имеется насечка (нарезка)

Напильник ( а) представляет собой стальной брусок определенного профиля и длины, на поверхности которого имеется насечка (нарезка)

Напильники с насечкой №0 и 1 (драчевые) имеют наиболее крупные зубья и служат для грубого (чернового) опиливания с точностью 0,5—0,2 мм

Напильники с насечкой №0 и 1 (драчевые) имеют наиболее крупные зубья и служат для грубого (чернового) опиливания с точностью 0,5—0,2 мм

Опиливание металла При опиливании заготовку закрепляют в тисках, при этом опиливаемая поверхность долж­на выступать над уровнем гу­бок тисков на 8—10 мм

Опиливание металла При опиливании заготовку закрепляют в тисках, при этом опиливаемая поверхность долж­на выступать над уровнем гу­бок тисков на 8—10 мм

Опиливание начинают, как правило, с проверки припуска на обработку, который мог бы обеспечить изготовление детали по размерам, указанным на чертеже

Опиливание начинают, как правило, с проверки припуска на обработку, который мог бы обеспечить изготовление детали по размерам, указанным на чертеже

Размеченную заготовку вставляют в рамку, слегка прижимая ее винтами к внутренней стенке рамки

Размеченную заготовку вставляют в рамку, слегка прижимая ее винтами к внутренней стенке рамки

Контроль правильности внут­реннего угла производится также угольником

Контроль правильности внут­реннего угла производится также угольником

Пройму средних размеров обсверливают по контуру свер­лом диаметром 3—5 мм вблизи разметочных линий, затем крейцмейселем или зубилом прорубают оставшиеся перемычки

Пройму средних размеров обсверливают по контуру свер­лом диаметром 3—5 мм вблизи разметочных линий, затем крейцмейселем или зубилом прорубают оставшиеся перемычки

При опиливании металла ручными и механизированными ин­струментами следует соблюдать правила техники безопасности

При опиливании металла ручными и механизированными ин­струментами следует соблюдать правила техники безопасности

Что такое надфили и для чего они служат? 7

Что такое надфили и для чего они служат? 7

Рис. 1 Инструменты для обработки отверстий: а — спиральное сверло; б — цилиндрический зенкер; в — конические (угловые) зенковки; г — торцовая зенковка (цековка); д…

Рис. 1 Инструменты для обработки отверстий: а — спиральное сверло; б — цилиндрический зенкер; в — конические (угловые) зенковки; г — торцовая зенковка (цековка); д…

Зенкер (рис. 1, б) — стержень из стали марок

Зенкер (рис. 1, б) — стержень из стали марок

Рис. 3 Ручная и электрические дрели: а — ручная (1 — упор; 2 — корпус с зубчатой передачей; 3 — привод с рукояткой; 4 —…

Рис. 3 Ручная и электрические дрели: а — ручная (1 — упор; 2 — корпус с зубчатой передачей; 3 — привод с рукояткой; 4 —…

Выводят сверло из отверстия, выключают двигатель и отсоединяют машинку от электросети

Выводят сверло из отверстия, выключают двигатель и отсоединяют машинку от электросети

Используя в работе электрические сверлильные машинки, нужно соблюдать следующие правила электробезопасности

Используя в работе электрические сверлильные машинки, нужно соблюдать следующие правила электробезопасности

При этом сторона треугольника ВС составит шаг винтовой линии,

При этом сторона треугольника ВС составит шаг винтовой линии,

Как определяют число ходов резьбы и какая зависимость между ходом, шагом и заходом?Ход — это осевое перемещение винта на один его оборот

Как определяют число ходов резьбы и какая зависимость между ходом, шагом и заходом?Ход — это осевое перемещение винта на один его оборот

Трубная (цилиндрическая) резь­ба — это дюймовая с мелким шагом

Трубная (цилиндрическая) резь­ба — это дюймовая с мелким шагом

M = Pd = 20 × 0,4=8Н ∙ м. 9

M = Pd = 20 × 0,4=8Н ∙ м. 9

Рис. 5. Приёмы удаления сломанных метчиков из отверстия

Рис. 5. Приёмы удаления сломанных метчиков из отверстия

Причинами неудовлетвори­тельного качества нарезаемой резьбы, часто приводящему к браку, являются: 1

Причинами неудовлетвори­тельного качества нарезаемой резьбы, часто приводящему к браку, являются: 1

Практическая работа №9 Цель работы:

Практическая работа №9 Цель работы:

Длина заклёпки l (в мм) с полукруглой головкой (рис

Длина заклёпки l (в мм) с полукруглой головкой (рис

Определите длину стержня заклёпки с полукруглой головкой с помощью номограммы (рис

Определите длину стержня заклёпки с полукруглой головкой с помощью номограммы (рис

D = d + 0,2 мм; в) для стальных заклёпок диаметр заклёпок выбирается по справочным таблицам

D = d + 0,2 мм; в) для стальных заклёпок диаметр заклёпок выбирается по справочным таблицам

В чем особенности показанных на рис

В чем особенности показанных на рис

Какими должны быть припуски на шабрение для плоскостей и отвер­стий?

Какими должны быть припуски на шабрение для плоскостей и отвер­стий?

Притирка: а — плоский притир с канавками, б — плоский притир гладкий, в — шаржирование плоского притира, г — шаржирование круглого притира

Притирка: а — плоский притир с канавками, б — плоский притир гладкий, в — шаржирование плоского притира, г — шаржирование круглого притира
Материалы на данной страницы взяты из открытых истончиков либо размещены пользователем в соответствии с договором-офертой сайта. Вы можете сообщить о нарушении.
11.11.2021